You are on page 1of 140

DELHI ACADEMY OF MEDICAL SCIENCES PVT. LTD.

ANATOMY 1.

MAHARASTRA PAPER - 2011

Ans: Ref:

2.

The process by which the three germ layers are formed during the development of the embryo is called a) gastrulation b) implantation c) fertilization d) ovulation a) gastrulation Langmans Medical Embryology; 9/e, pg 65 The most characteristic event occurring during the third week of gestation is gastrulation, the process that establishes all three germ layers (ectoderm, mesoderm and endoderm) in the embryo. Gastrulation begins with the formation of the primitive streak on the surface of the epiblast. The cells of the epiblast migrate towards the primitive streak. Upon arrival in the region of the streak, they become flask-shaped, detach form the epiblast and slip beneath it. This inward movement is known as invagination. Once the cells have invaginated, some dislocate the hypoblast, creating the embryonic endoderm; and others come to lie between the epiblast and the newly formed endoderm to form the mesoderm. Cells in the remaining epiblast then form ectoderm. Thus, the epiblast, through the process of gastrulation, is the source of all of the germ layers. The paramesonephric duct in females gives rise to all of the following except a) Fallopian tubes b) Ovary c) Cervix d) Upper part of vagina b) Ovary Inderbir Singhs Human Embryology; 7/e, pg 267 The gonads (testes and ovary) are derived from the coelomic epithelium covering the nephrogenic

Ans: Ref: cord.

In the female, the paramesonephric ducts give origin to the uterine (Fallopian) tubes, the uterus and part of the vagina. When traced caudally, they cross to the medial side of the mesonephric ducts. Here, the ducts of the two sides meet and fuse in the midline to form the uterovaginal canal. Structures developing from mesonephric (Wolffian) duct: Males Females Ureteric bud Collecting tubules, calyces, Ureteric bud Collecting tubules, calyces, pelvis, ureter pelvis, ureter Trigone of bladder Trigone of bladder Posterior wall of prostatic urethra, cranial to the Posterior wall of prostatic urethra, cranial to the opening of the ejaculatory ducts opening of the ejaculatory ducts Epididymis Remnants: Appendix of epididymis Epoophoron (Gartners duct) Ductus deferens Paroophoron Ejaculatory ducts Prostate (mesodermal part) Seminal vesicles Remnants: Superior aberrant ductule (epigenital tubercule) Inferior aberrant ductule Paradidymis (paragenital tubercle) Structures developing from paramesonephric (Mullerain) duct: Males Appendix of testis Prostatic utricle

Females Fallopian tubes Uterus Part of vagina

3.

Ans: Ref:

Myelin synthesis in CNS is carried out by a) Microglia b) Astrocytes c) Schwann cells d) Oligodendocytes d) Oligodendocytes Ganongs Review of Medical Physiology; 23/e, pg 80 There are two major types of glial cells in the vertebrate nervous system: macroglia and microglia. Macroglia:

25853434, 42433051, 9873314110, 9953550295, 8447461114

DELHI ACADEMY OF MEDICAL SCIENCES PVT. LTD.

MAHARASTRA PAPER - 2011

There are three types of macroglia: oligodendrocytes, Schwann cells and astrocytes. Oligodendrocytes and Schwann cells are involved in myelin formation around axons in the CNS and peripheral nervous system, respectively. Astrocytes, which are found throughout the brain, are of two types: Fibrous astrocytes, which contain many intermediate filaments, are found primarily in the white matter. Protoplasmic astrocytes are found in gray matter and have a granular cytoplasm. Both types send processes to blood vessels, where they induce capillaries to form the tight junctions making up the blood-brain barrier. They also send processes that envelop synapses and the surface of nerve cells. Microglia are scavenger cells that resemble tissue macrophages and remove debris resulting from injury, infection and disease. Microglia arise from macrophages outside of the nervous system and are physiologically and embryologically unrelated to other neural cell types. 4. Corpora arenacea are seen in which of the following organs? a) Posterior pituitary gland b) Anterior pituitary gland c) Pineal gland d) Prostate gland Ans: c) Pineal gland Ref: Lasts Anatomy: Regional and Applied; 10/e, pg 464. The pineal gland projects from the posterior wall of the third ventricle, lying above the superior colliculi of the midbrain. It is a soft, conical body, less than half a centimeter long, and is one of the few regions with no blood-brain barrier. It contains a number of corpora arenacea. These calcify, and to such an extent that after the age of 40 years, they may throw a tiny shadow in the radiographs of the skull. A displaced calcified pineal indicates a space-occupying lesion above the tentorium. Note: Corpora amylacea are found in the prostate gland (option d) 5. Crypts of Leiberkuhn are seen in which of the following organ? a) Tonsil b) Uterine tubes c) Gall bladder d) Duodenum d) Duodenum B. D. Chaurasias Human Anatomy, Volume 2: Lower Limb and Abdomen; 4/e, pg 250. Histology of Duodenum: Mucous membrane: shows evaginations in the form of villi and invaginations to form crypts of Leiberkuhn. The lining of the villi is of columnar cells with microvilli. Muscularis mucosa comprises of two layers. Submucosa: is full of mucous secreting Brunners glands. Musclaris externa: comprises inner circular and outer longitudinal layers of muscle fibres Serosa: The outermost layer is mostly connective tissue.

Ans: Ref: 6.

Which of the following muscles is inserted into the rough impression on the anterior surface of greater trochanter? a) Gluteus maximus b) Gluteus minimus c) Gluteus medius d) Piriformis Ans: b) Gluteus minimus Ref: B. D. Chaurasias Human Anatomy, Volume 2: Lower Limb and Abdomen; 4/e, pg 19 Muscle attachments on the greater trochanter of femur: 1. Piriformis: is inserted into the apex. 2. Gluteus minimus: is inserted into the rough lateral part on the anterior surface. 3. Obturator internus and the two gemelli: are inserted into the upper rough impression on the medial surface 4. Obturator externus: is inserted into the trochanteric fossa. 5. Gluteus medius: is inserted into the ridge on the lateral surface. Ref: B. D. Chaurasias Human Anatomy, Volume 2: Lower Limb and Abdomen; 4/e, pg 73 Muscle Origin Insertion Gluteus maximus Outer slope of dorsal segment of iliac crest Deep fibres of the lower part gluteal tuberosity Posterior gluteal line Greater part of the muscle Posterior part of gluteal surface of ilium behind iliotibial tract the posterior gluteal line Aponeurosis of erector spinae Dorsal surface of lower part of sacrum Side of coccyx

25853434, 42433051, 9873314110, 9953550295, 8447461114

DELHI ACADEMY OF MEDICAL SCIENCES PVT. LTD.


Sacrotuberous ligament Fascia covering gluteus maximus Gluteal surface of ilium between the anterior and inferior gluteal lines Gluteal surface of ilium between the anterior and posterior gluteal lines Pelvic surfaces of middle 3 pieces of sacrum (by 3 digitations) Upper margin of greater sciatic notch and the adjoining areas of the sacroiliac joint and sacrotuberous ligament

MAHARASTRA PAPER - 2011

Gluteus minimus

Gluteus medius Piriformis

Greater trochanter of femur, on a ridge on the lateral part of anterior surface Greater trochanter of femur, on oblique ridge on lateral surface Apex of greater trochanter of femur

7.

Ans: Ref: 1. 2. 3. 4. 5. 6. 8.

All of the following veins drain into the IVC except a) Right renal vein b) Left renal vein c) Right gonadal vein d) Left gonadal vein d) Left gonadal vein B. D. Chaurasias Human Anatomy, Volume 2: Lower Limb and Abdomen; 4/e, pg 316 Tributaries of the IVC The common iliac veins formed by the union of the external and internal iliac veins. Each common iliac vein receives an iliolumbar vein. The median sacral vein joins the left common iliac vein. The third and fourth lumbar veins open on the posterior aspect of IVC. The veins of the left side cross behind the aorta to reach the IVC. The first and the second lumbar veins may end up in the third lumbar vein, the ascending lumbar vein, the azygos vein (on the right side) or the hemiazygos vein (on the left side). The ascending lumbar vein is an anastomotic channel which connects the lateral sacral, iliolumbar and subcostal veins. On joining the subcostal vein, it forms the azygos vein on the right side and the hemiazygos vein on the left side. The right testicular/ovarian vein opens just below the entrance of the renal veins. The left gonadal vein drains into the left renal vein. The renal veins open just below the transpyloric plane. Each renal vein lies in front of the corresponding artery. The left renal vein receives the left suprarenal and left gonadal veins. The right suprarenal vein emerges from the hilum of the gland and soon opens into the IVC. The left suprarenal vein drains into the left renal vein. The hepatic veins three large and many small veins open directly into the anterior surface of the IVC just before it pierces the diaphragm.

The free margin of lesser omentum forming the anterior boundary of epiploic foramen contains which of the following? a) hepatic artery, portal vein and CBD b) gastric lymph nodes c) IVC d) right phrenic nerve Ans: a) hepatic artery, portal vein and CBD Ref: B. D. Chaurasias Human Anatomy, Volume 2: Lower Limb and Abdomen; 4/e, pg 226 The right free margin of the lesser omentum contains The hepatic artery proper The portal vein The bile duct Lymph nodes and lymphatics and The hepatic plexus of nerves. All of these are enclosed in a perivascular fibrous sheath. Along the lesser curvature of the stomach and along the upper border of the adjoining part of duodenum, it contains The right gastric vessels The left gastric vessels The gastric group of lymph nodes and lymphatics and Branches from the gastric nerves. 9. The most common location of spleniculi is a) Broad ligament of uterus b) Hilum of spleen

25853434, 42433051, 9873314110, 9953550295, 8447461114

DELHI ACADEMY OF MEDICAL SCIENCES PVT. LTD.

MAHARASTRA PAPER - 2011

Ans: Ref: 1. 2. 3. 10.

c) Leinorenal ligament d) Spermatic cord c) Leinorenal ligament B. D. Chaurasias Human Anatomy, Volume 2: Lower Limb and Abdomen; 4/e, pg 283 Accessory spleens or spleniculi: These may be found in Derivatives of dorsal mesogastrium Gastroslenic ligament Lienorenal ligament Gastrophrenic ligament and Greater omentum Broad ligament of uterus Spermatic cord. Anorectal ring is made up of all of the following except a) Puborectalis b) Superficial external anal sphincter c) Deep external anal sphincter d) Internal anal sphincter b) Superficial external anal sphincter B. D. Chaurasias Human Anatomy, Volume 2: Lower Limb and Abdomen; 4/e, pg 383 Anorectal ring: This is a muscular ring present at the anorectal junction. It is formed by the fusion of Puborectalis Deep external anal sphincter and Internal anal sphincter. The ring is less marked anteriorly where the fibres of puborectalis are absent. It is easily felt by a finger in the anal canal. Surgical division of this ring results in rectal incontinence. All of the following muscles are anatomically related to the Eustachian tube except a) Tensor tympani b) Palatoglossus c) Levator veli palatini d) Tensor veli palatini a) Tensor tympani & b) Palatoglossus P. L. Dhingras Diseases of the Ear, Nose and Throat; 3/e, pg 73 Muscles related to the Eustachian tube: Three muscles are related to the tube: Tensor veli palatini Levator veli palatini and Salpingopharyngeus. The medial fibres of the tensor veli palatini are attached to the lateral lamina of the tube, and when they contract, they help to open the tubal lumen. These fibres have therefore also been called dilator tubae muscle. The exact role of levator veli palatini and salpingopharyngeus to open the tube is uncertain. It is believed that levator veli palatini muscle, which runs inferior and parallel to the cartilaginous part of the tube forms a bulk under the medial lamina, and during contraction pushes it upwards and medially; thus assisting in opening the tube. Killians dehiscence is a posterior projection seen between which of the following muscles? a) Palatoglossus and palatopharyngeus b) Superior and middle constrictor c) Middle and inferior constrictor d) Thyropharyngeus and cricopharyngeus d) Thyropharyngeus and cricopharyngeus P. L. Dhingras Diseases of the Ear, Nose and Throat; 3/e, pg 290 The inferior constrictor muscle has two parts: thyropharyngeus with oblique fibres and cricopharyngeus with transverse fibres. Between these two parts exists a potential gap known as Killians dehiscence. It is also called the gateway of tears as perforation can occur at this site during oesophagoscopy. It is also the site of herniation of pharyngeal mucosa in cases of pharyngeal pouch. All of the following are intrinsic muscles of the larynx that lie within the laryngeal cartilage framework except a) Cricothyroid b) Thyroepiglottic c) Interarytenoid

Ans: Ref:

11.

Ans: Ref:

12.

Ans: Ref: 13.

25853434, 42433051, 9873314110, 9953550295, 8447461114

DELHI ACADEMY OF MEDICAL SCIENCES PVT. LTD.

MAHARASTRA PAPER - 2011

d) Superior constrictor d) Superior constrictor P. L. Dhingras Diseases of the Ear, Nose and Throat; 3/e, pg 337-338 Muscles of the larynx: They are of two types: intrinsic, which attach the laryngeal cartilages to each other; and extrinsic, which attach larynx to the surrounding structures. Intrinsic muscles of the larynx: They may act on the vocal cords or the laryngeal inlet. 1. Muscles acting on the vocal cords: o Abductors: Posterior cricoarytenoid o Adductors: Lateral cricoarytenoid Interarytenoid (Transverse arytenoids) o Tensors: Cricothyroid Vocalis (internal part of thyroarytenoid) 2. Muscles acting on the laryngeal inlet: o Openers of the inlet: Thyroepiglottic (part of thyroarytenoid) o Closers of the inlet: Interarytenoid (oblique part) Aryepiglottic (posterior oblique part of interarytenoid) Extrinsic muscles: They connect the larynx to the neighbouring structures. They are divided into elevators and depressors of the larynx. 1. Elevators of the larynx Primary elevators: act directly as they are attached to the thyroid cartilage. Stylopharyngeus Salpingopharyngeus Palatopharyngeus and Thyrohyoid Secondary elevators: act indirectly as they are attached to the hyoid bone. Mylohyoid (main) Digastric Stylohyoid and Geniohyoid. 2. Depressors: they include Sternohyoid Sternothyroid and Omohyoid Ans: Ref: 14. Nucleus ambiguus contains the centre for fibres of all of the following nerves except a) Hypoglossal n. b) Vagus n. c) Accessory n. d) Glossopharyngeal a) Hypoglossal n. B. D. Chaurasias Human Anatomy, Volume 3: Head, Neck and Brain; 4/e, pg 324.

Ans: Ref:

The nucleus ambiguus lies deep to the reticular formation of the medulla. It gives origin to the motor fibres of the cranial nerves IX, X and XI. The nuclei of several cranial nerves are seen in the floor of the fourth ventricle; the hypogloassal nucleus lies in a paramedian position. PHYSIOLOGY 15. The correct formula for Laplaces law (where P = pressure inside a hollow viscus, T = wall tension of the viscus and r = radius of the viscus) is a) P = T/r b) P = 2T/r c) P = r/T d) P = R/2T a) P = T/r Ganongs Review of Medical Physiology; 23/e, pg 542-543 It is perhaps surprising that structures as thin-walled and delicate as the capillaries are not more prone to rupture. The principle reason for their relative invulnerability is their small diameter. The protective effect of small size in this case is an example of the operation of the law of Laplace, an important physical principle. The law states that the tension in the wall of a cylinder (T) is equal to the product of the transmural pressure (P) and the radius (r) divided by the wall thickness (w):

Ans: Ref: o o o o

25853434, 42433051, 9873314110, 9953550295, 8447461114

DELHI ACADEMY OF MEDICAL SCIENCES PVT. LTD.

MAHARASTRA PAPER - 2011

T = Pr/w The transmural pressure is the pressure inside the cylinder minus the pressure outside the cylinder, but because tissue pressure in the body is low, it can generally be ignored and P equated to the pressure inside the viscus. o In a thin walled viscus, w is very small and it can be ignored too, but it becomes a significant factor in vessels such as arteries. o Therefore, in a thin-walled viscus, P = T divided by the two principal radii of curvature of the viscus: P = T (1/r1 + 1/r2) o In a sphere, r1 = r2, so P = 2T/r o In a cylinder such as blood vessel, one radius is infinite, so P = T/r Now, why the answer is option a and not b: Note that while describing the law, in general, Ganong makes use of the word cylinder. So, it is assumed that a hollow viscus is usually cylindrical in nature. This accords to the fact that the blood vessels, gut and other organs are cylindrical in nature. Hence, whenever the hollow viscus is not specified, it is taken to be a cylinder. o Nernst potential for K+ ion across the cell membrane is a) + 61 mV b) + 10 mV c) - 90 mV d) - 94 mV Ans: d) - 94 mV > c) - 90 mV Ref: Guyton & Halls Textbook of Medical Physiology; 10/e, pg 55 Contribution of the Potassium Diffusion Potential: We make the assumption that the only movement of ions through the membrane is diffusion of potassium ions. Because of the high ratio of potassium ions inside to outside, 35 to 1, the Nernst potential corresponding to this ratio is -94 mV because the logarithm of 35 is 1.54 and this times -61 mV is -94 mV. Therefore, if potassium ions were the only factor causing the resting potential, this resting potential inside the fiber would also be equal to -94 mV. Now, let us see what Ganong has to say Ref: Ganong, 23/e, Pg 8, Table 1-1. Concentration (mmol/L of H2O) Ion Inside cell Outside cell Equilibrium potential (mV) Na+ 15.0 150.0 +60 K+ 150.0 5.5 -90 Cl9.0 125.0 -70 Thus, according to Guyton, the answer is (d), but according to Ganong, it is (c). But the Nernst potential for Na+ as given by Guyton is +61 mV, while as given by Ganong is +60 mV. In the question, the option (a) is +61 mV. So, (my personal) interpretation is that the examiner must have picked up this question from Guyton, hence option (d), i.e. -94 mV is a better option to go with. 17. The correct equation for the equilibrium potential for Cl- ion is a) E = 61.5 log [Cl-]o [Cl-]i b) E = -61.5 log [Cl-]i [Cl-]o c) E = 61.5 log [Cl-]i [Cl-]o d) E = -93.5 log [Cl-]o [Cl-]i Ans: c) E = 61.5 log [Cl-]i [Cl-]o Ref: Ganongs Review of Medical Physiology, 23/e, pg 7 The membrane potential at which the equilibrium for an ion exists is the equilibrium potential. Its magnitude can be calculated from the Nernst equation, as follows: ECl = RT log [Cl-]o FZCl [Cl-]i where, ECl = equilibrium potential for ClR = gas constant T = absolute temperature F = the faraday (number of coulombs per mole of charge) ZCl = valence of Cl- (-1) [Cl-]o = Cl- concentration outside the cell [Cl-]i = Cl- concentration inside the cell Converting from the natural log to the base 10 log and replacing some of the constants with numerical values, the equation becomes: E = 61.5 log [Cl-]I at 37OC 16.

25853434, 42433051, 9873314110, 9953550295, 8447461114

DELHI ACADEMY OF MEDICAL SCIENCES PVT. LTD.


[Cl-]o

MAHARASTRA PAPER - 2011

The voltage gated Na+ channels are stimulated when the potential across the cell membrane reaches up to a) - 50 to 70 mV b) - 70 to 90 mV c) - 90 to 110 mV d) - 110 to 130 mV Ans: a) - 50 to 70 mV Ref: Guyton & Halls Textbook of Medical Physiology; 10/e, pg 56-57 Activation of the Sodium Channel: When the membrane potential becomes less negative than during the resting state, rising from -90 mV towards zero, it finally reaches a voltage, somewhere between -70 and -50 mV, that causes a sudden conformational change in the activation gate, flipping all the way to the open position. This is called the activated state; during this state, sodium ions can literally pour inwards through the channel, increasing the sodium permeability of the membrane, as much as 500-5000 fold. 18. Which of the following favour the movement of plasma into the interstitium from the capillaries? a) Negative interstitial fluid pressure b) Decreased plasma colloid oncotic pressure c) Increased capillary hydrostatic pressure d) All of the above Ans: d) All of the above Ref: Guyton & Halls Textbook of Medical Physiology; 10/e, pg 166 The four primary forces that determine whether fluid moves out of the blood into the interstitial fluid or in the opposite direction are called Starling forces in honour of the physiologist who first demonstrated their importance. They are 1. The capillary pressure (Pc), which tends to force the fluid outward through the capillary membrane. 2. The interstitial fluid pressure (Pif), which tends to force the fluid inward through the capillary membrane when Pif is positive but outward when Pif is negative. 3. The plasma colloid osmotic pressure (p), which tends to cause osmosis of fluid inwards through the capillary membrane. 4. The interstitial fluid colloid osmotic pressure (if), which tends to cause osmosis of fluid outward through the capillary membrane. 20. Water acts as a excellent biological medium for all the substances in the body due to all of the following features except a) b) ability to form covalent bonds c) favouring hydrophobic interactions d) ability to bind with hydrogen b) ability to form covalent bonds Harpers Illustrated Biochemistry; 27/e, pg 5. Water is an ideal biological solvent. Water molecules form dipoles. A water molecule is an irregular, slightly skewed tetrahedron with oxygen at its centre. It is a dipole, a molecule with electrical charge distributed asymmetrically about its structure. The strongly electronegative oxygen atom pulls electrons away from the hydrogen nuclei, leaving them with a partial positive charge, while its two unshared electron pairs constitute a region of local negative charge. Water molecules form hydrogen bonds. An unshared hydrogen nucleus covalently bound to an electron-withdrawing oxygen or nitrogen atom can interact with an unshared electron pair or another oxygen or nitrogen atom to form a hydrogen bond. Since water molecules contain both of these features, hydrogen bonding favours the self-association of water molecules into ordered arrays. Hydrogen bonding enables water to dissolve many organic biomolecules that contain functional groups which can participate in hydrogen bonding. The oxygen atoms of aldehydes, ketones and amides provide pairs of electrons that can serve as hydrogen acceptors and as donors of unshielded hydrogen atoms for the formation of hydrogen bonds. Covalent and non-covalent bonds stabilize biologic molecules. The covalent bond is the strongest force that holds molecules together. Non-covalent forces, while of lesser magnitude, make significant contributions to the structure, stability and functional competence of macromolecules in living cells. These forces, which can either be attractive or repulsive, involve interactions both within the biomolecule and between it and the water that forms the principal component of the surrounding environment. water forms non-covalent bonds and not covalent ones] Hydrophobic interactions: 19.

Ans: Ref: 1. 2. 3. [Thus, 4.

25853434, 42433051, 9873314110, 9953550295, 8447461114

DELHI ACADEMY OF MEDICAL SCIENCES PVT. LTD.

MAHARASTRA PAPER - 2011

5. 6.

Hydrophobic interaction refers to the tendency of non-polar compounds to self-associate in an aqueous environment. Self-association arises from the need to minimize energetically unfavourable interactions between nonpolar groups and water. Water molecules adjacent to a hydrophobic group are restricted in the number of orientations (degrees of freedom) that permit them to participate in the maximum number of energetically favourable hydrogen bonds. Electrostatic interactions. Interactions between charged groups help shape biomolecular structure. Electrostatic interactions between oppositely charged groups within or between biomolecules, termed as salt bridges often facilitate the binding of charged molecules and ions to proteins and nucleic acids. Van der Waals forces. Van der Waals forces arise from attractions between transient dipoles generated by the rapid movement of electrons on all neutral atoms. They are significantly weaker than hydrogen bonds but potentially extremely numerous. Which of the following parameters remains unchanged during isotonic exercise? a) Respiratory rate b) Stroke volume c) Total peripheral resistance d) Heart rate c) Total peripheral resistance Ganongs Review of Medical Physiology; 23/e, pg 101 Isotonic contraction: Contraction against a constant load with a decrease in muscle length (same tension) Isometric contraction: Contraction without an appreciable decrease in the length of the whole muscle. DeLisas Physical Medicine and Rehabilitation: Principles and Practice; 5/e, pg 1632-1633 Dynamic (isotonic) exercise effort is typically expressed in either absolute units (e.g. VO2 in litres per minute) or relative units (e.g. percentage of an individuals VO2max). Absolute units provide a measure of work performed per unit time, whereas relative units reflect the degree of effort or how strenuous the exercise feels. Most physiologic changes with exercise are more proportional to relative than to absolute work units within a certain intensity range. This includes heart rate, ventilation, sympathetic/parasympathetic nerve outflow, circulating hormones and core temperature. The one parameter that increases more in proportion to absolute work performed than relative intensity is cardiac output, which increases about 5 to 6 mL/min for each rise in VO2 of 1 L/min. Several factors may influence the magnitude of changes produced by dynamic exercise, such as body posture, age, gender, fitness level, disease state and mode of exercise (e.g. leg vs arm). Many of the cardiovascular adjustments to dynamic exercise are regulated by changes in autonomic nervous activity outflow. Parasympathetic tone exists at rest, and its withdrawl at the onset of exercise allows heart rate to rise. When work intensity reaches about 50% VO2max, parasympathetic withdrawl appears to be exhausted, and any further rise in heart rate is totally dependent on increased sympathetic nerve activity. In addition to increasing heart rate, sympathetic nerve activity increases myocardial contractility, mobilizes nutrients, influences several circulating hormone levels and contributes to blood flow redistribution by vasoconstriction in inactive regions. Although muscle sympathetic nerve activity appears to increase in active muscles, metabolic byproducts override this vasoconstriction effect to produce vasodilation. Control of the autonomic nervous system during exercise originates from both central and peripheral receptors located in the motor cortex, ergoreceptrs and arterial and cardiopulmonary baroreceptors. The rise in cardiac output (between 4- and 6-to 7-fold at maximal effort) with upright leg exercise stems from a rise in heart rate (2- to 3-fold) and stroke volume (about 1.5-2 fold). The rise in stroke volume results from increased myocardial preload and contractility. Preload increases as a result of enhanced venous return, which is brought on by venoconstriction and muscle contraction. An increase in contractility leads to more complete emptying of the heart (i.e. decreased left ventricular endsystolic volume), whereas an increase in preload increases left-ventricular end-diastolic volume. The net effect is increased stroke volume. Up to 80% of cardiac output can be distributed to the active muscles at maximum effort, compared with only about 20% of cardiac output being distributed to the muscles at rest. This marked blood flow redistribution is accomplished by arterial vasodilation in the active muscles and arterial vasoconstriction in other vascular regions (e.g. sphlancnic, inactive muscle, renal). Total systemic vascular resistance declines progressively with increasing work intensity. The precise mechanisms leading to vasodilation in the active muscle remain debatable but likely stem from changes in several local factors, including potassium ion, hydrogen ion, endothelium-derived relaxing factor, adenosine, osmolarity and others. Which of the following are rapidly adapting cutaneous receptors that respond to deep pressure?

21.

Ans: Ref: Ref:

22.

25853434, 42433051, 9873314110, 9953550295, 8447461114

DELHI ACADEMY OF MEDICAL SCIENCES PVT. LTD.

MAHARASTRA PAPER - 2011

a) Merkels discs b) Ruffinis end organs c) Pacinian corpuscles d) Free nerve endings Ans: c) Pacinian corpuscles Ref: Ganongs Review of Medical Physiology; 23/e, pg 150, 153 Touch and pressure are sensed by four types of mechanoreceptors. 1. Meissners corpuscles are dendrites encapsulated in connective tissue and respond to changes in texture and slow vibrations. 2. Merkel cells are expanded dendritic endings, and they respond to sustained pressure and touch. 3. Ruffini corpuscles are enlarged dendritic endings with elongated capsules, and they respond to sustained pressure. 4. Pacinian corpuscles consist of unmyelinated dendritic endings of a sensory nerve fibre, 2 m in diameter, encapsulated by concentric lamellae of connective tissue that give the organ the appearance of a cocktail onion. These receptors respond to deep pressure and fast vibration. Meissner and Pacinian corpuscles are examples of rapidly adapting (phasic) receptors, and Merkel cells and Ruffini endings are examples of slowly adapting (tonic) receptors. All of the following cause a shift in the oxygen dissociation curve to the left except a) Increased fetal haemoglobin concentration b) Increased pH c) Increased myoglobin concentration d) Increased CO2 concentration Ans: d) Increased CO2 concentration Ref: Ganongs Review of Medical Physiology; 23/e, pg 611 Three important conditions affect the oxygen-haemoglobin dissociation curve: the pH, the temperature and the concentration of 2,3-BPG. A rise in temperature or a fall in pH shifts the curve to the right. When the curve is shifted in this direction, a higher pO2 is required for Hb to bind a given amount of O2. Conversely, a fall in temperature or a rise in pH shifts the curve to the left and a lower pO2 is required to bind a given amount of O2. The pH of the blood falls as its CO2 content increases, so that when the pCO2 rises, the curve shifts to the right and the P50 rises. Most of the unsaturation of Hb that occurs in the tisues is secondary to the decline in the PO2, but an extra 1-2% unsaturation is due to the rise in pCO2 and the consequent shift of the dissociation curve to the right. Summary: Factors shifting O2-Hb dissociation curve to the: Right Left pH pH pCO2 pCO2 Temperature Temperature 2,3-BPG 2,3-BPG High altitude HbF Methaemoglobin / Myoglobin 24. In the heart, the repolarisation wave first starts from which of the following areas? a) Epicardial surface of apex b) Basal part of septum c) Papillary muscle d) Subendocardium a) Epicardial surface of apex Ganongs Review of Medical Physiology; 23/e, pg 492 Spread of cardiac excitation: Depolarization initiated in the SA node spreads radially through the atria, then converges on the AV node. Atrial depolarization is complete in about 0.1 s. Because conduction in the AV node is slow, a delay of about 0.1 s (AV nodal delay) occurs before excitation spreads to the ventricles. From the top of the septum, the wave of depolarization spreads in the rapidly conducting Purkinje fibres to all parts of the ventricles in 0.08-0.1 s. In humans, depolarization of the ventricular muscle starts at the left side of the interventricular septum and moves first to the right across the mid portion of the septum. The wave of depolarization then spreads down the septum to the apex of the heart. It returns along the ventricular walls to the AV groove, proceeding from the endocardial to the epicardial surface. The last parts of the heart to be depolarized are the posterobasal portion of the left ventricle, the pulmonary conus and the uppermost portion of the septum. 23.

Ans: Ref:

25853434, 42433051, 9873314110, 9953550295, 8447461114

DELHI ACADEMY OF MEDICAL SCIENCES PVT. LTD.

MAHARASTRA PAPER - 2011

[Note: As the wave of repolarization is exactly opposite to that of depolarization, it begins in the epicardium and proceeds towards the endocardium.] 25. Glucocorticoids cause an increase in which of the following? a) Lymphocytes b) Eosinophils c) Red blood cells d) None of these c) Red blood cells K. D. Tripathis Essentials of Medical Pharmacology; 6/e, pg 278 Glucocorticoids increase the number of RBCs, platelets and neutrophils in the circulation. They enhance the rate of destruction of lymphoid cells (T cells are more sensitive than B cells); but in man the effect on normal lymphoid tissue is modest. However, a marked lytic response is shown by malignant lymphatic cells; basis of their use in lymphomas. Apart from lymphocytes, they also decrease eosinophils and basophils. This is not due to destruction of these cells but due to their sequestration in tissues. Blood counts come back to normal in 24 hours. Thrombospondin is a a) Contractile protein b) Cytoskeletal protein c) Angiogenic protein d) Anti-angiogenic protein d) Anti-angiogenic protein http://en.wikipedia.org/wiki/Thrombospondin Thrombospondins (TSP) are secreted proteins with antiangiogenic abilities. It is a family of five proteins: TSP 1-5. TSP-1 was first isolated from platelets that had been stimulated with thrombin, and so was designated 'thrombin-sensitive protein'. Since its first recognition, functions for TSP-1 have been found in multiple biological processes including angiogenesis, apoptosis, activation of TGF- and immune regulation. TSP-1 inhibits the proliferation and migration of endothelial cells by interactions with CD36 expressed on the surface of these cells. Inhibitory peptides and fragments of TSP1 bind to CD36, leading to the expression of FAS ligand (FasL), which activates its specific, ubiquitous receptor, Fas. This leads to the activation of caspases and apoptosis of the cell. The half-life of normally circulating monocytes in the circulation is a) 10 12 hours b) 1 3 days c) 1 2 weeks d) 1 2 months b) 1 3 days Ganongs Review of Medical Physiology; 23/e, pg 65 Monocytes enter the blood from the bone marrow and circulate for about 72 hours. They then enter the tissues and become tissue macrophages. Their life span in the tissues is unknown, but bone marrow transplantation data in humans suggest that they persist for about 3 months. The function of Paneth cells is to secrete which of the following? a) mucus b) antibacterial like lysozyme c) alkaline fluid d) acidic fluid b) antibacterial like lysozyme Inderbir Singhs Textbook of Human Histology; 5/e, pg 247 Paneth cells (Zymogen cells) These cells are found only in the deeper parts of the intestinal crypts. They contain prominent eosinophilic secretory granules. On e/m, they are seen to contain considerable rough ER woth other organelles and some irregular microvilli The cells are rich in zinc. They are known to produce lysozyme which destroys bacteria. They may also produce other enzymes. Which of the following require/s co-transport with sodium for its/their absorption in the GI tract? a) Glucose

Ans: Ref: 26.

Ans: Ref: 27.

Ans: Ref: 28.

Ans: Ref: 29.

25853434, 42433051, 9873314110, 9953550295, 8447461114

10

DELHI ACADEMY OF MEDICAL SCIENCES PVT. LTD.

MAHARASTRA PAPER - 2011

b) Proteins c) Galactose d) All of these Ans: d) All of these Ref: Ganongs Review of Medical Physiology; 23/e, pg 453 The transport of most hexoses is dependent on Na+ in the intestinal lumen; a high concentration of Na+ on the mucosal surface of the cells facilitates and a low concentration inhibits sugar influx into the epithelial cells. This is because glucose and Na+ share the same co-transporter, or symport, the sodium-dependent glucose transporter (SGLT, Na+-glucose transporter). The members of this family are SGLT1 and SGLT2. SGLT1 also transports galactose. However, fructose utilizes a different mechanism. Its absorption is independent of Na+ or the transport of glucose and galactose. Ref: Ganongs Review of Medical Physiology; 23/e, pg 455 At least seven different transport systems transport amino acids into enterocytes. Five of these requires Na+ and co-transport amino acids and Na+ in a fashion similar to that of Na+ and glucose. Two of these five also require Cl-. In the other two systems, transport is independent of Na+ [The confusion in this question is created by the option (b), i.e. proteins; as it is the amino acids which are absorbed from the GIT and not intact proteins. However, since two of the three options, i.e. options (a) and (c) are both correct, the most appropriate answer in this case would be (d): all of these] 30. True statement regarding the relationship of somatostatin, insulin and glucagon is a) Somatostatin stimulates insulin release b) Insulin stimulates somatostain and glucagon release c) Glucagon inhibits insulin release d) Somatostatin inhibits insulin and glucagon release d) Somatostatin inhibits insulin and glucagon release Ganongs Review of Medical Physiology; 23/e, pg 330-331 Somatostatin 14 (SS 14) and its amino-terminal extended form (SS 28) are found in the D cells of the panctreatic islet. Both forms inhibit the secretion of insulin, glucagon and pancreatic polypeptide and act locally within the pancreatic islets in a paracrine fashion. SS 28 is more active than SS 14 in inhibiting insulin secretion, and it apparently acts via the SSTR5 receptor. Patients with somatostatin-secreting pancreatic tumours (somatostatinomas) develop hyperglycemia and other manifestations of diabetes that disappear when the tumour is removed. They also develop dyspepsia due to slow gastric emptying ad decreased gastric acid secretion; and gallstones, which are precipitated by decreased gall bladder contraction due to inhibition of CCK secretion. Parathyroid hormone (PTH) exerts its effects on calcium metabolism by acting on receptors present on the a) Osteoclasts b) Osteocytes c) Macrophages d) None of these b) Osteocytes Guyton & Halls Textbook of Medical Physiology; 10/e, pg 907 The cell membranes of both the osteoblasts and the osteocytes have receptor proteins for binding PTH. It seems that PTH can activate the calcium pump strongly, thereby causing rapid removal of calcium phosphate salts from those amorphous bone crystals that lie near the cells. PTH is believed to stimulate this pump by increasing the calcium permeability of the bone fluid side of the osteocytic membrane, thus allowing calcium ions to diffuse into the membrane cells from the bone fluid. Then, the calcium pump on the other side of the cell membrane transfers the calcium ions the rest of the way into the extracellular fluid. A much better known effect of PTH and one for which the evidence is much clearer is its activation of the osteoclasts. Yet the osteoclasts themselves do not have membrane receptor proteins for PTH. Instead, it is believed that the activated osteoblasts and osteocytes send a secondary but unknown signal to the osteoclasts, causing them to set about their usual task of gobbling up the bone over a period of weeks or months. Capacitation of sperms is an activating process that takes place in the a) Vas deferens b) Epididymis c) Urethra d) Female genital tract

Ans: Ref: 31.

Ans: Ref:

32.

25853434, 42433051, 9873314110, 9953550295, 8447461114

11

DELHI ACADEMY OF MEDICAL SCIENCES PVT. LTD.


Ans: Ref: Ref: 33.

MAHARASTRA PAPER - 2011

d) Female genital tract Ganongs Review of Medical Physiology; 23/e, pg 404 Once ejaculated into the female, the spermatozoa move up to the uterus to the isthmus of the fallopian tubes, where they slow down and undergo capacitation. This further maturation process involves two components: increasing the motility of the spermatozoa and facilitating their preparation for the acrosome reaction. However, the role of capacitation appears to be facilitatory rather than obligatory, because fertilization is readily produced in vivo. Guyton & Halls Textbook of Medical Physiology; On coming in contact with the fluids of the female genital tract, multiple changes occur that activate the sperm for the final processes of fertilization. These collective changes are called capacitation of the spermatozoa. This normally requires from 1 to 10 hours. Renal transport maximum (Tm) is defined for a) Glomerular filtration b) Urinary excretion c) Tubular reabsorption and secretion d) c) Tubular reabsorption and secretion Ganongs Review of Medical Physiology; 23/e, pg 648 Tubular function: Mechanisms of tubular reabsorption and secretion: Like transport systems elsewhere, renal transport systems have a maximal rate, or transport maximum (Tm), at which they can transport a particular solute. Thus, the amount of a particular solute transported is proportional to the amount present up to the Tm for the solute, but at higher concentrations, the transport mechanism is saturated and there is no appreciable increment in the amount transported. Guyton & Halls Textbook of Medical Physiology; Transport Maximum for Substances That Are Actively Reabsorbed. For most substances that are actively reabsorbed or secreted, there is a limit to the rate at which the solute can be transported, often referred to as the transport maximum. This limit is due to saturation of the specific transport systems involved when the amount of solute delivered to the tubule (referred to as tubular load) exceeds the capacity of the carrier proteins and specific enzymes involved in the transport process.

Ans: Ref: Ref:

In a normal individual, the slow-waves predominantly occur in the first third of the sleep during the night in the phases of a) NREM 3 and 4 b) NREM 1 c) NREM 1 and 2 d) REM Ans: a) NREM 3 and 4 Ref: Ganongs Review of Medical Physiology; 23/e, pg 234-235 There are two kinds of sleep: REM (rapid eye movement) and NREM (non-REM) sleep or slow-wave sleep. NREM sleep is divided into four stages. A person falling asleep enters stage 1. The EEG begins to show a low-voltage, mixed frequency pattern. A theta rhythm (4-7 Hz) can be seen at this early stage of slow-wave sleep. Stage 2 is marked by the appearance of sinusoidal waves called sleep spindles (12-14 Hz) and occasional high-voltage biphasic waves called L complexes. In stage 3, a high-amplitude delta rhythm (0.5-4 Hz) dominates the EEG waves. Maximum slowing with large waves is seen in stage 4. Thus, the characteristic of deep sleep is a pattern of rhythmic slow waves, indicating marked synchronization; it is sometimes referred to as slow-wave sleep. Note: REM sleep is characterized by beta waves. [Though slow waves are characteristic of Stage 4, the best option amongst the given options is option (a), Stages 3 and 4] 35. Which of the following is/are known to influence food intake? a) Leptin b) Neuropeptide Y c) Ghrelin d) All of these d) All of these (a): Leptin Ganongs Review of Medical Physiology; 23/e, pg 399 It has been argued for some time that a critical body weight must normally be reached for puberty to occur. Thus, for example, young women who engage in strenuous athletics lose weight and stop menstruating, as do girls with anorexia nervosa. If these girls start to eat and gain weight, they menstruate again; i.e. they go back through puberty.

34.

Ans: Option Ref:

25853434, 42433051, 9873314110, 9953550295, 8447461114

12

DELHI ACADEMY OF MEDICAL SCIENCES PVT. LTD.


Option Ref: Option Ref: Ref: 36.

MAHARASTRA PAPER - 2011

It now appears that leptin, the satiety-producing hormone secreted by fat cells, may be the link between body weight and puberty. Obese ob/ob mice that cannot make leptin are infertile, and their fertility is restored by injections of leptin. Leptin treatment also induces precocious puberty in immature female mice. However, the way leptin fits into the overall control of puberty remains to be determined. (b): Neuropeptide Y Ganongs Review of Medical Physiology; 23/e, pg 145 Neuropeptide Y is a polypeptide containing 36 amino acid residues that act on at least 2 of the 4 known Gprotein coupled receptors: Y1, Y2, Y4 and Y5. Neuropeptide Y is found throughout the brain and the autonomic nervous system. When injected into the hypothalamus, this polypeptide increases food intake. Inhibitors of neuropeptide Y synthesis decrease food intake. Neuropeptide Y-containing neurons have their cell bodies in the arcuate nuclei and project to the paraventricular nuclei. (c): Ghrelin Ganongs Review of Medical Physiology; 23/e, pg 384 The main site of ghrelin synthesis and secretion is the stomach, but it is also produced in the hypothalamus and has marked growth hormone stimulating activity. In addition, it appears to be involved in the regulation of food intake. Ganongs Review of Medical Physiology; 23/e, pg 448 Ghrelin is secreted primarily by the stomach and appears to play an important role in the central control of food intake. It also stimulates growth hormone secretion by acting directly on the receptors in the pituitary. Oxygen debt in aerobic muscles is characterized by a) Excess of lactate b) Release of oxygen by myoglobin c) Depletion of phosphocreatine d) All of these d) All of these Ganongs Review of Medical Physiology; 23/e, pg 104 Oxygen debt: During exercise, the muscle blood vessels dilate and blood flow is increased so that the available O2 supply is increased. Up to a point, the increase in O2 consumption is proportional to the energy expended, and all the energy needs are met by the aerobic processes. However, when muscular exertion is very great, aerobic resynthesis of energy stores cannot keep pace with their utilization. Under these conditions, phosporylcreatine is still used to resynthesize ATP. In addition, some ATP synthesis is accomplished by using the energy released by the anaerobic breakdown of glucose to lactate. Use of the anaerobic pathway is self-limiting because in spite of rapid diffusion of lactate into the bloodstream, enough accumulates in the muscles to eventually exceed the capacity of the tissue buffers and produce an enzyme-inhibiting decline in pH. However, for short periods, the presence of an anaerobic pathway for glucose breakdown permits muscular exertion of a far greater magnitude than would be possible without it. After a period of exertion is over, extra O2 is consumed to remove the excess lactate, replenish the ATP and phosphorylcreatine stores and replace the small amounts of O2 that were released by myoglobin. The amount of extra O2 consumed is proportional to the extent to which the energy demands during exertion exceeded the capacity for the aerobic synthesis of energy stores, i.e. the extent to which the oxygen debt was incurred. The O2 debt is measured experimentally by determining O2 consumption after exercise, until a constant, basal consumption is reached and subtracting the basal consumption from the total. The amount of this debt may be six times the basal O2 consumption, which indicates that the subject is capable of six times the exertion that would have been possible without it.

Ans: Ref:

BIOCHEMISTRY 37. All of these protect RBCs from free radical damage except a) Superoxide dismutase b) Catalase c) Glutathione d) Glutamine Ans: d) Glutamine Ref: Harpers Illustrated Biochemistry; 27/e, pg 96 98 Option (a): Superoxide dismutase: Ref: Harpers Illustrated Biochemistry, 27/e, pg 97-98

25853434, 42433051, 9873314110, 9953550295, 8447461114

13

DELHI ACADEMY OF MEDICAL SCIENCES PVT. LTD.


MAHARASTRA PAPER - 2011

Superoxide dismutase protects aerobic organisms against oxygen toxicity. Transfer of a single electron to O2 generates the potentially damaging superoxide anion free radical (O2-.), the destructive effects of which are amplified by its giving rise to free radical chain reactions. The ease with which superoxide can be formed from oxygen in tissues and the occurrence of superoxide dismutase, the enzyme responsible for its removal in all aerobic organisms indicate that the potential toxicity of oxygen is due to its conversion to superoxide. In this reaction, superoxide acts as both oxidant and reductant. Thus, superoxide dismutase protects aerobic organisms against the potential deleterious effects of superoxide. Option (b): Catalase: Ref: Harpers Illustrated Biochemistry; 27/e, pg 96. Catalase uses hydrogen peroxide as electron donor and electron acceptor. Catalase is a haemoprotein containing four haeme groups. In addition to possessing peroxidase activity, it is able to use one molecule of H2O2 as a substrate electron donor and another molecule of H2O2 as an oxidant or electron acceptor. Under most conditions in vivo, the peroxidase activity of catalase seems to be favoured. Catalase is found in blood, bone marrow, mucous membrane, kidney and liver. Its function is assumed to be the destruction of hydrogen peroxide formed by the action of oxidases. Peroxisomes are found in many tissues, including liver. They are rich in oxidases and catalases. Thus, the enzymes that produce H2O2 are grouped with the enzyme that destroys it. Option (c): Glutathione Ref: Harpers Illustrated Biochemistry; 27/e, pg 96. In erythrocytes and other tissues, the enzyme glutathione peroxidase, containing selenium as a prosthetic group, catalyses the destruction of H2O2 and lipid hydroperoxides through the conversion of reduced to oxidized glutathione, protecting membrane lipids and haemoglobin against oxidation by peroxides. Glucose-6-phosphate is the initial substrate for all of the following reactions except a) Glycolysis b) HMP shunt c) Glycogenesis d) Neoglucogenesis Ans: d) Neoglucogenesis Option (a): Glycolysis. Ref: Harpers Illustrated Biochemistry; 27/e, pg 151 Glucose enters glycolysis by phosphorylation to glucose-6-phosphate, catalyzed by hexokinase, using ATP as the phosphate donor. Under physiologic conditions, the phophorylation of glucose to glucose-6-phosphate can be regarded as irreversible. Option (b): HMP shunt. Ref: Harpers Illustrated Biochemistry; 27/e, pg 177. The pentose phosphate pathway (hexose monophosphate shunt) is a more complex pathway than glycolysis. Three molecules of glucose-6-phosphate give rise to three molecules of CO2 and three five-carbon sugars. Dehydrogenation of glucose-6-phosphate to 6-phosphogluconate occurs via the formation of 6phosphogluconolactone catalyzed by glucose-6-phosphate dehydrogenase, an NADP-dependent enzyme. Option (c): Glycogenesis. Ref: Harpers Illustrated Biochemistry; 27/e, pg 159. Glycogenesis occurs mainly in muscle and liver. As in glycolysis, glucose is phosphorylated to glucose-6-phosphate, catalyzed by hexokinase in muscle and glucokinase in liver. Glucose-6-phosphate is then isomerized to glucose-1-phosphate by phosphoglucomutase. Option (d): Neoglucogenesis. Ref: Harpers Illustrated Biochemistry; 27/e, pg 167 Gluconeogenesis is the process of converting non-carbohydrate precursors to glucose or glycogen. The major substrates are the glucogenic amino acids, lactate, glycerol and propionate. Liver and kidney are the major gluconeogenic tissues. 39. Phospholipid is the most important lipid in which of the following lipoproteins? a) Chylomicrons b) LDL c) VLDL d) HDL d) HDL Harpers Illustrated Biochemistry; 27/e, pg 218, Table 25.1 38.

Ans: Ref:

Lipoprotei n CM

Source Intestine

Diameter (nm) 90 1000

Density (g/mL) < 0.95

Composition Protein (%) 12

Lipid (%) 98 99

Main lipid components TG

Apolipoprotei ns A-I, A-II, A-IV,

25853434, 42433051, 9873314110, 9953550295, 8447461114

14

DELHI ACADEMY OF MEDICAL SCIENCES PVT. LTD.

MAHARASTRA PAPER - 2011


B-48, C-I, CII, C-III, E B- 48, E B-100, C-I, CII, C-III B-100, E B-100 PL, CH A-I, A-II, A-IV, C-I, C-II, CIII, D, E A-I fatty

CM remnants VLDL IDL LDL HDL HDL-1 HDL-2 HDL-3

CM Liver (Intestine) VLDL VLDL

45 150 30 90 25 35 20 25

< 1.006 0.95 1.006 1.006 1.019 1.019 1.063 20 25 10 20 5 10

68 7 10 11 21 32 33 57 99

92 94 90 93 89 79 68 67 43 1

TG, PL, CH. TG TG, CH CH

Liver, intestine, VLDL, CM

1.019 1.063 1.063 1.125 1.125 1.210 >1.210 >1.281

Pre-HDL <5 Albumin/ free Adipose tissue fatty acids CM Chylomicrons VLDL Very low density lipoproteins IDL Intermediate density lipoproteins HDL High density lipoprotein TG Triacylglycerol PL Phospholipid CH Cholesterol 40.

Free acids

Which of the following lipoproteins is the fastest migrating on electrophoresis and has the lowest triacylglycerol content? a) LDL b) IDL c) VLDL d) HDL Ans: d) HDL Ref: Biochemistry by U. Stayanarayana and U. Chakrapani; 3/e, pg 317 HDL forms the -band moving farthest from the origin towards the anode. For TG content, refer to Q. 39

25853434, 42433051, 9873314110, 9953550295, 8447461114

15

DELHI ACADEMY OF MEDICAL SCIENCES PVT. LTD.

MAHARASTRA PAPER - 2011

Which amino acid found in proteins is also a component of the urea cycle and is a precursor of nitric oxide? a) Arginine b) Histidine c) Glutamine d) Citrulline Ans: a) Arginine Ref: Harpers Illustrated Biochemistry; 27/e, pg 249-250 Steps of the urea cycle: 1. Condensation of CO2, ammonia and ATP to form carbamoyl phosphate is catalyzed by mitochondrial carbamoyl phosphate synthase I 2. Carbamoyl phosphate plus ornithine forms citrulline 3. Citrulline plus aspartate forms arginosuccinate 4. Cleavage of arginosuccinate forms arginine and fumarate 5. Cleavage of arginine releases urea and re-forms ornithine. Ref: Harpers Illustrated Biochemistry; 27/e, pg 580 NO is formed by the action of the enzyme NO synthase, which is cytosolic. The substrate is arginine and the products are citrulline and NO. 42. Alkaptonuria occurs due to deficiency of which of the following enzymes? a) Phenylalanine hydroxylase b) Tyrosine transaminase c) Homogentisic acid oxidase d) Hydroxyphenylpyruvate dehydrogenase c) Homogentisic acid oxidase Harpers Illustrated Biochemistry; 27/e, pg 259 Alkaptonuria was first recognized and described in the 16th century. Characterized in 1859, it provided the basis for Garrods classic ideas concerning heritable metabolic disorders. The defect is a lack of homogentisate oxidase. The urine darkens on exposure to air due to oxidation of excreted homogentisate. Later in the disease, there is arthritis and connective tissue pigmentation (ochronosis) due to oxidation of homogentisate to benzoquinone acetate, which polymerizes and binds to connective tissue. True about HGPRT is all except a) normally found in liver mitochondria b) enzyme of purine salvage pathway c) uses PRPP as a substrate d) deficiency causes Lesch-Nyhan syndrome a) normally found in liver mitochondria Harpers Illustrated Biochemistry; 27/e, pg 308

41.

Ans: Ref: 43.

Ans: Ref:

25853434, 42433051, 9873314110, 9953550295, 8447461114

16

DELHI ACADEMY OF MEDICAL SCIENCES PVT. LTD.

MAHARASTRA PAPER - 2011

44.

Lesch-Nyhan syndrome, an overproduction hyperuricemia, characterized by frequent episodes of uric acid lithiasis and a bizarre syndrome of self-mutilaton, reflects a defect in hypoxanthine-guanine phosphoribosyltransferase (HGPRT), an enzyme of purine salvage. The accompanying rise in intracellular phosphoribosyl pyrophosphate (PRPP) results in purine overproduction. Mutations that delete HGPRT activity include deletions, frameshift mutations, base substitutions and aberrant mRNA splicing.

Proof-reading activity is exhibited by which of the following enzymes involved in DNA replication? a) DNA helicase b) DNA polymerase II c) DNA polymerase III d) Restriction endonuclease Ans: b) DNA polymerase II Ref: Harpers Illustrated Biochemistry; 27/e, 27/e, pg 336. A number of different DNA polymerase molecules engage in DNA replication. These share three important properties: 1. Chain elongation 2. Processivity and 3. Proof-reading. Polymerase II (pol II) is mostly involved in proof reading and DNA repair. E.coli pol Mammalian pol Functions I Gap filling and synthesis of lagging strand II DNA proof-reading and repair DNA repair Mitochondrial DNA synthesis III Processive, leading strand synthesis The mutation that leads to sickle cell anaemia is an example of a) non-sense mutation b) missense mutation c) deletion d) insertion Ans: b) missense mutation Ref: Harpers Illustrated Biochemistry; 27/e, pg 368-370. Single base changes in mRNA molecules may have one of the several effects when translated into protein: 1. There may be no detectable effect because of the degeneracy of the code; such mutations are often referred to as silent mutations. 2. A missense effect will occur when a different amino acid is incorporated at the corresponding site in the protein molecule. The mistaken amino acid, or missense, depending upon its location in the specific protein, might acceptable, partially acceptable or unacceptable to the function of that protein molecule. a) Acceptable missense mutations: An example of an acceptable missense mutation in the structural gene for the chain of Hb could be detected by the presence of an electrophoretically altered Hb in the red cells of an apparently healthy individual. Hb Hikari has been found in at least 2 families of Japanese people. This Hb has asparagines substituted for lysine at the 61 position of chain. The corresponding transversion might be either AAA or AAC. The replacement of the specific lysine with asparagines apparently does not alter the normal function of the chain in these individuals. b) Partially acceptable missense mutations: A partially acceptable missense mutation is best exemplified by HbS, which is found in sickle cell anaemia. Here, glutamic acid, the normal amino acid in position 6 of the chain, has been replaced by valine. The corresponding single nucleotide change within the codon would be GAA or GAG of glutamic acid to GUA or GUG of valine. Clearly, this missense mutation hinders the normal function and results in sickle cell anaemia when the mutant gene is present in the homozygous state. The glutamate-to-valine change may be considered to be partially acceptable because HbS does bind and release oxygen, although abnormally. c) Unacceptable missense mutations: An unacceptable missense mutation in a Hb gene generates a non-functioning Hb molecule. For eg, the HbM mutations generate molecule that allow the Fe2+ of the heme moiety to be oxidized to Fe3+, producing methaemoglobin. Methaemoglobin cannot transport oxygen. 45.

25853434, 42433051, 9873314110, 9953550295, 8447461114

17

DELHI ACADEMY OF MEDICAL SCIENCES PVT. LTD.

MAHARASTRA PAPER - 2011

3. A nonsense codon may appear that would then result in the premature termination of amino acid incorporation into a peptide chain and the production of only a fragment of the intended protein molecule. The probability is high that a prematurely terminated protein molecule or peptide fragment will not function in its assigned role. Insulin normally performs all of the following functions except a) Increased lipogenesis b) Increased amino acid intake into cells c) Activation of lipoprotein lipase d) Activation of enzymes of glycolysis Ans: c) Activation of lipoprotein lipase Ref: Harpers Illustrated Biochemistry; 27/e, pg 219-220. Lipoprotein lipase is located on the walls of the blood capillaries, anchored to the endothelium by negatively charged proteoglycan chains of heparin sulfate. It has been found in heart, adipose tissue, spleen, lung, renal medulla, aorta, diaphragm and lactating mammary gland, although it is not active in adult liver. It is not normally found in blood; however, following injection of heparin, lipoprotein lipase is released from its heparin sulfate binding into the circulation. Both phospholipids and Apo C-II are required as co-factors for lipoprotein lipase activity, while Apo AII and C-III act as inhibitors. Option (a): Increased lipogenesis Ref: Harpers Illustrated Biochemistry; 27/e, pg 201-202. Insulin stimulates lipogenesis 1. Insulin activates acetyl-CoA carboxylase, probably through an activator protein and an insulin-stimulated protein kinase. 2. It increases the transport of glucose into the cell (e.g. in adipose tissue), increasing the availability of both pyruvate for fatty acid synthesis and glycerol-3-phosphate for esterification of the newly formed fatty acids. 3. It converts the inactive form of pyruvate dehydrogenase to the active form in the adipose tissue but not in liver. 4. It inhibits lipolysis in adipose tissue, thereby reduces the concentration of plasma free fatty acids and therefore long chain acyl-CoA, an inhibitor of lipolysis. Option (d): Activation of enzymes of glycolysis Ref: Harpers Illustrated Biochemistry; 27/e, pg 169. The enzymes involved in the utilization of glucose (i.e. those of glycolyis and lipogenesis) become more active when there is a superfluity of glucose, and under these conditions the enzymes of gluconeogenesis have low activity. Insulin, secreted in response to increased blood glucose, enhances the synthesis of the key enzymes in glycolysis. Ref: Ganongs Review of Medical Physiology; 23/e, pg 319 Effects of insulin on various tissues: 1. Adipose tissue: Increased glucose entry Increased fatty acid synthesis Increased glycerol phosphate synthesis Increased triglyceride deposition Activation of lipoprotein lipase Inhibition of hormone-sensitive lipase Increased K+ uptake 2. Muscle: Increased glucose entry Increased glycogen synthesis Increased amino acid uptake Incrreased protein synthesis in ribosomes Decreased protein catabolism Decreased release of gluconeogenic amino acids Increased ketone uptake Increased K+ uptake 3. Liver: Decreased ketogenesis Increased protein synthesis Increased lipid synthesis Decreased glucose output due to decreased gluconeogenesis, increased glycogen synthesis and increased glycolysis 4. General: Increased cell growth 46.

25853434, 42433051, 9873314110, 9953550295, 8447461114

18

DELHI ACADEMY OF MEDICAL SCIENCES PVT. LTD.


47.

MAHARASTRA PAPER - 2011

Ans: Ref: 48.

Xanthuneric acid in urine is excreted in the deficiency of a) Riboflavin b) Biotin c) Pantothenic acid d) Pyridoxine d) Pyridoxine Harpers Illustrated Biochemistry; 27/e, pg 262. Tryptophan is degraded to amphibolic intermediates via the kynurenine-anthranilate pathway. Hydrolytic removal of the formyl group of N-formylkynurenine, catalyzed by kynurenine formylase, produces kynurenine. Since kynureninase requires pyridoxal phosphate, excretion of xanthurenate in response to a tryptophan load is diagnostic of vitamin B6 deficiency. The pyrrole rings in haemoglobin are bound to each other by a) Electrostatic interactions b) Methylene bridges c) Co-ordinate bonds d) Covalent bonds b) Methylene bridges Harpers Illustrated Biochemistry; 27/e, pg 279. Porphyrins are cyclic compounds formed by the linkage of four pyrrole rings through methylene (_HC=) bridges. A characteristic property of the porphyrins is the formation of complexes with metal ions bound to the nitrogen atom of the pyrrole rings. Examples are the iron porphyrins such as heme of haemoglobin and the magnesium-containing porphyrin chlorophyll, the photosynthetic pigment of plants. Which of these enzymes does not play a role in the generation of proton gradient during electron transport? a) NADH oxidase b) Succinate CoQ dehydrogenase c) Cytochrome oxidase d) Cytochrome b-c1 b) Succinate CoQ dehydrogenase Harpers Illustrated Biochemistry; 27/e, pg 103

Ans: Ref: 49.

Ans: Ref:

Note that protons are produced by complexes I, III and IV but not II, i.e. succinate CoQ dehydrogenase. PHARMACOLOGY 50. If the volume of distribution of a drug is < 5L, then the drug is restricted to which of the following body compartments? a) Extracellular space b) Total body water c) Vascular d) Intracellular c) Vascular

Ans:

25853434, 42433051, 9873314110, 9953550295, 8447461114

19

DELHI ACADEMY OF MEDICAL SCIENCES PVT. LTD.

MAHARASTRA PAPER - 2011

Ref: Goodman & Gilmans The Pharmacological Basis of Therapeutics; 12/e, pg 30 & K.D.Tripathis Essentials of Medical Pharamacology; 6/e, pg 18 The volume of distribution (Vd) is defined as the volume that would accommodate all the drug in the body, if the concentration throughout was the same as that in the plasma. It does not necessarily refer to an identifiable physiological volume and hence is known as apparent volume of distribution. Drugs extensively bound to plasma proteins are largely restricted to the vascular compartement. Hence, their Vd is low (equal to intravascular volume; i.e. 0.15 L/kg or 5.5. L) E.g: Diclofenac, Warfarin. 51. Lipid-insoluble drugs do not enter cells. Their Vd approximates ECF volume, i.e. 0.25 L/kg or 12 L. E.g: Streptomycin, Gentamicin. Drugs sequestered in tissues may have Vd much more than total body water or even body mass, i.e. > 5L/kg or > 42 L. E.g: Morphine, Propranolol, Digoxin, Chloroquine. All of the following can cause deficiency of pyridoxine except a) Hydralazine b) Gresiofulvin c) Cycloserine d) Penicillamine b) Gresiofulvin K.D.Tripathis Essentials of Medical Pharamacology; 6/e, pg 876. Drug interactions of pyridoxine Isoniazid reacts with pyridoxal to form a hydrazone, and thus inhibits generation of pyridoxal phosphate. Isoniazid also combines with pyridoxal phosphate to interfere with its co-enzyme function. Due to the formation of hydrazones, the renal excretion of pyriodoxine compounds is increased. Thus, isoniazid therapy produces a pyridoxine deficiency state. Hydralazine, cycloserine and penicillamine also interfere with pyridoxine utilization and action. Oral contraceptives reduce pyridoxal phosphate levels in some women. Pyridoxine, in promoting the formation of dopamine from levodopa from peripheral tissues reduces its availability in the brain, abolishing the therapeutic effect in Parkinsonism, but not when a peripheral decarboxylase inhibitor is combined with it. 4-deoxypyridoxine is a vit. B6 antagonist. Dobutamine acts on which of the following receptors? a) and b) D1 c) D2 d) All of these a) and K.D.Tripathis Essentials of Medical Pharamacology; 6/e, pg 126 Dobutamine: A derivative of dopamine, but not a D1 or D2 receptor agonist. Though it acts on both and adrenergic receptors, the only prominent action of clinically employed doses (2 8 g/kg/min IV infusion) is increased force of cardiac contraction and output, without significant change in heart rate, peripheral resistance and BP. As such, it has been considered to be a relatively selective 1 agonist. It is used as inotropic agent in pump failure accompanying myocardial infarction, cardiac surgery, and for short term management of severe congestive heart failure. It is less arrhythmogenic than adrenaline. Which of these is a specific antagonist of 2 receptors? a) Prazosin b) Phenoxybenzamine c) Phentolamine d) Yohimbine d) Yohimbine K.D.Tripathis Essentials of Medical Pharamacology; 6/e, pg 135. Yohimbine: An alkaloid from West African plant Yohimbehe. It is a relatively selective 2 blocker with short duration of action. It also blocks 5-HT receptors. Heart rate and BP are generally elevated due to increased central sympathetic outflow as well as peripheral NA release. Other CNS effects include excitation, tremor, ADH release (antidiuresis), nausea and vomiting. It may cause congestion of genitals and has been claimed to be an aphrodisiac.

Ans: Ref: 1.

2. 3. 4. 5. 52.

Ans: Ref:

53.

Ans: Ref:

25853434, 42433051, 9873314110, 9953550295, 8447461114

20

DELHI ACADEMY OF MEDICAL SCIENCES PVT. LTD.

MAHARASTRA PAPER - 2011

This effect is only psychological, but can overcome psychogenic impotence in some patients. There are no valid indications for clinical uses of yohimbine. Option (a): Prazosin Ref: K.D.Tripathis Essentials of Medical Pharamacology; 6/e, pg 134 It is first of the highly selective 1 blockers having 1:2 selectivity ratio 1000:1. All subtypes of 1 receptor (1A, 1B, 1D) are blocked equally. Option (b): Phenoxybenzamine Ref: K.D.Tripathis Essentials of Medical Pharamacology; 6/e, pg 133 It cyclizes spontaneously in the body giving rise to a highly reactive ethyleniminium intermediate which reacts with adrenoceptors and other biomolecules by forming strong covalent bonds. The blockade develops gradually (even after IV injection) and lasts for 3 4 days. Option (c): Phentolamine Ref: K.D.Tripathis Essentials of Medical Pharamacology; 6/e, pg 134 This congener of tolazoline is a rapidly acting blocker with a short duration of action (in minutes). It equally blocks 1 and 2 receptors. Which of these drugs is a non-selective -blocker? a) Pindolol b) Bisoprolol c) Esmolol d) Acebutolol Ans: a) Pindolol Ref: K.D.Tripathis Essentials of Medical Pharamacology; 6/e, pg 137 Classification of -blockers First generation (older, non-selective) Propranolol Sotalol Timolol Pindolol Second generation (1 selective) Metoprolol Bisoprolol Atenolol Esmolol Acebutolol Third generation (with additional -blocking and/or vasodilating property) Labetalol Celiprolol Carvedilol Nebivolol 55. Which of these drugs used for glaucoma is free of systemic side-effects/complications and can be used as once-a-day instillation? a) Brimonidine b) Dipivefrine c) Latanoprost d) Dorzolamide c) Latanoprost K.D.Tripathis Essentials of Medical Pharamacology; 6/e, pg 147 Latanoprost: Instilled in the eye, this PGF2 derivative has shown efficacy similar to timolol (IOT reduction by 25 35%) and the effect is well-sustained over long-term. It reduces IOT in normal pressure glaucoma also. Though ocular irritation and pain are frequent, no systemic side effects are reported. Blurring of vision, increased iris pigmentation, thickening and darkening of eyelashes have occurred in some cases. Because of good efficacy, once daily application and absence of systemic complications, PG analogues have become the first choice drugs in developed countries. High cost limits their use in resource poor countries. 54.

Ans: Ref: 56.

The antiplatelet action of aspirin in small doses lasts for how many days? a) 3 days b) 7 days c) 14 days d 21 days Ans: b) 7 days Ref: K.D.Tripathis Essentials of Medical Pharamacology; 6/e, pg 189 Aspirin, even in small doses, irreversibly inhibits TXA2 synthesis by platelets. Thus, it interferes with platelet aggregation and bleeding time is prolonged to nearly twice the normal value. This effect lasts for about a week (turnover time of platelets). Ref: K.D.Tripathis Essentials of Medical Pharamacology; 6/e, pg 609 Aspirin acetylates and inhibits the enzymes COX1 and TX-synthase inactivating them irreversibly.

25853434, 42433051, 9873314110, 9953550295, 8447461114

21

DELHI ACADEMY OF MEDICAL SCIENCES PVT. LTD.

MAHARASTRA PAPER - 2011

57.

Because platelets are exposed to aspirin in the portal circulation before it is deacetylated during first pass in the liver and because platelets cannot synthesize fresh enzyme (they have no nuclei) TXA2 formation is suppressed at very low doses and till fresh platelets are formed. Thus, aspirin induced prolongation of bleeding time lasts for 5 7 days.

Fully humanized antibody that is used in the treatment of rheumatoid arthritis is a) Etanercept b) Anakinra c) Infliximab d) Adalimumab Ans: d) Adalimumab Ref: Katzungs Basic and Clinical Pharmacology; 9/e, pg 592 Adalimumab: Adalimumab is a recombinant human anti-TNF monoclonal antibody. This compound complexes with soluble TNF- and prevents its interaction with p55 and p75 cell surface receptors. This results in down-regulation of macrophage and T-cell function. Option (a): Etanercept Ref: Katzungs Basic and Clinical Pharmacology; 9/e, pg 593 Etanercept is a recombinant fusion protein consisting of two soluble TNF p75 receptor moieties linked to the Fc portion of human IgG1. It binds TNF- molecules and also inhibits lymphotoxin-. Option (c): Infliximab Ref: Katzungs Basic and Clinical Pharmacology; 9/e, pg 592 Infliximab is a chimeric (25% mouse, 75% human) monoclonal antibody that binds with high affinity to soluble and poorly membrane-bound TNF-. Its mechanism of action is probably the same as that of adalimumab. Option (b): Anakinra Ref: K.D.Tripathis Essentials of Medical Pharamacology; 6/e, pg 205. It is a recombinant human IL-1 receptor antagonist. Though clinically less effective than TNF- inhibitors, it has been used in cases who have failed on one or more DMARDs. Dose: 100 mg s.c. daily. 58. Colchicine a) is a analgesic anti-inflammatory drug b) inhibits the synthesis of uric acid c) has uricosuric action d) none of these d) none of these K. D. Tripathis Essentials of Medical Pharmacology. 6/e, pg 206 Colchicine: Colchicine is neither analgesic nor anti-inflammatory, but it especially suppresses gouty inflammation. It does not inhibit the synthesis or promote the excretion of uric acid. Thus, it has no effect on blood uric acid levels. Colchicine does not affect phagocytosis of urate crystals but inhibits release of the glycoprotein and the subsequent events. By binding to fibrillar protein tubulin, it inhibits granulocyte migration into the inflamed joint and thus interrupts the vicious cycle. Other actions of colchicines are: Anti-mitotic causes metaphase arrest by binding to microtubules on mitotic spindle. It was tried for cancer chemotherapy but abandoned due to toxicity. It is used to produce polyploidy in plants. Increases gut motility through neural mechanisms. Which of these inhalational steroids is useful for the management of symptoms of acute bronchial asthma? a) Budesonide b) Beclomethasone c) Ciclesonide d) None of these d) None of these K.D.Tripathis Essentials of Medical Pharamacology; 6/e, pg 225 Inhaled steroids suppress bronchial inflammation, increase peak expiratory flow rate, reduce need for 2 agonist inhalations and prevent episodes of acute asthma. However, they have no role during an acute attack or in status asthmaticus. Peak effect is seen 4 7 days of instituting inhaled steroids and benefit persists for a few weeks after discontinuation.

Ans: Ref: 1. 2. 59.

Ans: Ref:

25853434, 42433051, 9873314110, 9953550295, 8447461114

22

DELHI ACADEMY OF MEDICAL SCIENCES PVT. LTD.

MAHARASTRA PAPER - 2011

Ref: K.D.Tripathis Essentials of Medical Pharamacology; 6/e, pg 227 Management of status asthmaticus/Acute severe asthma: 1. Hydrocortisone hemisuccinate: 100 mg (or equivalent dose of another glucocorticoid) IV stat f/b 100 200 mg 4 8 hrly infusion, may take up to 6 hrs to act. 2. Nebulized salbutamol: (2.5 5 mg) + Ipratropium bromide: (0.5 mg) intermittent inhalations driven by O2 3. High flow humidified oxygen inhalation. 4. Salbutamol/Terbutaline: 0.4 mg IM/sc may be added, since inhaled drug may not reach smaller bronchi due to severe narrowing/plugging. 5. Intubation and mechanical ventilation, if needed. 6. Treat chest infection with intensive antibiotic therapy. 7. Correct dehydration and acidosis with saline + sodium bicarbonate/lactate infusion. Aminophylline 250 500 mg diluted in 20 50 mL glucose (5%) solution injected IV over 20 30 min had been routinely used, but recent evidence shows that it does not afford additional benefit; may even produce more adverse effects; use is restricted to resistant cases. 60. Which of these drugs is a glucagon-like peptide-1 (GLP-1) agonist? a) Glargine b) Pramlintide c) Exenatide d) Sitagliptin c) Exenatide K.D.Tripathis Essentials of Medical Pharamacology; 6/e, pg 273 Exenatide: Exenatide is a synthetic GLP-1 analogue, resistant to DPP-4, but with similar actions, viz. enhancement of post-prandial insulin release, suppression of glucagon release and appetite as well as slowing of gastric emptying. It has been marketed in the U.S. to be used as an additional drug with metformin and/or sulfonyureas in type 2 diabetics who have inadequate response to the oral hypoglycemics. Exenatide is injected s.c. twice daily 1 hour before meals; acts for 6 10 hours. Nausea is an important side-effect.

Ans: Ref:

61.

Peliosis heapatis is associated with all of the following except a) NSAIDs b) Oral contraceptive pills c) Androgens d) Danazol Ans: a) NSAIDs Ref: http://en.wikipedia.org/wiki/Peliosis_hepatis Peliosis Hepatis is an uncommon vascular condition characterised by randomly distributed multiple bloodfilled cavities throughout the liver. The size of the cavities usually ranges between a few millimetres to 3 cm in diameter. It has been increasingly recognised with wide ranging conditions from AIDS to the use of anabolic steroids. It also occasionally affects spleen, lymph nodes, lungs, kidneys, adrenal glands, bone marrow and other parts of gastrointestinal tract Etiology: 1. Infections: HIV Bacillary peliosis (caused by genus Bartonella, bacteria responsible for cat-scratch disease which are identified histologically adjacent to the peliotic lesions Staphylococcus aureus 2. Chronic conditions End stage renal failure Kwashiorkor Tuberculosis and other chronic infections. 3. Malignancy: Monoclonal gammopathies (multiple myeloma and Waldenstrom macroglobulinemia) Hodgkin disease Malignant histiocytosis Seminoma Hepatocellular adenoma and Hepatocarcinoma 4. Renal transplants: It can be found in up to 20% patients, can be related to azathioprine or cyclosporine use and may be associated with increased risk of transplant rejection 5. Drugs and toxins: Corticosteroids Androgens Azathioprine

25853434, 42433051, 9873314110, 9953550295, 8447461114

23

DELHI ACADEMY OF MEDICAL SCIENCES PVT. LTD.

MAHARASTRA PAPER - 2011

Tamoxifen Pathogenesis: The pathogenesis of peliosis hepatis is unknown. There are several hypotheses, such as, 1. sinusoidal epithelial damage 2. increased sinusoidal pressure due to obstruction in blood outflow from the liver, or 3. hepatocellular necrosis. Two morphologic patterns of hepatic peliosis were described by Yanoff and Rawson. 1. Phlebectatic: the blood-filled spaces are lined with endothelium and are associated with aneurysmal dilatation of the central vein. 2. Parenchymal: the spaces have no endothelial lining and they usually are associated with haemorrhagic parenchymal necrosis. Clinical features: The condition is typically asymptomatic and is discovered following evaluation of abnormal liver function tests. However, when severe, it can manifest as jaundice, hepatomegaly, liver failure and haemoperitoneum Treatment: Usually directed towards management of underlying cause. In rare circumstances partial resection of liver or transplant may be required. 62. Cytochrome CYP3A4 are induced by all of the following anti-epileptics except a) Primidone b) Phenytoin c) Phenobarbitone d) Valproate d) Valproate Goodman & Gilmans The Pharmacological Basis of Therapeutics; 12/e, pg 597

Ans: Ref:

Valproate primarily inhibits the metabolism of drugs that are substrates for CYP2C9 including phenytoin and phenobarbitone. Valproate also inhibits UGT and thus inhibits the metabolism of lamotrigine and lorazepam. Option (a): Primidone and Opiton (c): Phenobarbitone. Ref: K. D. Tripathis Essentials of Medical Pharmacology; 6/e, pg 393 Barbiturates induce the metabolism of many drugs and reduce their effectiveness including warfarin, steroids, oral contraceptives, tolbutamide, griseofulvin, chloramphenicol, theophylline. Barbiturates are C/I in acute intermittent porphyria. They exacerbate it by inducing microsomal enzymes (-ALA synthase) and increasing porphyriin synthesis. Option (b): Phenyotin. Ref: K. D. Tripathis Essentials of Medical Pharmacology; 6/e, pg 405 Phenytoin induces microsomal enzymes and increases the degradation of steroids (leading to failure of oral contraceptives), digitoxin, doxycycline, theophylline. 63. Entacapone acts by selective inhibition of a) Dopa decarboxylase b) COMT c) MAO-A d) MAO-B b) COMT K.D.Tripathis Essentials of Medical Pharamacology; 6/e, pg 273 Two selective, potent and reversible COMT inhibitors Tolcapone and Entacapone have been introduced as adjuvants to levodopa/carbidopa for advanced Parkinsons disease (PD). When peripheral decarboxylation of levodopa is blocked by carbidopa or benserazide, it is mainly metabolized by COMT to 3-O-methyldopa. Blockade of this pathway by entacapone/tolcapone prolongs the T of levodopa and allows a larger fraction of administered dose to cross into the brain. Since COMT plays a role in the degradation of dopamine in brain as well, COMT inhibitors could preserve dopamine formed in the striatum and supplement the peripheral effect. However, entacapone acts only in the periphery (probably because of the short duration of action of ~ 2 hrs). For tolcapone also, the central action is less important. Both entacapone and tolcapone enhance and prolong the therapeutic effect of levodopa/carbidopa in advanced and fluctuating PD.

Ans: Ref:

25853434, 42433051, 9873314110, 9953550295, 8447461114

24

DELHI ACADEMY OF MEDICAL SCIENCES PVT. LTD.


MAHARASTRA PAPER - 2011

They may be used to smoothen wearing off, increase on time, decrease off time, improve activities of daily living and allow levodopa dose to be reduced. They are not indicated in early PD cases. Dose: 200 mg with each dose of levodopa/carbidopa. S/e of entacapone: Worsening of levodopa effects such as nausea, vomiting, dyskinesia, postural hypotension, hallucinations, etc. These can be minimized by adjustment of levodopa dose. Diarrhea (in 10-18% of patients) Yellow-orange discolouration of urine Naloxone is a opioid antagonist that acts at the following receptor/s a) (mu) b) (delta) c) (kappa) d) All of these d) All of these K.D.Tripathis Essentials of Medical Pharamacology; 6/e, pg 463 (mu) Agonist (strong) Antagonist (strong) P. agonist, Antagonist (weak) P. agonist (weak) P. agonist Antagonist (strong) Antagonist (strong) Agonist (moderate) Agonist (strong) Agonist (weak) (kappa) Agonist (weak) Agonist (moderate) Agonist (moderate) Agonist (strong) Antagonist (moderate) Antagonist (moderate) Antagonist (strong) Agonist (strong) (delta) Agonist (weak) Antagonist (weak) Antagonist (weak) Agonist (strong) Agonist (strong) Agonist (weak)

64.

Ans: Ref:

Ligand Morphine Nalorphine Pentazocine Butorphanol Buprenorphine Naloxone Naltrexone Met/Leu encephalin -endorphin Dynorphins A, B

65.

Which of the following drugs used in obesity acts as a endogenous cannabinoid receptor (CB1) antagonist? a) Rimonabant b) Orlistat c Sibutramine d) Fenfluramine a) Rimonabant K.D.Tripathis Essentials of Medical Pharamacology; 6/e, pg 115, 131 Rimonabant: A selective cannabinoid-1 receptor antagonist. It blocks hunger-promoting action of cannabis. It has been found in clinical trials to decrease appetitie and help weight reduction by the obese. It has the potential to help in smoking cessation as well as maintain smoking abstinence It has been tried as an anti-smoking and anti-obesity drug. Nausea is a side effect.

Ans: Ref:

Option (b): Orlistat Ref: K.D.Tripathis Essentials of Medical Pharamacology; 6/e, pg 131 An inhibitor of gastric and pancreatic lipase. It interferes with digestion and absorption of dietary triglycerides. Absorption of cholesterol and fat soluble vitamins is also impaired. It has facilitated weight loss in clinical trials. Fluid motions, steatorrhoea, abdominal pain, nausea, flatulence and vitamin deficiency are the side effects.

Option (c): Sibutramine Ref: K.D.Tripathis Essentials of Medical Pharamacology; 6/e, pg 129 A recently introduced anti-obesity drug. It inhibits the re-uptake of both NA and 5-HT but does not have clinically useful antidepressant property. It suppresses appetitie in a manner similar to fenfluramine and appears to stimulate thermogenesis by indirectly activating 3 system in adipose tissue.

25853434, 42433051, 9873314110, 9953550295, 8447461114

25

DELHI ACADEMY OF MEDICAL SCIENCES PVT. LTD.


MAHARASTRA PAPER - 2011

It can cause loss of 3-9 kg weight, but many subjects regain the same when therapy is discontinued. Side effects include dry mouth, constipation, anxiety, insomnia, mood swings, chest pain and a mild increase in BP and HR. A number of serious adverse reaction reports including cardiovascular events and deaths has been received; an ongoing study is assessing its impact on long-term mortality and morbidity.

Option (d): Fenfluramine Ref: K.D.Tripathis Essentials of Medical Pharamacology; 6/e, pg 129 Fenfluramine and dexfenfluramine reduce food-seeking behavior by enhancing serotonergic transmission in the hypothalamus. However, tolerance to the anorectic action develops in 2-3 months. They have tranquilling, rather than stimulant, property and were extensively used by slimming centres. In the late 1990s, echocardiographic abnormalities, valvular defects, pulmonary hypertension and sudden deaths were related to the use of a combined preparation of fenfluramine + phentermine. Similar valvular lesions are known to occur in carcinoid. The US-FDA recommended discontinuation of fenfluramine, dexfenfluramine and their combinations. These are now banned in India and most other countries.

[Note: Drugs that were approved at some point of time to treat obesity: 1. Noradrenergic agents: Phentermine Phenylpropanolamine Diethylpropion Mazindol 2. Serotonergic agents: Fenfluramine Dexfenfluramine 3. Noradrenergic/serotonergic agent: Sibutramine 4. Cannabinoid receptor (CB-1) antagonists: Rimonabant 5. Inhibitor of gastric and pancreatic lipase: Orlistat 6. Sucrose polyesters: Olestra Drugs that are under investigation for treatment of obesity: 1. Leptin analogues 2. Neuropeptide Y antagonists 3. 3 adrenergic agonists 66. Which of these drug/s is/are least likely to increase the levels of bradykinin? a) Pirindopril b) Ramipril c) Irbesartan d) All of these c) Irbesartan K.D.Tripathis Essentials of Medical Pharamacology; 6/e, pg 535 Pharmacologically, AT1 receptor antagonists differ from ACE inhibitors in the following ways: They do not interfere with the degradation of bradykinin and other ACE substrates due to lack of ACE inhibition; no rise in level or potentiation of bradykinin occurs. Consequently, ACE inhibitor related cough is rare. They result in more complete inhibition of AT1 receptor activation, because alternative pathway of angiotensin-II generation and consequent AT1 receptor activation remain intact with ACE inhibitors. They result in indirect AT2 receptor activation. Due to blockade of AT1 receptor mediated feedback inhibition, more angiotensin-II iis produced which acts on AT2 receptors that remain unblocked. ACE inhibitors however result in depression of both AT1 and AT2 activation.

Ans: Ref:

Thus, irbesartan, which is a AT1 receptor blocker, is less likely to increase the levels of bradykinin; as compared to pirindopril and ramipril, which are ACE inhibitors. 67. Coronary-steal phenomenon is caused by

25853434, 42433051, 9873314110, 9953550295, 8447461114

26

DELHI ACADEMY OF MEDICAL SCIENCES PVT. LTD.


a) b) c) d) Ans: Ref: Dipyridamole Diltiazem Nitroglycerine Verapamil

MAHARASTRA PAPER - 2011

a) Dipyridamole K.D.Tripathis Essentials of Medical Pharamacology; 6/e, pg 535 Dipyridamole is a powerful coronary dilator; increases the total coronary flow by preventing uptake and degradation of adenosine which is a local mediator involved in autoregulation of coronary flow in response to ischaemia. It dilates the resistance vessels and abolishes autoregulation, but has no effect on larger conducting coronary vessels. The pharmacological success but therapeutic failure of dipyridamole has been explained on the basis of coronary steal phenomenon. By dilating resistance vessels in non-ischaemic zone as well, it diverts the already reduced blood flow away from the ischaemic zone. Which of the following drugs used to treat peptic ulcer disease can lead to reduced gastrin secretion? a) Omeprazole b) Esmoprazole c) Misoprostol d) All of these c) Misoprostol K.D.Tripathis Essentials of Medical Pharamacology; 6/e, pg 633-634 PGE2 and PGI2 are produced in the gastric mucosa and appear to serve a protective role by inhibiting acid secretion and promoting mucus and HCO3- secretion. In addition, PGs inhibit gastrin production, increase mucosal blood flow and probably have an ill-defined cytoprotective action. However, the most important appears to be their ability to reinforce the mucus layer covering the gastric and duodenal mucosa, which is buffered by HCO3- secreted into this layer by the underlying epithelial cells. Misoprostol (methyl-PGE1 ester) inhibits acid output dose-dependently. The primary use of PG analogues is in the prevention and treatment of NSAID-associated gastrointestinal injury and blood loss. However, PPIs are more effective, more convenient, better tolerated and cheaper. Major problems in the use of misoprostol are diarrhea, abdominal cramps, uterine bleeding, abortion and the need for multiple daily doses. The patient acceptability is poor.

68.

Ans: Ref:

Options (a): Omeprazole and (b): Esmoprazole Ref: K.D.Tripathis Essentials of Medical Pharamacology; 6/e, pg 632 Because of marked and long-lasting acid suppression by PPIs, compensatory hypergastrinemia has been observed. This has been found to induce proliferation of parietal cells and gastric carcinoid tumours in rats, but not in human beings.

[Note: The question read reduced gastrin secretion and not gastric secretion] 69. Erthromycin binds to 50S ribosome and inhibits peptide synthesis by blocking which step? a) Translation b) Transcription c) Translocation d) Initiation c) Translocation K.D.Tripathis Essentials of Medical Pharamacology; 6/e, pg 727 Erythromycin acts by inhibiting bacterial protein synthesis. It combines with 50S ribosome subunits and interferes with translocation. After peptide bond formation between the newly attached amino acid and nascent peptide chain at the acceptor (A) site, the elongated peptide is translocated back to the peptidyl (P) site, making the A site available for next aminoacyl tRNA attachment. This is prevented by erythromycin and the ribosome fails to move along the mRNA to expose the next codon. As an indirect consequence, peptide chain may be prematurely terminated; synthesis of larger proteins is specifically suppressed.

Ans: Ref:

25853434, 42433051, 9873314110, 9953550295, 8447461114

27

DELHI ACADEMY OF MEDICAL SCIENCES PVT. LTD.

MAHARASTRA PAPER - 2011

70.

The anti-tuberculous drug that acts by inhibition of DNA-dependent RNA synthesis is a) Rifampicin b) Ethambutol c) Isoniazid d) Streptomycin a) Rifampicin K.D.Tripathis Essentials of Medical Pharamacology; 6/e, pg 741

Ans: Ref:

Rifampin inhibits DNA-dependent RNA synthesis. Probably, the basis of its selective toxicity is that mammalian RNA polymerase does not avidly bind rifampin, Option (b): Ethambutol Ref: K.D.Tripathis Essentials of Medical Pharamacology; 6/e, pg 742 The mechanism of action of ethambutol is not fully understood, but it has been found to inhibit arabinosyl transferases involved in arabinogalactan synthesis and to interfere with mycolic acid incorporation in mycobacterial cell wall. Option (c): Isoniazid Ref: K.D.Tripathis Essentials of Medical Pharamacology; 6/e, pg 740 The most plausible mechanism of action of INH is inhibition of synthesis of mycolic acids which are unique fatty acid component of mycobacterial cell wall. Option (d): Streptomycin Ref: K.D.Tripathis Essentials of Medical Pharamacology; 6/e, pg 719-720 1. 2. The general pattern of action of aminoglycosides can be described in two main steps: Transport of the aminoglycoside through the bacterial cell wall and cytoplasmic membrane Binding to ribosomes resulting in inhibition of protein synthesis. Once inside the bacterial cell wall, streptomycin binds to 30S ribosomes, but other aminoglycosides bind to additional sites on 50S subunit, as well as to 30S-50S interface. They freeze initiation of protein synthesis, prevent polysome formation and promote their disaggregation to monosomes so that only one ribosome is attached to each strand of mRNA. Binding of aminoglycoside to 30S-50S juncture causes distortion of mRNA codon recognition resulting in misreading of the code; one or more wrong amino acids are entered in the peptide chain and /or peptides of abnormal length are produced. Gancyclovir is more effective than acyclovir against which of the following organisms? a) HSV b) CMV c) EBV d) VZV b) CMV K.D.Tripathis Essentials of Medical Pharamacology; 6/e, pg 769-770 Gancyclovir is an analogue of acyclovir which is active against all herpes viruses including H. simplex, V. zoster, EBV and CMV. It is more active than acyclovir against CMV. The active triphosphate metabolite of gancyclovir attains much higher concentrations inside CMV infected cells. The plasma t of gancyclovir is 2-4 hours, but that of its triphosphate inside CMV-infected cells is > 24 hours. These factors account for its high activity against CMV infections. CMV can develop gancyclovir resistance by mutation. Systemic toxicity of gancyclovir is very high Bone marrow depression Rash Fever Vomiting Neuropsychiatric disturbances

71.

Ans: Ref:

25853434, 42433051, 9873314110, 9953550295, 8447461114

28

DELHI ACADEMY OF MEDICAL SCIENCES PVT. LTD.

MAHARASTRA PAPER - 2011

Hence its use is restricted to severe CMV infections (pneumonia/colitis) in immunocompromised patients (AIDS patients, transplant recipients. IV infusion of 10 mg/kg/d of gancyclovir has prevented blindness in AIDS patients with CMV retinitis. Gancyclovir therapy has been found to lower HBV titre in chronic hepatitis B. 72. Which antiretroviral drug is known to be associated with nephrolithiasis? a) Saquinavir b) Ritonavir c) Nelfinavir d) Indinavir d) Indinavir K.D.Tripathis Essentials of Medical Pharamacology; 6/e, pg 773 The most prominent adverse effects associated with protease inhibitors are Gastrointestinal intolerance Asthenia Headache Dizziness Limb and facial tingling Numbness Rashes Lipodystrophy (abdominal obesity, buffalo hump with wasting of limbs and face) and Dyslipidemia (raised TG and cholesterol)

Ans: Ref:

Diabetes may be exacerbated Indinavir crystallizes in urine and increases the risk of urinary calculi. Excess fluids must be consumed with indinavir to avoid nephrolithiasis. 73. A pregnant lady in the 3rd trimester is visiting a chloroquine-resistant malaria endemic area. Which drug can be used as prophylaxis for her? a) Mefloquine b) Doxycycline c) Atovaquone/Proguanil d) Primaquine a) Mefloquine K.D.Tripathis Essentials of Medical Pharamacology; 6/e, pg 782-783 Suppressive prophylaxis of malaria: Chloroquine: In travelers, start one week before and continue till one month after return from endemic area. The last dose should be over 3 days with primaquine. It should not be given for > 3 years for fear of cumulative toxicity. Proguanil daily with chloroquine weekly affords substantial protection against moderately chloroquineresistant P. falciparum, but less than that afforded by mefloquine. This has been successfully used in Africa. In India, NVBDCP recommends it for visitors to areas with chloroquine resistance. Mefloquine weekly till 4 weeks after return from endemic area has been used for areas where chloroquine-resistant P. falciparum is prevalent. In India, use of mefloquine for prophylaxis is not allowed among residents, but may be used by travelers. Doxycycline starting daily before travel and taken till 4 weeks afer return from endemic area for chloroquineresistant P. falciparum is an alternative regimen for individuals unable to take mefloquine. It is contraindicated in pregnant women and children < 8 years. Goodman & Gilmans The Pharmacological Basis of Therapeutics; 12/e, pg 1387-1388 Atovaquone/Proguanil is not recommended for prophylaxis of Children < 5 kg. Pregnant women Women breast-feeding infants weighing < 5 kg. Doxycycline is contraindicated in Children < 8 years of age. Pregnant women. Primaquine is contraindicated in Persons with G6PD deficiency.

Ans: Ref: 1. 2. 3. 4.

Ref:

25853434, 42433051, 9873314110, 9953550295, 8447461114

29

DELHI ACADEMY OF MEDICAL SCIENCES PVT. LTD.


MAHARASTRA PAPER - 2011

Pregnancy Lactation unless the infant being breastfed has documented normal G6PD levels.

Thus, options (b), (c) and (d) are all C/I in pregnancy. Hence, the best answer of choice amongst the given options is option (a): mefloquine. Mefloquine is C/I in Persons allergic to mefloquine or related compounds (quinine, quinidine, etc.) Perosns with active depression, a recent history of depression, generalized anxiety disorder, psychosis, schizophrenia, other major psychiatric disorders and seizures.

PATHOLOGY 74. Which of the following genes is responsible for inhibiting apoptosis? a) p53 b) N-myc c) ras d) Bcl-2 d) Bcl-2 Robbins & Cotrans Pathologic Basis of Disease; 8/e, pg 28-29 The intrinsic (mitochondrial) pathway of apoptosis is the result of increased mitochondrial permeability and release of pro-apoptotic molecules into the cytoplasm, without a role of death receptors. Growth factors and other survival signals stimulate the production of anti-apoptotic members of the Bcl-2 family of proteins. This family is named after Bcl-2, which was identified as a oncogene in a B cell lymphoma and is homologous to the C. elegans protein, Ced9. There are more than 20 proteins in this family, all of which function to regulate apoptosis; the two main anti-apoptotic ones are Bcl-2 and Bcl-x. These anti-apoptotic proteins normally reside in mitochondrial membranes and the cytoplasm. When cells are deprived of survival signals or subjected to stress, Bcl-2 and/or Bcl-x are lost from the mitochondrial membrane and are replaced by the pro-apoptotic members of the family, such as Bak, Bax and Bim. When Bcl-2/Bcl-x levels decrease, the permeability of the mitochondrial membrane increases, and several proteins that can activate the caspase cascade leak out. One of these proteins is cytochrome c, well known for its role in mitochondrial respiration. In the cytosol, cytochrome c binds to a protein called Apaf-1 (apoptosis-activating factor-1, homologous to Ced-4 in C. elegans), and the complex activates caspase-9. (Bcl-2 and Bcl-x may also directly inhibit Apaf-1 activation, and their loss from cells may permit activation of Apaf-1). Other mitochondrial proteins, such as apoptosis inducing factor (AIF), enter the cytoplasm, where they bind to and neutralize various inhibitors of apoptosis, whose normal function is to block caspase activation. The net result is the initiation of a caspase cascade. Thus, the essence of the intrinsic pathway is a balance between pro-apoptotic and protective molecules that regulate mitochondrial permeability and the release of death inducers that are normally sequestered within the mitochondria.

Ans: Ref:

Option a: p53. Ref: Robbins & Cotrans Pathologic Basis of Disease; 7/e, pg 302 The major functional activities of the p53 protein are cell cycle arrest and initiation of apoptosis in response to DNA damage. p53 is called in to apply emergency brakes when DNA is damaged by irradiation, UV light, or mutagenic chemicals, and also in response to changes in cellular redox potential, hypoxia, senescence, and other stress conditions that may not directly damage DNA. Option (b): N-myc. Ref: Robbins & Cotrans Pathologic Basis of Disease; 8/e, pg pg 284; 7/e, pg 298 The MYC protein is rapidly translocated to the nucleus, sometimes as a dimer with another protein, called MAX. This heterodimer binds to DNA sequences in target genes and is a potent transcriptional activator. Some of its target genes, such as ornithine decarboxylase and cyclin D2, are known to be associated with cell proliferation. While on one hand, MYC activation is linked to proliferation, on the other hand, cells in culture undergo apoptosis if MYC activation occurs in the absence of survival signals (growth factors). The MYC proto-oncogene contains several sequences that encode the growth promoting and apoptotic activities, but it is not clear whether MYC-induced apoptosis occurs in vivo.

25853434, 42433051, 9873314110, 9953550295, 8447461114

30

DELHI ACADEMY OF MEDICAL SCIENCES PVT. LTD.

MAHARASTRA PAPER - 2011

Option (c): ras Ref: Robbins & Cotrans Pathologic Basis of Disease; 7/e, pg pg 296 Several studies indicate that RAS plays an important role in mitogenesis induced by growth factors. Recent studies have revealed that, in addition to its role in transducing growth factor signals, RAS is also involved in regulation of cell cycle. The passage of cells from G1 to S phase is modulated by cyclins and CDKs. RAS proteins can indirectly regulate the levels of cyclins by activating the MAP-kinase pathway and the AP-1 transcription factor. Which of these co-receptors helps in the fusion of HIV to the cell membranes of macrophages? a) CD4 b) CXCR4 c) CCR5 d) CD8 c) CCR5 Robbins & Cotrans Pathologic Basis of Disease; 7/e, pg 249 The discovery of co-receptors for HIV infection has also solved previously puzzling observations of HIV tropism. It has been known for some time that HIV strains can be classified into two groups on the basis of their ability to infect macrophages and established CD4+ T cell lines. Macrophage-tropic (M-tropic) strains, their name notwithstanding, can infect both monocytes/macrophages and freshly isolated peripheral blood T-cells, but not in vitro propagated T-cell lines. By contrast, the T-cell line tropic strains can infect only T-cells, both freshly isolated and maintained in culture. This sensitivity is based on co-receptor usage: M-tropic strains use CCR5, whereas T-tropic strains bind to CXCR4. Because CCR5 is expressed on both monocytes and T-cells, they are susceptible to infection by Mtropic strains; CXCR4 is expressed on T-cells but not on monocytes/macrophages, and hence T-cells but not macrophages can be infected with T-tropic strains. Primary (freshly isolated) T-cells express both CCR5 and CXCR4 and hence can be infected by either of the two viral types. The role of P-selectin in inflammation is a) Rolling b) Transmigration c) Lymphocyte homing d) Adhesion a) Rolling Robbins & Cotrans Pathologic Basis of Disease; 8/e, pg 49 Leukocyte molecule Sialyl-Lewis-X-modified proteins Sialyl-Lewis-X-modified proteins L-selectin CD11/CD18 (2) integrins (LFA-1, Mac-1) VLA-4 (1 integrin) Major role Rolling (neutrophils, monocytes, T lymphocytes) Rolling and adhesion (neutrophils, monocytes, T lymphocytes) Rolling (neutrophils, monocytes) Adhesion, arrest, transmigration (neutrophils, monocytes, lymphocytes) Adhesion (eosinophils, monocytes, lymphocytes)

75.

Ans: Ref:

76.

Ans: Ref:

Endothelial molecule P-selectin E-selectin GlyCam-1, CD34 ICAM-1 (Ig family)

VCAM-1 (Ig family)

L-selectin is expressed weakly on neutrophils. It is involved in the binding of circulating T-lymphocytes to the high endothelial venules in lymph nodes and mucosal lymphoid tissues, and subsequent homing of lymphocytes to these tissues. Which of these interleukins is not associated with stimulating the proliferation and differentiation of lymphocytes? a) IL-1

77.

25853434, 42433051, 9873314110, 9953550295, 8447461114

31

DELHI ACADEMY OF MEDICAL SCIENCES PVT. LTD.


b) c) d) Ans: Ref: 1. 2. 3. 4. 5. 78. IL-2 IL-4 IL-12

MAHARASTRA PAPER - 2011

a) IL-1 Robbins & Cotrans Pathologic Basis of Disease; 7/e, pg 202 Classification of cytokines according to functions Cytokines that mediate innate (natural) immunity IL-1 TNF- Type-1 IFN IL-6 IL-12 IFN-gamma Cytokines that regulate lymphocyte growth, activation and differentiation IL-2 IL-4 IL-12 IL-15 TGF- Cytokines that activate inflammatory cells IFN-gamma IL-5 TNF- (Lymphotoxin) Cytokines that affct leukocyte movement C-C chemokines C-X-C chemokines Cytokines that stimulate haematopoeisis Colony stimulating factors All of the following molecules are acute-phase reactants except a) C-reactive protein b) Tissue factor c) Fibrinogen d) Serum-amyloid associated protein b) Tissue factor Robbins & Cotrans Pathologic Basis of Disease; 8/e, pg 74 Acute phase proteins are plasma proteins, mostly synthesized in the liver, whose plasma concentrations may increase several hundred-fold as a part of the response to inflammatory stimuli. Three of the best known of these proteins are C-reactive protein (CRP), fibrinogen and serum amyloidassociated protein (SAA) Synthesis of these molecules by hepatocytes is up-regulated by cytokines, especially IL-6 (for CRP and fibrinogen) and IL-1 or TNF (for SAA) All of the following are true regarding Chediak-Higashi syndrome except a) Neutrophilia b) Defective granulation c) Decreased microbial killing d) Large granules in neutrophils a) Neutrophilia Robbins & Cotrans Pathologic Basis of Disease; 8/e, pg 55

Ans: Ref:

79.

Ans: Ref:

Chediak-Higashi syndrome is an autosomal recessive condition characterized by defective fusion of phagosomes and lysosomes in phagocytes. The main leukocyte abnormalities in Chediak-Higashi syndrome are: 1. Neutropenia (decreased number of neutrophils) 2. Defective degranulation and 3. Delayed microbial killing. Leukocytes contain giant granules, which can be readily seen in peripheral blood smears and are thought to result from an aberrant phagolysosome fusion.

25853434, 42433051, 9873314110, 9953550295, 8447461114

32

DELHI ACADEMY OF MEDICAL SCIENCES PVT. LTD.

MAHARASTRA PAPER - 2011

80.

Heparin a) b) c) d)

present in the plasma is secreted by which of the following cells? Macrophages Basophils Eosinophils Mast cells

Ans: Ref:

d) Mast cells Robbins & Cotrans Pathologic Basis of Disease; 8/e, pg 199

Mediators released on mast cell degranulation Preformed (Primary) mediators: 1. Vasoactive amines: The most important mast cell-derived amine is histamine. Histamine causes intense smooth muscle contraction, increased vascular permeability and increased mucus secretion by nasal, bronchial and gastric glands. 2. Enzymes: These are contained in the granule matrix and include neutral proteases (chymase, tryptase) and several acid hydrolases. The enzymes cause tissue damage and lead to the generation of kinins and activated components of complement (e.g: C3a) by acting on their precursor proteins. 3. Proteoglycans: These include heparin, a well-known anticoagulant, and chondroitin sulfate. The proteoglycans serve to package and store the amines in the granules. Lipid (Secondary) mediators: 1. Leukotrienes: LTC4 and LTD4 are the most potent spasmogenic agents known. They are thousand times more active than histamine in increasing vascular permeability and causing bronchial smooth muscle contraction. LTB4 is highly chemotactic for neutrophils, eosionophils and monocytes. 2. Prostaglandin D2: This is the most abundant mediator produced in mast cells by the COX pathway. It causes intense bronchospasm as well as increased mucus secretion. 3. Platelet activating factor (PAF): PAF is produced by some mast cell populations. It causes platelet aggregation, release of histamine, bronchospasm, increased vascular permeability and vasodilation. In addition, it is highly chemotactic for neutrophils and eosinophils; and at high concentrations, activates the imflammatory cells causing them to degranulate. Although the production of PAF is also triggered by the activation of phospholipase A2, it is not a product of arachidonic acid metabolism. 81. Which of these is/are factor/s developed for angiogenesis in the heart? a) VEGF (Vascular Endothelial Growth Factor) b) FGF (Fibroblast Growth Factor) c) HIF-1 (Hypoxia Inducible Factor-1) d) All of these d) All of these Braunwald's Heart Disease: A Textbook of Cardiovascular Medicine; 8/e

Ans: Ref:

Experiments for novel therapeutic options for myocardial ischaemia Gene transfer is a useful approach to introduce nucleic acids to somatic cells of an animal to define gene function, dissect disease pathophysiology, or achieve a therapeutic effect. Over the past decade, gene transfer has been employed in many animal models of cardiovascular disease. Two splice variants of vascular endothelial growth factor (VEGF-A), VEGF165 and VEGF121, have angiogenic properties in animal models and have been evaluated in clinical trials. Placental growth factor (PlGF) also has angiogenic activity in animals. PlGF binds to VEGFR-1 and contributes to angiogenesis under pathological conditions. Interestingly, PlGF also promotes angiogenesis and arteriogenesis through mobilization of hematopoietic stem cells and endothelial progenitor cells from the bone marrow. The clinical relevance of VEGF, PlGF, and endothelial progenitor cells in the clinical treatment of ischemia remains undetermined. Other growth factors have been evaluated for their angiogenic properties in gene transfer models. Hypoxia-inducible transcription factor-1-alpha activates several angiogenic growth factors including VEGF-A, VEGFR-2, insulin-like growth factor-2, and erythropoietin. Adenoviral-mediated VEGF and fibroblast growth factor delivery and vascular growth has been demonstrated in the myocardium and skeletal muscle by the proliferation and enlargement of capillaries, although cessation of therapy (or extinction of the transgene) leads to regression of most of the vessels. Expression of VEGF for more than 4 weeks leads to sufficient vascular remodeling that new vessel growth persists for several months after VEGF treatment is withdrawn. To summarize, factors involved in angiogenesis in the heart VEGF VEGFR-2 PlGF FGF

25853434, 42433051, 9873314110, 9953550295, 8447461114

33

DELHI ACADEMY OF MEDICAL SCIENCES PVT. LTD.


82. IGF-2 Erythropoietin HIF-1 Autosomal dominant mode of inheritance is seen in a) Wilson disease b) Multiple endocrine neoplasia Type 2 c) Haemochromatosis d) All of these b) Multiple endocrine neoplasia Type 2 Robbins & Cotrans Pathologic Basis of Disease; 8/e, pg 274

MAHARASTRA PAPER - 2011

Ans: Ref: 1. 2. 3. 4. 5.

Autosomal dominant inherited cancer syndromes: Retinoblastoma mutation in Rb gene. They also have a greatly increased risk of developing a second cancer, particularly osteosarcoma. Familial adenomatous polyposis coli mutation in the APC tumour suppressor gene. Li-Fraumeni syndrome mutations of the p53 gene. Multiple endocrine neoplasia (MEN-1 and MEN-2) mutations in gene that encode the menin transcription factor and RET tyrosine kinase, respectively. Hereditary non-polyposis colon cancer inactivation of a DNA mismatch repair gene.

Option (a): Wilsons disease Ref: Robbins & Cotrans Pathologic Basis of Disease; 8/e, pg 863 Wilsons disease is an autosomal recessive disorder caused by mutation in the ATP7B gene, resulting in impaired copper excretion into the bule and a failure to incorporate copper into the ceruloplasmin. Option (c): Hemochromatosis. Ref: Robbins & Cotrans Pathologic Basis of Disease; 8/e, pg 861 Hemochromatosis was first described by von Recklinghausen in 1889. Primary or hereditary hemochromatosis is a homozygous recessive inherited disorder, that is caused by excessive iron absorption. 83. The pattern of inheritance seen in cases of ABO blood groups and HLA antigens is a) Pseudo-dominance b) Dominance c) Co-dominance d) c) Co-dominance Robbins & Cotrans Pathologic Basis of Disease; 8/e, pg 140

Ans: Ref:

Although gene expression and mendelian traits are usually described as dominant or recessive, in some cases, both the alleles of a gene pair contribute to the phenotype a condition called as co-dominance. Histocompatibility and blood group antigens are good examples of co-dominant inheritance. 84. Edwards syndrome is trisomy of which chromosome? a) 13 b) 15 c) 18 d) 21 c) 18 Robbins & Cotrans Pathologic Basis of Disease; 8/e, pg 162-163 Trisomy 18: Edwards syndrome. Clinical features: Prominent occiput Mental retardation Low set ears Micrognathia Short neck Overlapping fingers

Ans: Ref:

25853434, 42433051, 9873314110, 9953550295, 8447461114

34

DELHI ACADEMY OF MEDICAL SCIENCES PVT. LTD.


Congenital heart defects Renal malformations Limited hip abduction Rocker-bottom feet

MAHARASTRA PAPER - 2011

Option (a): 13 Ref: Robbins & Cotrans Pathologic Basis of Disease; 8/e, pg 162-163 Trisomy 13: Pataus syndrome. Clinical manifestations: Microcephaly Mental retardation Microphthalmia Cleft lip and palate Polydactyly Congenital cardiac defects Umbilical hernia Renal defects Rocker-bottom feet.

Option (d): 21 Ref: Robbins & Cotrans Pathologic Basis of Disease; 8/e, pg 162-163 Trisomy 21: Downs syndrome. Clinical manifestations: Mental retardation Epicanthic folds Flat facial profile Anti-Mongoloid slant Abundant neck skin Simian crease Hypothyroidism Congenital heart defects Intestinal stenosis Umbilical hernia Hypotonia Saddle gap gap between the first and second toe Increased predisposition to juvenile myeloid leukaemia and Alzheimers disease. An ectopic rest of normal cell tissue is called a) Choristoma b) Hamartoma c) Angioma d) Carcinoma a) Choristoma Robbins & Cotrans Pathologic Basis of Disease; 8/e, pg 262 Choristoma is a congenital anomaly better described as a heterotopic rest of cells. For example, a small nodule of well developed and normally organized pancreatic substance may be found in the submucosa of the stomach, duodenum or small intestine. This heterotopic rest may be replete with islets of Langerhans and exocrine glands.

85.

Ans: Ref:

Option (b): Hamartoma Ref: Robbins & Cotrans Pathologic Basis of Disease; 8/e, pg 262 86. Hamartomas present as disorganized but benign-appearing masses composed of cells indigenous to the particular site. They were once thought to be a developmental malformation, unworthy of the oma designation. However, many hamartomas, including pulmonary chondroid hamartomas, have clonal, recurrent translocations, involving genes encoding certain chromatin proteins. Thus, through molecular biology, they have finally earned their oma designation. AFP elevation is seen in which of the following testicular tumours? a) Yolk sac tumour

25853434, 42433051, 9873314110, 9953550295, 8447461114

35

DELHI ACADEMY OF MEDICAL SCIENCES PVT. LTD.


b) c) d) Ans: Ref: Seminoma Choriocarcinoma Spermatocytic seminoma

MAHARASTRA PAPER - 2011

a) Yolk sac tumour Robbins & Cotrans Pathologic Basis of Disease; 8/e, pg 989-990 Yolk sac tumour: Also known as endodermal sinus tumour, it is the most common testicular tumour in infancy and children up to 3 years of age. Present within and outside the cytoplasm are eosinophilic, hyaline-like globules in which -fetoprotein (AFP) and 1-antitrypsin can be demonstrated by immunocytochemical staining. The presence of AFP in the tumour cells is highly characteristic and it underscores their differentiation into yolk sac cells.

Option (c): Choriocarcinoma. Ref: Robbins & Cotrans Pathologic Basis of Disease; 8/e, pg 990 87. The syncytiotrophoblastic cells (from which the tumour arises) are large and have many irregular or lobular hyperchromatic nuclei and an abundant eosinophilc vacuolated cytoplasm. hCG can be readily demonstrated in the cytoplasm. The most widely used tumour marker for pancreatic cancer is a) CA 125 b) CA 19-9 c) CEA d) None of these b) CA 19-9 Sabistons Textbook of Surgery; 18/e, pg Ref: 88. The two most widely used pancreatic cancer serum markers are the CEA and the Lewis blood group carbohydrate antigen CA 19-9. Both are frequently elevated in patients with advanced disease, but unfortunately, the circulating levels of these tumor markers are often normal in patients with early, potentially curable, tumors. Thus, using these tumor markers to screen patients with vague symptoms or those in high-risk groups has not been shown to be useful in detecting early disease. With a cutoff value of 37 U/mL, CA 19-9 has been reported to have a sensitivity of 86% and a specificity of 87%. CA 19-9 can also be elevated in patients with cholangitis and jaundice not caused by pancreatic cancer. Extremely high levels of either CA 19-9 or CEA usually indicate unresectable or metastatic disease. Schwartz Principles of Surgery; 8/e, CA19-9 is a mucin-associated carbohydrate antigen that can be detected in the serum of patients with pancreatic cancer. Serum levels are elevated in about 75% of patients with pancreatic cancer; however, CA19-9 is also elevated in about 10% of patients with benign diseases of the pancreas, liver, and bile ducts. CA19-9 is thus neither sufficiently sensitive nor specific to allow an earlier diagnosis of pancreatic cancer. In which of the following thyroid cancers is deposition of amyloid due to altered calcitonin seen? a) Follicular b) Papillary c) Medullary d) A variant of papillary c) Medullary Robbins & Cotrans Pathologic Basis of Disease; 8/e, pg 1125 Microscopically, medullary carcinomas of thyroid are composed of polygonal to spindle-shaped cells, which may form nests, trabeculae and even follicles. Small, more anaplastic cells are present in some tumours and may be the predominant cell type. Acellular amyloid deposits, derived from altered calcitonin polypeptides, are present in the adjacent stroma in many cases. Calcitonin is readily demonstrable within the cytoplasm of the tumour cells as well as in the stromal amyloid by immunohistochemical methods.

Ans: Ref:

Ans: Ref:

25853434, 42433051, 9873314110, 9953550295, 8447461114

36

DELHI ACADEMY OF MEDICAL SCIENCES PVT. LTD.

MAHARASTRA PAPER - 2011

As with all neuroendocrine tumours, electron microscopy reveals variable numbers of membrane-bound electron-dense granules within the cytoplasm of the neoplastic cells. Libman-Sacks endocarditis is seen in a) Rheumatoid endocarditis b) SLE c) Infective endocarditis d) None of these b) SLE Robbins & Cotrans Pathologic Basis of Disease; 8/e, pg 569 Endocarditis of Systemic Lupus Erythematosus (Libman-Sacks endocarditis): Mitral and tricuspid valculitis with small, sterile vegetations, called Libman-Sacks endocarditis, is occasionally encountered in SLE. The lesions are small (1-4 mm in diameter), single or multiple, sterile, pink vegetations that often have a warty/verrucous appearance. They may be located on the undersurface of the atrioventricular valves, on the valvular endocardium, on the chords or on the mural endocardium of the atria or the ventricles. Histologically, the vegetations consist of a finely granular, fibrinous eosinophilic material that may contain haematoxylin bodies, homogenous remnants of nuclei damaged by anti-nuclear antigen bodies. An intense valvulitis may be present, characterized by fibrinoid necrosis of the valve substance that is often contiguous with the vegetations. Active leaflet vegetations can be difficult to distinguish from those of infective endocarditis; fibrosis and serious deformities can result that resemble chronic rheumatoid heart disease and require surgery. Schaumann bodies are a characteristic feature of a) Tuberculosis b) Aspergillosis c) Sarcoidosis d) Bronchogenic carcinoma c) Sarcoidosis Robbins & Cotrans Pathologic Basis of Disease; 8/e, pg 702 Histology of Sarcoidosis: All involved tissues show the classic well-formed non-caseating granulomas, each composed of an aggregate of tightly clustered epithelioid cells, often with Langhans or foreign-body type giant cells. Central necrosis is unusual. With chronicity, the granulomas may become enclosed within fibrous rims or may eventually be replaced by hyaline fibrous scars. Laminated concretions composed of calcium and proteins known as Schaumann bodies and stellate inclusions known as asteroid bodies enclosed within giant cells are found in approximately 60% of the granulomas. Though characteristic, these microscopic features are not pathognomonic of sarcoidosis, because asteroid and Schaumann bodies may be encountered in other granulomatous diseases (e.g. tuberculosis). The type of emphysema that is most commonly associated with -1 antitrypsin deficiency is a) Irregular b) Centriacinar c) Panacinar d) Mixed c) Panacinar Robbins & Cotrans Pathologic Basis of Disease; 8/e, pg 684 Types of emphysema: Emphysema is classified according to its anatomic distribution within the lobule.

89.

Ans: Ref:

90.

Ans: Ref:

91.

Ans: Ref:

1.

Centriacinar (centrilobular) emphysema: In this type of emphysema, the central or the proximal parts of the acini, formed by respiratory bronchioles, are affected, whereas distal alveoli are spared. Thus, both emphysematous and normal air spaces exist within the same acinus and lobule. The lesions are more common and usually more severe in the upper lobes, particularly in the apical segments.

25853434, 42433051, 9873314110, 9953550295, 8447461114

37

DELHI ACADEMY OF MEDICAL SCIENCES PVT. LTD.


2. 3. 4. 92.

MAHARASTRA PAPER - 2011

The walls of the emphysematous spaces often contain large amounts of black pigment. Inflammation around bronchi and bronchioles is common. In severe centriacinar emphysema, the distal acinus also may be involved, and differentiation from panacinar emphysema becomes difficult. Centricacinar emphysema occurs predominantly in heavy smokers, often in association with chronic bronchitis. Panacinar emphysema: In this type, the acini are uniformly enlarged from the level of the respiratory bronchiole to the terminal blind alveoli. The prefix pan refers to the entire acinus, but not to the entire lung. In contrast to centriacinar emphysema, panacinar emphysema tends to occur more commonly in the lower zones and in the anterior margins of the lung, and it is usually most severe at the bases. This type of emphysema is associated with 1-antitrypsin deficiency. Distal acinar (Paraseptal) emphysema: In this type, the proximal portion of the acinus is normal, and the distal part is predominantly involved. The emphysema is more striking adjacent to the pleura, along the lobular connective tissue septa, and at the margins of the lobules. It occurs adjacent to the areas of fibrosis, scarring or atelectasis; and is usually more severe in the upper half of the lungs. The characteristic findings are of multiple, continuous, enlarged air spaces from less than 0.5 to more than 2 cm in diameter, sometimes forming cyst-like structures. This type of emphysema probably underlies many of the cases of spontaneous pneumothorax in young adults. Irregular Emphysema: Irregular emphysema, so named because the acinus is irregularly involved, is almost invariably associated with scarring. Thus, it may be the most common form of emphysema, as careful search of most lungs at autopsy shows one or more scars from a healed inflammatory process. In most cases, these foci of irregular emphysema are aymptomatic and clinically insignificant. Mallory a) b) c) d) hyaline is seen in which of the following conditions? Hepatitis B Hepatitis C Alcoholic cirrhosis None of these

Ans: Ref:

c) Alcoholic cirrhosis Robbins & Cotrans Pathologic Basis of Disease; 8/e, pg 858 Mallory bodies: Scattered hepatocytes accumulate tangled skeins of cytokeratin intermediate filaments such as cytokeratin 8 and 18; in complex with other proteins such as ubiquitin. These are called as Mallory bodies Mallory bodies are visible as eosinophilic cytoplasmic clumps in hepatocytes. These inclusions are a characteristic but not specific feature of alcoholic liver disease; since they are present in o Alcoholic liver disease o Non-alcoholic fatty liver disease (NAFLD) or Non-alcoholic steatohepatitis (NASH) o Primary biliary cirrhosis o Wilsons disease o Chronic cholestatic syndromes and o Hepatocellular tumours

Note: Mallory bodies are rarely evident at the stage of alcoholic cirrhosis; they are present mainly in the early stages of alcoholic liver disease. 93. The single most factor that determines the progression of liver disease to cirrhosis is a) Biochemical features b) Etiology c) Bridging necrosis d) Mallory-hyaline bodies b) Etiology Robbins & Cotrans Pathologic Basis of Disease; 7/e, pg 898

Ans: Ref:

25853434, 42433051, 9873314110, 9953550295, 8447461114

38

DELHI ACADEMY OF MEDICAL SCIENCES PVT. LTD.

MAHARASTRA PAPER - 2011

Causes of chronic hepatitis: Hepatitis viruses (HBV, HCV, HBV + HDV) Chronic alcoholism Wilsons disease -1 antitrypsin deficiency Drugs Isoniazid, -methyldopa, methotrexate) and Autoimmunity

In all instances of chronic hepatitis, etiology is the single most indicator of likelihood to progress to cirrhosis 94. The lesions found in idiopathic nephrotic syndrome include all except a) Membranous b) Minimal change c) Membranoproliferative d) Mesangioproliferative d) Mesangioproliferative Robbins & Cotrans Pathologic Basis of Disease; 8/e, pg 923

Ans: Ref:

Causes of nephrotic syndrome Primary glomerular diseases Membranous glomerulopathy (option a) Minimal change disease (option b) Focal segmental glomerulosclerosis Membranoproliferative glomerulonephritis (option c) Other proliferative glomerulonephritides: Focal Pure mesangial (option d) IgA nephropathy Systemic diseases Diabetes mellitus Amyloidosis Systemic lupus erythematosus Drugs NSAIDs Penicillamine Street heroin Infections Malaria Syphilis Hepatitis B Hepatitis C HIV Malignant diseases Carcinomas Lymphomas Miscellaneous Bee-sting allergy Hereditary nephritis [Note: Mesangioproliferative glomerulonephritis is a less common cause of primary/idiopathic nephrotic syndrome] 95. Mesangial deposits of IgA on renal biopsy is a feature of a) Wegeners granulomatosis b) Henoch-Schonlein purpura c) Churg-Strauss syndrome d) All of these b) Henoch-Schonlein purpura Robbins & Cotrans Pathologic Basis of Disease; 8/e, pg 934 On histologic examination, the renal lesions of Henoch-Schonlein purpura vary from mild focal mesangial proliferation to diffuse mesangial proliferation and/or endocapillary to crescentic glomerulonephritis.

Ans: Ref:

25853434, 42433051, 9873314110, 9953550295, 8447461114

39

DELHI ACADEMY OF MEDICAL SCIENCES PVT. LTD.


96.

MAHARASTRA PAPER - 2011

Whatever the histologic lesions, the prominent feature by fluorescent microscopy is the deposition of IgA, sometimes with IgG and C3, in the mesangial region. The skin lesions consist of subepidermal haemorrhages and a necrotizing vasculitis involving the small vessels of the dermis. IgA deposits are also present in such vessels. Vasculitis also occurs in other organs, such as the gastrointestinal tract, but is rare in the kidney. Crescentic nephritis can be seen in a) Goodpasture syndrome b) Wegeners granulomatosis c) Henoch-Schonlein purpura d) All of these d) All of these Robbins & Cotrans Pathologic Basis of Disease; 8/e, pg 920

Ans: Ref:

RPGN (crescentic glomerlonephritis) may be caused by a number of different diseases, some restricted to the kidney and others systemic. Although no single mechanism can explain all cases, there is little doubt that in most cases the glomerular injury is immunologically mediated. A practical classification divides RPGN into three groups on the basis of immunological findings. Rapidly Progressive Glomerulonephritis Type I (Anti-GBM antibody) Renal-limited Goodpasture syndrome (option a) Type II (Immune complex) Idiopathic Post-infectious glomerulonephritis Lupus nephritis Henoch-Schonlein purpura (IgA nephropathy) (option c) Others Type III (Pauci-immune) ANCA-associated Idiopathic Wegeners granulomatosis (option b) Microscopic polyangitis 97. Which of these is considered as an indicator of malignancy in pheochromocytoma? a) Extensive nuclear pleomorphism b) High mitotic rate c) Capsular and vascular invasion d) None of these d) None of these Robbins & Cotrans Pathologic Basis of Disease; 8/e, pg 1161 Determining malignancy in pheochromocytoma can be vexing. There is no histologic feature that reliably predicts clinical behavior. Several histologic features; such as number of mitoses, confluent tumour necrosis and spindle cell morphology have been associated with an aggressive behavior and increased risk of metastases, but these are not entirely reliable. Tumours with benign histologic features may metastasize, while bizzarely pleomorphic tumours may remain confined to the adrenal gland. In fact, cellular and nuclear ploemorphism, including the presence of giant cells and mitotic figures are often seen in benign pheochromocytomas, while cellular monotony is paradoxically associated with an aggressive behavior. Even capsular and vascular invasion may be encountered in benign lesions. Therefore, the definitive diagnosis of malignancy in pheochromocytomas is based exclusively on the presence of metastases. These may involve regional lymph nodes as well as more distant sites, including liver, lung and bone.

Ans: Ref:

MICROBIOLOGY 98. Temperature required in a hot air oven for a holding period of 1 hour in degree Celsius is a) 100

25853434, 42433051, 9873314110, 9953550295, 8447461114

40

DELHI ACADEMY OF MEDICAL SCIENCES PVT. LTD.


b) c) d) Ans: Ref: 120 140 160

MAHARASTRA PAPER - 2011

d) 160 Ananthanarayan and Panickers Textbook of Microbiology; 7/e, pg 29 Temperature (OC) 121 126 134 160 170 180 190 Holding time (in minutes) 15 10 3 45 18 7.5 1.5

Method Autoclave

Hot air oven

99.

Loefflers serum slope is a (m = medium) a) Basal m. b) Enriched m. c) Selective m. d) Differential m. b) Enriched m Ananthanarayan and Panickers Textbook of Microbiology; 7/e, pg 37 Enriched media: In these media, substances such as blood, serum or egg are added to a basal medium. They are used to grow bacteria which are more exacting in their nutritional needs. Eg: blood agar, chocolate agar and egg containing media.

Ans: Ref:

Note: As serum is added to Loefflers serum slope, it is a enriched medium Option (a): Basal m. Ref: Ananthanarayan and Panickers Textbook of Microbiology; 7/e, pg 37 Simple medium/basal medium An example is nutrient broth. It consists of peptone, meat extract, sodium chloride and water. Nutrient agar, made by adding 2% agar to nutrient broth is the simplest and most common medium in routine diagnostic laboratories.

Option (c): Selective medium Ref: Ananthanarayan and Panickers Textbook of Microbiology; 7/e, pg 37 Selective medium. An inhibiting substance is added to a solid medium, to prevent the growth of other organisms. It enables a greater number of required bacterium to form colonies than the other bacteria. Eg: deoxycholate citrate medium for dysentery bacilli.

Option (d): Differential medium Ref: Ananthanarayan and Panickers Textbook of Microbiology; 7/e, pg 37 Differential medium: It is a medium which has substances incorporated in it, enabling it to bring out differing characteristics of bacteria and thus, helping to distinguish between them. Eg: MacConkeys medium, which consists of peptone, lactose, agar, neutral red and taurocholate [mnemonic: PLANT] shows up lactose fermenters as pink colonies while non-lactose fermenters are colourless or pale. Which of the following statements is NOT true regarding the capsule of micro-organisms? a) It is stained by Gram stain b) It inhibits phagocytosis c) It protects against lytic enzymes d) It can be lost on repeated sub-cultures a) It is stained by Gram stain

100.

Ans:

25853434, 42433051, 9873314110, 9953550295, 8447461114

41

DELHI ACADEMY OF MEDICAL SCIENCES PVT. LTD.


Ref: Ananthanarayan and Panickers Textbook of Microbiology; 7/e, pg 14

MAHARASTRA PAPER - 2011

Capsules can be readily demonstrated by negative staining in wet films with India ink, when they are seen as clear halos around the bacteria, against a clear background. Option (b): It inhibits phagocytosis Ref: Ananthanarayan and Panickers Textbook of Microbiology; 7/e, pg 14 Capsules contribute to the virulence of pathogenic bacteria by inhibiting phagocytosis Option (c): It protects against lytic enzymes Ref: Ananthanarayan and Panickers Textbook of Microbiology; 7/e, pg 14 Capsules protect bacteria from deleterious agents such as lytic enzymes found in nature. Option (d): It can be lost on repeated sub-cultures Ref: Ananthanarayan and Panickers Textbook of Microbiology; 7/e, pg 14 Loss of the capsule by mutation may render the bacterium avirulent. Repeated subcultures in vitro lead to loss of the capsule and also of virulence. 101. The transfer of DNA from one bacterium to another through the agency of a bacteriophage is a) Transformation b) Transduction c) Lysogenic conversion d) Conjugation b) Transduction Ananthanarayan and Panickers Textbook of Microbiology; 7/e, pg 55

Ans: Ref:

The transfer of a portion of DNA from one bacterium to another by a bacteriophage is known as transduction Option (a): Transformation. Ref: Ananthanarayan and Panickers Textbook of Microbiology; 7/e, pg 55 Transformation is the transfer of genetic information through the agency of free DNA. It was the first example of genetic exchange in bacteria to have been discovered. Option (c): Lysogenic conversion Ref: Ananthanarayan and Panickers Textbook of Microbiology; 7/e, pg 56 Bacteriophages exhibit two types of life cycle. In the virulent or lytic cycle, large numbers of progeny phages are built up inside the host bacterium, which ruptures to release them. In the temperate or non-lytic cycle, the host bacterium is unharmed. The phage DNA becomes integrated with the bacterial chromosome as the prophage, which multiplies synchronously with the host DNA and is transferred to the daughter cells. This process is called lysogeny and bacteria harbouring prophages are called lysogenic bacteria. In lysogenic bacteria, the prophage behaves as an additional segment of the bacterial chromosome, coding for new characteristics. This process by which the prophage DNA confers genetic information to a bacterium is called lysogenic conversion or phage conversion.

Option (d): Conjugation Ref: Ananthanarayan and Panickers Textbook of Microbiology; 7/e, pg 56 Conjugation is a process whereby a male or donor bacterium mates or makes physical contact with a female or recepient bacterium and transfers genetic elements into it. It was first described by Lederberg and Tatum in 1946. The maleness or donor status of a cell is determined by the presence in it of a plasmid that codes for specialized fimbriae (sex pilus) which projects from the surface of the cell. The plasmid DNA replicates and a copy of it passes from the donor to the recipient cell, probably along the sex pilus. As a result, the recipient attains donor status and can in turn conjugate with other female cells.

25853434, 42433051, 9873314110, 9953550295, 8447461114

42

DELHI ACADEMY OF MEDICAL SCIENCES PVT. LTD.


102.

MAHARASTRA PAPER - 2011

The isohaemagglutinins (anti-A and anti-B) belong to which of the following categories? a) IgG b) IgM c) IgA d) IgD b) IgM Ananthanarayan and Panickers Textbook of Microbiology; 7/e, pg 89

Ans: Ref:

The isohaemagglutinins (anti-A and anti-B) and many other natural antibodies to microorganisms are usually IgM, as also antibodies to typhoid O antigen (endotoxin) and regain antibodies in syphilis. 103. The Fc portion of which of the following Ig binds to complement and fixes it? a) IgM b) IgE c) IgD d) IgA a) IgM Robbins and Cotrans Pathologic Basis of Disease; 8/e, pg 202 Opsonisation and phagocytosis: Phagocytosis is largely responsible for depletion of cells coated with antibodies. Cells opsonized by IgG antibodies are recognized by phagocyte Fc receptors, which are specific for the Fc portions of some IgG subclasses. In addition, when IgM or IgG antibodies are deposited on the surfaces of the cells, they may activate the complement system by the classical pathway. Complement activation generates by-products, mainly C3b and C4b, which are deposited on the surfaces of the cells and recognized by phagocytes that express receptors for these proteins. The net result is phagocytosis of the opsonized cells and their destruction. Complement activation of cells also leads to the formation of the membrane attack complex, which disrupts membrane integrity by drilling holes through the lipid bilayer, thereby causing osmotic lysis of the cells. This mechanism of depletion is probably effective only with cells that have thin cell walls, such as Neisseria bacteria. The most common cause of pedal botyromycosis is a) Streptomyces b) Staphylococcus aureus c) Nocardia braziliense d) All of these b) Staphylococcus aureus Ananthanarayan and Panickers Textbook of Microbiology; 7/e, pg 402

Ans: Ref:

104.

Ans: Ref:

Mycetomas are usually caused by fungi but may be caused by bacteria as well. Even Staph aureus and other pyogenic bacteria may occasionally cause a mycetoma-like lesion (botyromycosis). Ref: Ananthanarayan and Panickers Textbook of Microbiology; 7/e, pg 618

Mycetomas may be caused by a number of actiinomycetes and filamentous fungi. A similar condition called botyromycosis is caused by Staphylococcus aureus and some other bacteria. Etiological diagnosis, therefore, is of importance in treatment 105. The most commonly used laboratory test to distinguish between staphylococci and micrococci is a) Catalase test b) Coagulase test c) Hugh-Leifsons oxidative fermentive test d) c) Hugh-Leifsons oxidative fermentive test Ananthanarayan and Panickers Textbook of Microbiology; 7/e, pg 200-201

Ans: Ref:

25853434, 42433051, 9873314110, 9953550295, 8447461114

43

DELHI ACADEMY OF MEDICAL SCIENCES PVT. LTD.

MAHARASTRA PAPER - 2011

Micrococci are Gram +ve cocci which occur mostly in pairs, tetrads or irregular clusters. They are catalase +ve and oxidase +ve. They are aerobic with a strictly respiratory metabolism. They are parasitic on mammalian skin and ordinarily non-pathogenic. They resemble staphylococci but in stained smears, the cells are generally larger and more Gram variable than staphylococci. In cultures, they form smaller colonies. The common laboratory test to differentiate between micrococci and staphylococci is the Hugh and Leifsons oxidation-fermentation test, in which micrococci show oxidative and staphylococci show fermentive patterns. Which of these is known to secrete an exotoxin? a) Shigella flexneri b) Shigella boydii c) Shigella sonnei d) Shigella dysentriae d) Shigella dysentriae Ananthanarayan and Panickers Textbook of Microbiology; 7/e, pg 286

106.

Ans: Ref:

Shigella dysentriae type 1 forms a toxin (Shiga toxin), the earliest example of an exotoxin produced by a Gram negative bacillus. Three types of toxic activity have been demonstrated: Neurotoxicity Enterotoxicity and Cytotoxicity Note: 107. Sh. dysentriae is also the only member of the family that is always catalase negative. Mycobacterium avium is a) Photochromogen b) Non-photochromogen c) Scotochromogen d) Rapid grower b) Non-photochromogen Ananthanarayan and Panickers Textbook of Microbiology; 7/e, pg 365-367

Ans: Ref:

Runyons classification of atypical mycobacteria: Group I Photochromogens: these strains form colonies that produce no pigment in the dark but a yellow orange pigment appears when the young culture is exposed to light for one hour in the presence of air and reincubated for 24 28 hours. M. kansasii M. marinum M. simiae Group II Scotochromogens: These strains form pigmented colonies (yellow-orange-red) even in the dark. E.g: M. scrofulaceum M. gordonae M. szulgai Group III Non-photochromogens: These strains do not form pigment even on exposure to light. E.g: M. avium M. intracellulare M. xenopi M. malmoense M. ulcerans (skin pathogen) Group IV Rapid growers: They are capable of rapid growth, colonies appearing within 7 days of incubation at 37 OC or 25 OC. E.g: M. fortuitum M. chelonae M. phlei M. smegmatis

25853434, 42433051, 9873314110, 9953550295, 8447461114

44

DELHI ACADEMY OF MEDICAL SCIENCES PVT. LTD.


108. M. vaccae

MAHARASTRA PAPER - 2011

In Schwann cells, Mycobacterium leprae divides after a) 2 3 days b) 12 13 days c) 22 23 days d) 33 43 days

Ans: b) 12 13 days Ref: http://www.ebi.ac.uk/2can/genomes/bacteria/Mycobacterium_leprae.html Mycobacterium leprae: It is an acid fast, straight or slightly curved rod-shaped organism with parallel-sized and rounded ends. It is an obligate intracellular organism and grows and divides inside macrophages and Schwann cells. It is characterized by an extremely slow doubling time of 12-14 days and favouring temperatures of around 30OC.

[Note: M. leprae is the only species of mycobacteria to infect peripheral nerves]

109.

HACEK a) b) c) d)

group of organisms includes all of these except Haemophilus Actinobacter Corynebacterium Eikenella

Ans: Ref:

c) Corynebacterium Ananthanarayan and Panickers Textbook of Microbiology; 7/e, pg 338

The acronym HACEK refers to a group of fastidious, slow-growing bacteria, normally resident in the mouth; which can sometimes cause severe infections, particularly endocarditis. The group includes Haemophilus species (parainfluenzae, aphrophilus, paraphrophilus) Actinobacillus actinomycetemcomitans Cardiobacterium hominis Eikenella corrodens and Kingella kingae. Blood cultures from HACEK patients take 7 30 days to become positive. Antibiotic sensitivity tests are essential for effective therapy as drug resistance is very common. 110. Which of these organisms require a high concentration of serum containing cholesterol and other lipids in the medium for its growth? a) Mycobacteria b) Rickettsia c) Mycoplasma d) Chlamydia c) Mycoplasma Ananthanarayan and Panickers Textbook of Microbiology; 7/e, pg 396 111. Meida for cultivating mycoplasma are enriched with 20% horse or human serum and yeast extract. Penicillin and thallium acetate are added as selective agents. The high concentration of serum is necessary as a source of cholesterol and other lipids. Which of these Rickettsial infections cannot be diagnosed by Weil-Felix reaction? a) Epidemic typhus b) Endemic typhus c) Q fever d) Scrub typhus c) Q fever Ananthanarayan and Panickers Textbook of Microbiology; 7/e, pg 418

Ans Ref:

Ans: Ref:

25853434, 42433051, 9873314110, 9953550295, 8447461114

45

DELHI ACADEMY OF MEDICAL SCIENCES PVT. LTD.

MAHARASTRA PAPER - 2011

The Weil-Felix reaction is an agglutination test in which sera are tested for agglutinins to the O antigens of certain non-mobile Proteus strains; OX-19, OX-2 and OX-K. The test is negative in rickettsial pox, trench fever and Q fever. Disease Epidemic typhus Brill-Zinsser disease Endemic typhus Tick typhus/Tickborne spotted fever Scrub typhus Rickettsial pox Q fever Trench fever Agglutination pattern with OX-19 OX-2 +++ + Usually or weakly Usually or weakly + + +++ ++ ++ -

OX-K + -

112.

Which of the following is the causative agent of Lymphogranuloma venereum? a) Herpes simplex b) C. granulomatis (Klebsiella granulomatis) c) Treponema pallidum d) Chlamydia trachomatis d) Chlamydia trachomatis Ananthanarayan and Panickers Textbook of Microbiology; 7/e, pg 424 Human diseases caused by chlamydiae:

Ans: Ref:

Species C. trachomatis

Serotype A, B, Ba, C DK

C. psittaci C. pneumoniae

L1, L2, L3 Many serotypes Only one serotype

Disease Endemic blinding trachoma Inclusion conjunctivitis (neonatal and adult) Genital chlamydiasis Infant pneumonia Lymphogranuloma venereum Psittacosis Acute respiratory failure

113.

Which of these organisms is the cause of white piedra? a) Trichosporon beigelii b) Piedraria hortae c) Exophiala werneckii d) Pityrosporum orbiculare a) Trichosporon beigelii Ananthanarayan and Panickers Textbook of Microbiology; 7/e, pg 613 Piedra: Piedra is a fungal infection of the hair, characterized by the appearance of firm, irregular nodules along the hair shaft. The nodules are composed of fungus elements cemented together on the hair. Two varieties of piedra arre recognized o Black piedra: caused by Piedraria hortae and o White piedra: caused by Trichosporon beigelii

Ans: Ref:

Option (c): Exophiala werneckii Ref: Ananthanarayan and Panickers Textbook of Microbiology; 7/e, pg 612 Tinea nigra is a localized infection of the stratum corneum, particularly of the palms, producing black or brownish macular lesions. It is found mainly in the tropics and is caused by Cladosporium werneckii (now designated as Hortaea werneckii). Skin scrapings show brownish, branched septate hyphae and budding cells. Colonies on Saborauds medium are grey or black in colour.

Option (d): Pityrosporum orbiculare

25853434, 42433051, 9873314110, 9953550295, 8447461114

46

DELHI ACADEMY OF MEDICAL SCIENCES PVT. LTD.


Ref: 114. Ananthanarayan and Panickers Textbook of Microbiology; 7/e, pg 612

MAHARASTRA PAPER - 2011

Pityriasis versicolor (Tinea versicolor) is a chronic, usually asymptomatic involvement of the stratum corneum, characterized by discrete or confluent macular areas of discolouration or depigmentation of skin. The areas mainly involved are the chest, abdomen, upper limbs and back. The causative agent is a lipophilic yeast-like fungus Pityrosporum orbiculare (Malassezia furfur). The disease is particularly prevalent in the tropics. It occurs mainly in young adults. Diagnosis is established by examination of skin scrapings, which show an abundance of yeast-like cells and short, branched filaments (Sphagetti and meatball appearance). The fungus can be grown on Saborauds agar with a layer of olive oil. In the formol-ether method for concentration of stool specimen, the parasites are present in a) ether b) faecal specimen c) formol water d) sediment d) sediment K. D. Chatterjee: Parasitology (Protozoology and Helminthology) in relation to clinical medicine, 12/e, pg 211. Formol-ether concentration method. (Ritchiefs modified method, Ridley and Hagwood, 1956) 1 g of faeces is emulsified in 7 mL of 10% formol-saline and kept for 10 min for fixation. It is then strained through a wire gauze (40 meshes to an inch). The filtrate is collected in a centirifuge tube. To it 3 mL of ether is added and the mixture is shaken vigorously for 1 minute. It is then centrifuged at 2000 rpm for 2 min and allowed to settle. The debris are loosened with a stick, the upper part of the test tube is cleared of fatty debris and the supernatant fluid is decanted, leaving 1 or 2 drops. The deposit after shaking is poured on the glass slide, a coverslip is placed on it and the specimen is examined. It does not cause any distortion of any protozoal cysts or helminthic eggs and takes only 5 minutes. The living layer of hydatid cyst is a) Pseudocyst b) Ectocyst c) Endocyst d) None of these c) Endocyst K. D. Chatterjee: Parasitology (Protozoology and Helminthology) in relation to clinical medicine, 12/e, pg 124 Evolution (Pathogenesis) of hydatid cyst The cyst wall secreted by the embryo consists of 2 layers Outer cuticular layer (Ectocyst) It is a laminated hyaline membrane having a thickness of upto 1 mm. To the naked eye, the ectocyst has the appearance of the white of a hard-boiled egg. It is elastic and when incised or ruptured, curls on itself, exposing the inner layer containing the brood capsules and daughter cysts. Inner Germinal layer (Endocyst) It is cellular and consists of a number of nuclei embedded in a protoplasmic mass. It is very thin and measures about 22 to 25 m in thickness. It is the vital layer of the cyst and (a) gives rise to brood capsules (b) secretes the specific hydatid fluid (c) forms the outer layer Regression metamorphosis is observed in a) Hydatid cyst b) c) Bovine tapeworm d) Beef tapeworm a) Hydatid cyst

Ans: Ref:

115.

Ans: Ref:

1. 2.

116.

Ans:

25853434, 42433051, 9873314110, 9953550295, 8447461114

47

DELHI ACADEMY OF MEDICAL SCIENCES PVT. LTD.


Ref: 126

MAHARASTRA PAPER - 2011

K. D. Chatterjee: Parasitology (Protozoology and Helminthology) in relation to clinical medicine, 12/e, pg 124

Endogenous daughter cyst formation in hydatid cysts is the result of growth over many years and is therefore particularly seen in man. The daughter cysts develop inside the mother cyst and may arise from the detached fragment of the germinal layer or from regressive changes of the young brood capsule or the scolex bud. The daughter cysts also consist of an outer protective layer and an inner germinative layer from which brood capsules and scolices arise and even grand-daughter cysts may develop. It is to be observed that transformation of scolex into a daughter cyst within the same host, without passing through the adult stage in the definitive host (a phenomenon observed by Naunyn as early as 1862), is against the law of migration of cestodes enunciated by van Beneden. A fully developed scolex is the end-product and is unable to undergo regressive changes and develop into a cyst. Experimental work has confirmed that E. granulosus does not follow the general biological laws and Deve (1925) confirmed that regressive metamorphosis could occur within the hydatid scolices. Fish is the intermediate host for all of the following parasites except a) Paragonimus westermanii b) Clonorchis sinensis c) Diphyllobothrium latum d) Opistorchis viverinni a) Paragonimus westermanii K. D. Chatterjee: Parasitology (Protozoology and Helminthology) in relation to clinical medicine, 12/e, pg 155 Life cycle: P. westermanii passes its life cycle in three hosts 1 definitive and 2 intermediate. Definitive hosts: Man and other domestic animals. Usual hosts in Asia are the tiger and the leopard. Feline hosts serve as reservoirs of infection. Intermediate hosts: o First host: A fresh-water snail of the genus Melania (The optimum snail host is Semisulcospira libertina. Syn: Melania libertina) o Second host: A fresh-water crayfish or a crab

117.

Ans: Ref:

Option (b): Clonorchis sinensis Ref: K. D. Chatterjee: Parasitology (Protozoology and Helminthology) in relation to clinical medicine, 12/e, pg 151 Life cycle: C. sinensis passes its life cycle in three hosts 1 definitive and 2 intermediate. Definitive hosts: Man, dog, cat, pig. Adult worms are in the biliary tracts of the liver Intermediate hosts: where the larval development proceeds o First host: A snail of the subfamily Buliminae (Bithyniinae) o Second host: A cyprinoid fish.

Option (c): Diphyllobothrium latum Ref: K. D. Chatterjee: Parasitology (Protozoology and Helminthology) in relation to clinical medicine, 12/e, pg 112 Life cycle: D. latum passes its life cycle in three hosts 1 definitive and 2 intermediate. Definitive hosts: Man, dog and cat; man is the optimum host. The adult worms are found in the small intestine Intermediate hosts: Acquatic animals, where the larval stages are passed: o First host: A fresh-water crustacean, a Cyclops or a diaptomus. o Second host: A fresh-water fish: pike, trout, salmon, perch and other fish.

Option (d): Opistorchis viverinni Ref: K. D. Chatterjee: Parasitology (Protozoology and Helminthology) in relation to clinical medicine, 12/e, pg 151 Family Opisthorchiidae These are flat, transparent, medium-sized worms living in the biliary passages of fish-eating animals. Species infecting man are included in two genera: 1. Genus: Clonorchis. Species: C. sinensis 2. Genus: Opistorchis. Species: O. felineus, O. viverinni. The normal host of Opistorchis viverinni is the civet cat (Felis viverrus) (pg. 154)

118.

Filariform larvae is the infective form of a) Ascaris lumbricoides b) Enterobius vermicularis

25853434, 42433051, 9873314110, 9953550295, 8447461114

48

DELHI ACADEMY OF MEDICAL SCIENCES PVT. LTD.


c) d) Ans: c) Necator americanus Strogyloides stercoralis Necator americanus & d) Strogyloides stercoralis

MAHARASTRA PAPER - 2011

Option (c): Necator americanus Ref: K. D. Chatterjee: Parasitology (Protozoology and Helminthology) in relation to clinical medicine, 12/e, pg 174 Infecting agent: Filariform larva Portal of entry: Skin Migration: through lungs Site of location: Small intestine Option (d): Strongyloides stercoralis Ref: K. D. Chatterjee: Parasitology (Protozoology and Helminthology) in relation to clinical medicine, 12/e, pg 168 Infecting agent: Filariform larva Portal of entry: Skin Site of location: Lungs, Intestine

Option (a): Ascaris lumbricoides Ref: K. D. Chatterjee: Parasitology (Protozoology and Helminthology) in relation to clinical medicine, 12/e, pg 184 Infecting agent: Embryonated egg Portal of entry: Alimentary canal Migration: through lungs Site of location: Small intestine

Option (b): Enterobius vermicularis Ref: K. D. Chatterjee: Parasitology (Protozoology and Helminthology) in relation to clinical medicine, 12/e, pg 179 Life cycle: Each of the eggs, newly laid on the perianal skin, containing a tadpole-like larva, completes its development in 24-36 hrs time, in the presence of oxygen. Infection occurs by ingestion of these eggs. The eggs are dissolved by digestive juices and the larvae escape in the small intestine where they develop into adolescent worms. 119. Treatment with mebendazole single dose 100 mg (repeated 3 weeks later) is done for a) Enterobiasis b) Strongyloidiasis c) Trichinellosis d) Ascariasis a) Enterobiasis K. D. Tripathi: Essentials of Medical Pharmacology, 6/e, pg 809-810. Uses and administration of mebendazole Roundworm, Hookworm and Trichuris: 100 mg twice a day for 3 consecutive days. No fasting, purging or any other preparation of patients is needed. Enterobius: 100 mg single dose repeated after 2 3 weeks (to kill the ova that have developed later) Trichinella spiralis: 200 mg BD for 4 days (less effective than albendazole) Hydatid disease: 200 400 mg BD/TDS for 3 4 weeks (less effective than albendazole) Involvement of pulmonary lobules with visceral larva migrans is seen in a) Ascariasis b) Toxocara canis c) Strongyloidiasis d) All of these d) All of these

Ans: Ref: 1. 2. 3. 4.

120.

Ans:

Option (a): Ascariasis Ref: K. D. Chatterjee: Parasitology (Protozoology and Helminthology) in relation to clinical medicine, 12/e, pg 184 Migration through lungs:

25853434, 42433051, 9873314110, 9953550295, 8447461114

49

DELHI ACADEMY OF MEDICAL SCIENCES PVT. LTD.


MAHARASTRA PAPER - 2011

The larvae liberated in the small intestine do not directly develop into mature worms. The newly hatched larvae burrow their way through the mucous membrane of the small intestine and are carried by the portal circulation to the liver; here they live for a period of 3 4 days. Finally they pass out of the liver and via the right heart enter the pulmonary circulation. While in the lungs they grow much bigger and increase in length from 0.2 mm to 2 mm and moult twice (first, on the 5th or 6th day and the second, after the 10th day). Breaking through the capillary wall they reach the lung alveoli. The time taken for such migration is on an average 10 15 days.

Ref: K. D. Chatterjee: Parasitology (Protozoology and Helminthology) in relation to clinical medicine, 12/e, pg 206 207 Symptoms due to migrating larvae in the lungs Ascaris pneumonia (Loefflers syndrome). In heavy infections, typical symptoms of pneumonia like fever, cough and dyspnoea may appear. Sputum which is often blood-tinged may contain Ascaris larvae. Option (b): Toxocara canis Ref: K. D. Chatterjee: Parasitology (Protozoology and Helminthology) in relation to clinical medicine, 12/e, pg 206 207 Visceral larva migrans: Here, after the ingestion of infective stage of the eggs, the second stage larvae are hatched out in the intestine, which invade the intestinal wall and are carried to the extra-intestinal viscera, such as liver, lungs and other organs. In any of these sites, the progress is arrested by the formation of a granulomatous lesion. The common etiological agent for the development of such lesion is infection with a dog ascarid (Toxocara canis) or a cat ascarid (Toxocara catis)

Option (c): Strongyloidiasis: Ref: K. D. Chatterjee: Parasitology (Protozoology and Helminthology) in relation to clinical medicine, 12/e, pg 206 207 Pulmonary lesions: Haemorrhages in the lung alveoli and bronchopneumonia develop during migration of filariform larvae through the lungs and form an avenue of escape into the alveoli.

According to me, the best answer to the above question is option (d): all of these. But if the option (d) is changed into some other option, then the single best answer in that case would be option (b) Toxocara canis since it is the most common etiological agent of visceral larva migrans. FMT 121. A suit for negligence against a doctor must be filed within what time period? a) 6 months b) 12 months c) 2 years d) 3 years c) 2 years K.S.Narayan Reddys The Essentials of Forensic Medicine and Toxicology; 28/e, pg 37 122. A suit for damages for negligence against a doctor should be filed within 2 years from the date of alleged negligence. A suit filed after two years will be dismissed as being beyond the period of limitation.

Ans: Ref:

For an appeal against disciplinary action, final verdict is given by a) Central Government b) State Government c) Central Medical Council d) State Medical Council a) Central Government K.S.Narayan Reddys The Essentials of Forensic Medicine and Toxicology; 28/e, pg 21 Appeal against disciplinary action: If the name of any person is removed from the State Medical Register, he may appeal to the Central Government, after exhausting all remedies under the State Medical Council Act. Every such appeal should state the grounds of the appeal and be accompanied by all relevant documents within 30 days from the date of the decision appealed against.

Ans: Ref:

25853434, 42433051, 9873314110, 9953550295, 8447461114

50

DELHI ACADEMY OF MEDICAL SCIENCES PVT. LTD.

MAHARASTRA PAPER - 2011

The decision of the Central Government, which is given after consulting the Indian Medical Council, is binding on the State Government and the State Medical Council. For determining the age between 1-13 years, the best X-ray is a) Elbow b) Shoulder c) Hand and wrist d) Clavicle c) Hand and wrist K.S.Narayan Reddys The Essentials of Forensic Medicine and Toxicology; 28/e, pg 72 Estimation of age: 2 6 years: Ossification of tarsus and carpus and appearance of centres in epiphyses of long bones. The number of carpal bones seen on X-ray indicate the approximate age in years. 6 13 years: Eruption and calcification of permanent teeth is very helpful. Alterations occur in the centres that have already appeared and additional centres appear.

123.

Ans: Ref:

Now, read further: Ref: Parikhs 6/e, pg 2.10 X-ray Elbow Wrist Shoulder Crest of ilium Ischial tuberosity Clavicle (inner end) Females 13-14 16-17 17-18 18-19 21-22 21-22 Age Males 15-16 18-19 19-20 20-21 23-24 23-24

However, Reddy clearly mentions that carpals can be used to detect age less than 13 years and hence, is the single best answer of choice in this question. 124. In case of death due to police firing, the inquest shall be ordered by which of the following? a) b) c) Magistrate d) c) Magistrate K.S.Narayan Reddys The Essentials of Forensic Medicine and Toxicology; 28/e, pg 5

Ans: Ref:

Magistrates inquest: This is conducted by a District Magistrate (Collector / Deputy Commisioner), Subdivisional Magistrate (RDO), Tahsildar or any other Executive Magistrate. It is done in case of: 1. death in police custody, and while under police interrogation 2. death due to police firing 3. death in prison, reformatories, Borstal school 4. death in a psychiatric hospital 5. dowry deaths and 6. exhumation. 125. La facies sympathiteque is indicative of a) Ante-mortem hanging b) Post-mortem hanging c) Drowning d) Throttling a) Ante-mortem hanging K.S.Narayan Reddys The Essentials of Forensic Medicine and Toxicology; 28/e, pg 300 In case of hanging, the eyes are frequently protruded and firmer than usual due to congestion. The conjunctivae are usually congested. The eyes are closed or partially open, and the pupils are usually dilated.

Ans: Ref:

25853434, 42433051, 9873314110, 9953550295, 8447461114

51

DELHI ACADEMY OF MEDICAL SCIENCES PVT. LTD.


126.

MAHARASTRA PAPER - 2011

If the ligature knot presses on the cervical sympathetic chain, the eye on the same side may remain open and its pupil dilated. This is called la facies sympathetique. It indicates ante-mortem hanging In a case of salt-water drowning, in the first 8 12 minutes, all of these features occur except a) Massive pulmonary edema b) Severe hypovolemia c) Shock d) Acute hemolysis and hyperkalemia d) Acute hemolysis and hyperkalemia K.S.Narayan Reddys The Essentials of Forensic Medicine and Toxicology; 28/e, pg 321-322 Drowning in sea water: Due to the high salinity of the sea water (usually over 3% NaCl), water is drawn from the blood into the lung tissue and produces severe pulmonary edema and hypernatremia. This causes haemoconcentration. Simultaneously in an attempt to re-establish osmotic balance, salts from the water in the lungs pass into the bloodstream. A marked bradycardia occurs, probably due to the raised plasma sodium level. Slow death occurs from asphyxia.

Ans: Ref:

Note: As water is drawn into the lungs from the blood, it is logical that hypovolemia and shock may ensue. Drowning in fresh water: In drowning in fresh water (0.6% NaCl), two-and-a half litres of water may be inhaled and absorbed in three minutes; blood volume may increase by 50% causing a great strain on the heart due to hypervolemia. Haemodilution leads to haemolysis, haemoglobinemia and haemoglobinuria, marked hyponatremia and hyperkalemia The form of sexual perversion in which an individual achieves sexual gratification only by infliction of pain upon him by the partner is a) Sadism b) Fetichism c) Frotteurism d) Masochism d) Masochism K.S.Narayan Reddys The Essentials of Forensic Medicine and Toxicology; 28/e, pg 367

127.

Ans: Ref:

In masochism, sexual gratification is obtained or increased by the suffering of pain. Masochists get pleasure from being beaten, abused, tortured, humiliated, enslaved, degraded or dominated by their sexual partner, and they tend to place themselves repeatedly in self-defeating positions. Option (a): Sadism Ref: K.S.Narayan Reddys The Essentials of Forensic Medicine and Toxicology; 28/e, pg 366 In sadism, sexual gratification is obtained or increased from acts of physical cruelty or infliction of pain upon ones partner. Option (b): Fetichism Ref: K.S.Narayan Reddys The Essentials of Forensic Medicine and Toxicology; 28/e, pg 367 In fetichism, the person experiences sexual excitement leading to orgasm from part of the body of a woman or some article belonging to her that normally has no sexual influence on the mind, e.g: underclothing, brassiere, petticoat, stocking, shoes, etc. which act as substitute for the female love object. Option (c): Frotteurism Ref: K.S.Narayan Reddys The Essentials of Forensic Medicine and Toxicology; 28/e, pg 368 Frotteurism is contact with another person in order to obtain sexual satisfaction. Sexual satisfaction is obtained by rubbing his private parts against a females body in crowds. 128. The best preservative for preserving viscera for toxicological analysis is a) Saturated solution of common salt

25853434, 42433051, 9873314110, 9953550295, 8447461114

52

DELHI ACADEMY OF MEDICAL SCIENCES PVT. LTD.


b) c) d) Ans: Ref: 1. 2. 3. 4. 5. 6. 1. 2. 10% formalin Rectified spirit 0.9% NaCl

MAHARASTRA PAPER - 2011

a) Saturated solution of common salt K.S.Narayan Reddys The Essentials of Forensic Medicine and Toxicology; 28/e, pg 110-111 Viscera should be preserved if: Death is suspected to be due to poisoning either by the police or the doctor. Deceased was intoxicated or used to drugs Cause of death was not found after autopsy Death was due to burns Advanced decomposition Accidental death involving driver of a vehicle or machine operator. Preservatives for viscera: Saturated sodium chloride solution: It is NOT used in poisoning from corrosive substances EXCEPT Phenol Alkalis Corrosive sublimate Aconite

Rectified spirit, except in cases of Alcohol Kerosene Chloroform Chloral hydrate Ether Formaldehyde Formic acid Acetic acid Phenol Phosphorus Paraldehyde The organic acids and paraldehyde are soluble in alcohol and the phosphorescence of phosphorus is diminished by alcohol. 3. Sodium/Potassium fluoride, 10 mL For blood, urine, CSF and vitreous and for analysis of cocaine, cyanide and CO

The viscera should not be preserved in formaldehyde because the extraction of poisons, especially nonvolatile organic compounds becomes difficult. 1. 2. 3. 4. 5. 6. 129. Viscera to be preserved Stomach and its contents. The upper part of the small intestine (about 30 cm in length) and its contents Liver. 200 300 g Kidney. of each Blood. 30 mL (Minimum 10 mL) Urine. 30 mL Blackouts in a person of alcohol consumption is seen in which of the following conditions? a) Alcohol dependence b) Alcohol abstinence c) Alcohol intoxication d) Alcohol withdrawl c) Alcohol intoxication K.S.Narayan Reddys The Essentials of Forensic Medicine and Toxicology; 28/e, pg 510 Alcoholic palimpsests (Alcoholic blackouts): It is a condition seen among alcoholics and rarely in the non-addictive drinker, after drinking a moderate amount of alcohol. The behavior resembles the blackouts in anoxemia. This may result in the loss of memory of a period during the drinking spell, or in some cases, the inability to recall what happened over a period of days.

Ans: Ref:

25853434, 42433051, 9873314110, 9953550295, 8447461114

53

DELHI ACADEMY OF MEDICAL SCIENCES PVT. LTD.


MAHARASTRA PAPER - 2011

Amnesia can be fragmented or total. In the latter case, the memory with regard to the lost time is unlikely to return. It is a late manifestation of alcoholism. During such state, the person may perform a criminal act and may not remember it after he recovers from the effects of intoxication. Copper a) b) c) d) sulphate is used as an antidote in which of these poisonings? Cyanide poisoning Phosphorus poisoning Arsenic poisoning Barbiturate poisoning

130.

Ans: Ref:

b) Phosphorus poisoning Krishan Vijs Textbook of Forensic Medicine and Toxicology: Principles and Practice; 5/e, pg 570 Copper sulphate is used as an antidote in phosphorus poisoning.

Ref: http://chestofbooks.com/health/materia-medica-drugs/Pharmacology-Therapeutics/Copper-Cupri-SulphasCupri-Sulphatis-Copper-Sulphate-U.html Copper sulphate is the chemical antidote for phosphorus-poisoning, yet it should be given with great caution, lest of itself it produce features of copper sulphate poisoning. 131. Massive a) b) c) d) hepatic necrosis can be caused by which of the following poisons? Carbon tetrachloride Yellow phosphorus Edible poison of Mushroom All of the above

Ans: d) All of the above Ref: http://en.wikipedia.org/wiki/Acute_liver_failure, http://www.fpnotebook.com/GI/Pharm/HptxcMdctn.htm and www.bsg.org.uk/pdf_word_docs/postgrad_06f.ppt Acute liver failure: Acute liver failure is defined as the rapid development of hepatocellular dysfunction, specifically coagulopathy and mental status changes (encephalopathy) in a patient without known prior liver disease. The exact definition of "rapid" is somewhat questionable, and different sub-divisions exist which are based on the time from onset of first hepatic symptoms to onset of encephalopathy. One scheme defines "acute hepatic failure" as the development of encephalopathy within 26 weeks of the onset of any hepatic symptoms. This is sub-divided into "fulminant hepatic failure" (massive hepatic necrosis), which requires onset of encephalopathy within 8 weeks, and "subfulminant", which describes onset of encephalopathy after 8 weeks but before 26 weeks. Another scheme defines "hyperacute" as onset within 7 days, "acute" as onset between 7 and 28 days, and "subacute" as onset between 28 days and 24 weeks.

D/D of acute liver failure: 1. Viral Hepatitis A Hepatitis B Hepatitis C Hepatitis D Hepatitis E (mainly in pregnant females) 2. Drug-induced Amoxicillin-Clavulanic acid (MC) Acetaminophen Anti-Tuberculosis medications Trimethoprim-sulfamethoxazole Minocycline (mimics Autoimmune Hepatitis) Nitrofurantoin (mimics Autoimmune Hepatitis) Valproic Acid Niacin 3. Toxins Carbon tetrachloride Phosphorous

25853434, 42433051, 9873314110, 9953550295, 8447461114

54

DELHI ACADEMY OF MEDICAL SCIENCES PVT. LTD.


4. 5. Amanita phalloides (Mushroom) Vascular events Ischemic hepatitis Budd-Chiari VOD Heat shock liver Other Pregnancy related Wilson disease Lymphoma

MAHARASTRA PAPER - 2011

Symptoms Vomiting Upper Abdominal Pain Anorexia Jaundice Signs Neurologic changes (Hepatic Encephalopathy) o Altered Level of Consciousness (Delirium, coma) o Decerebrate rigidity (with severe cerebral edema) o Personality changes Jaundice Coagulopathy Bleeding (e.g. Gastrointestinal Bleeding) Acute Renal Failure (Hepatorenal Syndrome) Hypoglycemia Acute Pancreatitis Cardiopulmonary failure Ascites (due to Portal Hypertension) Prognosis: Factors associated with poor outcomes Advanced age Halothane exposure Hepatitis C Coma (80% mortality) Rapid decrease in liver span Respiratory failure Marked ProTime prolongation Factor V Level <20% Management ABC Management Endotracheal intubation often required Monitor Serum Glucose (Correct Hypoglycemia) Prevent GI Bleed H2 Blockers to maintain gastric pH >3.5 Lower Intracranial Pressure with IV Mannitol Liver Transplant Indications o Grade 3-4 Encephalopathy o Adverse prognostic indicators as above The most potent of the preparations of cannabis is a) Majun b) Charas c) Bhang d) Ganja b) Charas K.S.Narayan Reddys The Essentials of Forensic Medicine and Toxicology; 28/e, pg 534 Preparations of cannabis: Bhang (siddhi/sabji): It is prepared from dried leaves and fruit shoots. It is used as we use tea to prepare a decoction. It is the mildest and contains 15% of the active principle.

132.

Ans: Ref: 1.

25853434, 42433051, 9873314110, 9953550295, 8447461114

55

DELHI ACADEMY OF MEDICAL SCIENCES PVT. LTD.


2. 3. 4. 133.

MAHARASTRA PAPER - 2011

Fresh bhang is highly intoxicating and narcotic. Bhang kept in storage for 2 3 years is mildly stimulating and pleasure-giving. Majoon: It is a sweet prepared with bhang. It increases the appetite and sexual desire. Ganja (pot/grass/weed/maryjone): It is prepared from the flower tops of the female plant. It has a rusty-green colour and a characteristic odour. It is mixed and smoked with tobacco in a pipe or hukka. It contains 15-25% of the active principle. It is also used for smoking in cigarettes, which contain 0.3 to 0.6 g of cannabis and are known as reefer or joint. Charas (Hashish): It is the resin (dope or shit) exuding from the leaves and stems of the plant. It contains 25-40% of the active principle. It is dark-green or brown in colour. It is mixed with and smoked with tobacco in a pipe or hukka. The smoke is inhaled deeply into the lungs and retained for as long as possible for potent effects. Prazosin a) b) c) d) is effective in treating hypertension caused by the bite of which of the following? Russels viper Mesobuthus tamulus (Indian Red Scorpion) Naja naja All of these

Ans: b) Mesobuthus tamulus (Indian Red Scorpion) Ref: Bawaskar H.S. & Bawaskar P.H. Management of the cardiovascular manifestations of poisoning by the Indian red scorpion (Mesobuthus tamulus). British Heart Journal. 1992; 68(5). 62 patients who had been stung by a red scorpion were admitted from January to December 1990: 18 with hypertension, 15 with supraventricular tachycardia, 11 with pulmonary oedema, and 18 with local pain at the site of sting but no systemic involvement. The combination of nifedipine and prazosin was more successful in preventing myocardial damage in 16 patients with hypertension than was nifedipine alone in two other patients with hypertension. Prazosin alone helped to alleviate the cardiovascular manifestations in eight patients with pulmonary oedema and 15 with supraventricular tachycardia. One patient with pulmonary oedema died and two recovered after they were given intravenous sodium nitroprusside. Thus, nifedipine alone did not prevent myocardial damage unless the peripheral action of venom was blocked by prazosin.

Ref: Bawaskar HS, Bawaskar PH. Utility of scorpion antivenin vs prazosin in the management of severe Mesobuthus tamulus (Indian red scorpion) envenoming at rural setting. Journal of Association of Physicians in India. 2007; 55:14-21. Scorpion antivenom (SAV) is the specific antidote to scorpion venom. SAV did not prevent the cardiovascular morbidity and mortality (autonomic storm), hence its utility in the management for severe scorpion envenoming may be limited. Since 1983 the advent of prazosin revolutionized the management of severe scorpion sting. Since 2002 SAV against Indian red scorpion for the treatment of scorpion sting cases is available at primary health centers. We found that SAV is no more effective to alleviate or reverse the cardiovascular effects of scorpion venom actions in severe case as against prazosin, which prevents and cures the cardiovascular manifestations in a severe scorpion envenomation. Therefore role of SAV in severe scorpoin venomation needs to be relooked and prazosin needs to be a standard of care in such cases to overcome the autonomic storm.

PSM 134.

PQLI includes which of the following a) IMR, Literacy, Life Expectancy at birth b) IMR, Literacy, Life Expectancy at age one c) IMR, Life Expectancy at age one, Per capita income

25853434, 42433051, 9873314110, 9953550295, 8447461114

56

DELHI ACADEMY OF MEDICAL SCIENCES PVT. LTD.


d) Ans: Ref: 1. MMR, IMR and Per capita income

MAHARASTRA PAPER - 2011

b) IMR, Literacy, Life Expectancy at age one Parks Textbook of Preventive and Social Medicine; 20/e, pg 16-18 Composite indicators: PQLI (mnemonic OIL) Life Expectancy at age One Infant mortality rate Literacy HDI (Mnemonic: BPL) Life Expectancy at Birth Per Capita Income Literacy Minimum Value 25 yrs 0% $100 0% Maximum value 85 yrs 100% $ 40,000 100%

2.

Indicator Life Expectancy at Birth Per capita income: Combined gross enrolment ratio Real GDP per capita Adult literacy rate Formula:

Actual value Minimum value Maximum value Minimum value

Value: 0-1 Current value for India: 3. HPI for developing countries:

P1 = Long and healthy life: Probability of not surviving to age 40 P2 = Knowledge: Adult literacy rate P3 = A decent standard of living: Percentage of children under weight-for-age Percentage of children not using an improved water source. Formula:[1/3 (P1 + P2 + P3 )]1/ , where = 3 Current value for India: 31.3% HPI for developed countries: P1 = Probability at birth of not surviving to age 60 (times 100) P2 = Percentage of adults lacking functional literacy skills P3 = Percentage of population below income poverty line (50% of median adjusted household disposable income. P4 = Rate of long-term unemployment (lasting 12 months or more) Formula: [1/4 (P1 + P2 + P3 + P4 )]1/ , where = 3 135. Sullivans index is which of the following? a) Mortality indicator b) Health care delivery indicator c) Disability indicator d) Nutritional status indicator c) Disability indicator Parks Textbook of Preventive and Social Medicine; 20/e, pg 24-26

Ans: Ref:

Sullivans index: This index (expectation of life free from disability) is computed by subtracting from the life expectancy the probable duration of bed disability and inability to perform major activities, according to the crosssectional data from the population surveys. Sullivans index is considered one of the most advanced indicators currently available. 1. 2. 3. 4. 5. 6. Option (a): Mortality indicators Crude death rate Expectation of life Infant mortality rate Child mortality rate Under-5 proportionate mortality rate Maternal mortality rate

25853434, 42433051, 9873314110, 9953550295, 8447461114

57

DELHI ACADEMY OF MEDICAL SCIENCES PVT. LTD.


7. 8. 1. 2. 3. 4. 5. 1. Disease-specific mortality rate Proportional mortality rate Option (b): Health care delivery indicators Doctor-population ratio Doctor-nurse ratio Population-bed ratio Population per health/subcentre Population per traditional birth attendant Option (c): Disability indicators Event-type indicators Number of days of restricted activity Bed disability Work-loss days (or school-loss days) within a specified period Person-type indicators Limitation of mobility Limitation of activity Sullivans index HALE (Health-Adjusted Life Expectancy) DALY (Disability-Adjusted Life Year) Option (d): Nutritional status indicators Anthropometric measurements of preschool children Heights (and sometimes weights) of children at school entry Prevalence of low birth weight (less than 2.5 kg) Other indicators: Morbidity indicators Incidence & Prevalence Notification rates Attendance rates at out-patient departments Admission, readmission and discharge rates Duration of stay in hospital Spells of sickness or absence of work from school

MAHARASTRA PAPER - 2011

2.

1. 2. 3.

1. 2. 3. 4. 5. 6. 1. 2. 3. 4. 5. 6. 1. 2. 3. 4. 5. 6. 7. 8. 136.

Utilization rates Proportion of infants who are fully immunized against the 6 EPI diseases Proportion of pregnant women who receive antenatal care, or have their deliveries supervised by a trained birth attendant Percentage of population using the various methods of family planning Bed-occupancy rate Average length of stay Bed turn-over ratio Socio-economic indicators Rate of population increase Per capita GNP Level of unemployment Dependency ratio Literacy rates, especially female literacy rates Family size Housing (the number of persons per room) Per capita calorie availability. The time interval between the receipt of infection by a host and the maximum infectivity is known as a) generation time b) serial interval c) latent period d) quarantine period a) generation time Parks Textbook of Preventive and Social Medicine; 20/e, pg 95 Generation time: It is defined as the interval of time between the receipt of infection by a host and maximal infectivity of that host.

Ans: Ref:

25853434, 42433051, 9873314110, 9953550295, 8447461114

58

DELHI ACADEMY OF MEDICAL SCIENCES PVT. LTD.


MAHARASTRA PAPER - 2011

In general, generation time is roughly equal to the incubation period. However, these two terms are not the same. The time of maximum communicability may precede or follow the incubation period. A further difference is that the term incubation period can only be applied to infections that result in manifest disease, whereas generation time refers to transmission of infection, whether clinical or subclinical.

Option (b): Serial interval Ref: Parks Textbook of Preventive and Social Medicine; 20/e, pg 95 The gap in time between the onset of the primary case and the secondary case is called the serial interval. By collecting information about a whole series of such onsets, we get a distribution of secondary cases from which we can guess the incubation period of the disease

Option (c): Latent period Ref: Parks Textbook of Preventive and Social Medicine; 20/e, pg 94 The term latent period is used in non-infectious diseases as the equivalent of incubation period in infectious diseases. Latent period has been defined as the period from disease initiation to disease detection. In chronic diseases, the agent-host interactions leading to a sequence of cellular changes are not well understood.

Option (d): Quarantine period Ref: Parks Textbook of Preventive and Social Medicine; 20/e, pg 94 Incubation period is useful in determining the period of surveillance (or quarantine) which may be advised. This period is usually equal to the maximum incubation period of the disease. Ref: Parks Textbook of Preventive and Social Medicine; 20/e, pg 111

Quarantine has been defined as the limitation of freedom of movement of such well persons or domestic animals exposed to communicable diseases for a period of time not longer than the longest usual incubation period of the disease, in such manner as to prevent effective contact with those not so exposed. 137. The denominator for the calculation of positive predictive value includes a) true positives + false negatives b) true negatives + false positives c) true positives + false positives d) true negatives + false negatives c) true positives + false positives Parks Textbook of Preventive and Social Medicine; 20/e, pg 126-127 Diseased a (True positives) c (False negatives) a+c Not diseases b (False positives) d (True negatives) b+d Total a+b c+d a+b+c+d

Ans: Ref:

Screening test results Positive Negative

25853434, 42433051, 9873314110, 9953550295, 8447461114

59

DELHI ACADEMY OF MEDICAL SCIENCES PVT. LTD.

MAHARASTRA PAPER - 2011

138.

The strain used for the production of the mumps vaccine is a) Edmonston-Zagreb b) RA 27/3 c) Jeryll-Lynn d) Oka c) Jeryll-Lynn Parks Textbook of Preventive and Social Medicine; 20/e, pg 141 Widely-used live attenuated mumps vaccine strains include o Jeryll-Lynn o RIT 4385 o Leningrad-3 o L-Zagreb and o Urabe strains. Live attenuated mumps vaccine strains used only on a limited scale include the Hoshino, Torii and NKM-46 strains. The current mumps strain (Jeryll-Lynn) has the lowest associated incidence of post-vaccine aseptic meningitis (from 1 in 1,50,000 to 1 in 1.8 million). The WHO recommends that the Rubini mumps vaccine strain should not be used in national immunization programmes because of its demonstrated low effectiveness. Ananthanarayan and Panickers Textbook of Microbiology; 7/e, pg 514-515 The Jeryll-Lynn strain of mumps virus, attenuated by passage in eggs and grown in chick embryo fibroblast culture is used as the vaccine. It is recommended for use only after 1 year of age as maternal antibodies may interfere with the multiplication of the vaccine virus if given earlier. Contraindications are pregnancy, immunodeficiency and hypersensitivity to neomycin or egg protein. The vaccine is given as a single subcutaneous injection, either alone or in combination with measles and rubella vaccines (MMR vaccines). It provides effective protection for at least ten years. The vaccine may not prevent the disease if given after exposure to the infection but there is no contraindication for its use in this situation.

Ans: Ref:

Ref:

Option (a): Edmonston-Zagreb Ref: Parks Textbook of Preventive and Social Medicine; 20/e, pg 141 The observations have suggested that the HDC-Edmonston Zagreb strain vaccine protects children from 4 6 months of age from measles. Ref: Ananthanarayan and Panickers Textbook of Microbiology; 7/e, pg 520 The original live vaccines used the Edmonston strain developed by multiple passage through human kidney, amnion and chick embryo cultures. Due to its high risk of causing febrile rash (vaccination measles), further attenuation became necessary. The Schwartz and Moraten strains so developed were safe but effective only in children older than 15 months. In the tropics, measles is common and serious in children below 12 months. Therefore, the Edmonston-Zagreb strain, attenuated by passage in human diploid cells, is preferable as the vaccine strain because it is able to produce seroconversion even in infants 4 6 months old.

Option (b) RA 27/3 Ref: Parks Textbook of Preventive and Social Medicine; 20/e, pg 140 Since the isolation of the rubella virus in 1962, several live attenuated vaccines have been developed. In 1979, the RA 27/3 vaccine, produced in human diploid fibroblasts has replaced all the other vaccines. This is because RA 27/3 vaccine induces higher antibody titres and produces an immune response more closely paralleling natural infection than other vaccines. There is evidence that it largely prevents subclinical superinfection with wild virus. RA 27/3 is administered in a single dose of 0.5 mL subcutaneously. Seroconversion occurs in more than 95% vaccinees.

25853434, 42433051, 9873314110, 9953550295, 8447461114

60

DELHI ACADEMY OF MEDICAL SCIENCES PVT. LTD.


MAHARASTRA PAPER - 2011

Vaccine-induced immunity persists in most vaccinees for at least 14 to 16 years and is probably life-long. There is no evidence in favour of administration of second dose. It may provoke mild reactions in some subjects like malaise, fever, mild rash and transient arthralgia, but no serious disability. Pregnancy is considered a contraindication to rubella immunization. The recipients of the vaccine should be advised not to become pregnant over the next 3 months. Infants under one year should not be vaccinated due to the possible interference with persisting rubella antibody.

Option (d) Oka Ref: Ananthanarayan and Panickers Textbook of Microbiology; 7/e, pg 480 139. A live varicella vaccine was developed by Takahashi in Japan in 1974 by attenuating a strain of varicella virus (Oka strain, so named after the patient) by serial passage in tissue culture. Given subcutaneously, it induced a good antibody response, but it was very labile and had to be stored frozen. A modified lyopjilised form of the vaccine is now available, which can be stored between 2OC and 8OC. It is recommended for children 1 12 years old as a single subcutaneous dose, and for those older as 2 doses 6 10 weeks apart. It is safe and effective. Which of the following about the Sabin vaccine is not true? a) Three doses are given for primary immunization b) It is given at intervals of 4-6 weeks. c) It is given intramuscularly d) It contains all the 3 strains of poliovirus c) It is given intramuscularly Parks Textbook of Preventive and Social Medicine; 20/e, pg 180 Oral (Sabin) polio vaccine (OPV) OPV was described by Sabin in 1957. It contains live attenuated virus (types 1,2 and 3) (option d) grown in primary monkey kidney or human diploid cell cultures. The vaccine contains: (i) over 3,00,000 TCID50 of type 1 poliovirus (ii) over 1,00,000 TCID50 of type 2 virus, and (iii) over 3,00,000 TCID50 of type 3 virus per dose. The WHO programme on EPI and the National Immunization Programme in India recommend a primary course of 3 doses of OPV (option a) at one-month intervals, (option b) commencing the first dose when the infant is 6 weeks old. The dose is 2 drops or as stated on the label. WHO recommends that vaccinators use dropper supplied with the vial of oral polio vaccine. This is the most direct and effective way to deliver the correct drop size. Tilt the childs back, and gently squeeze the cheeks or pinch the nose to make the mouth open. Let the drops fall from the dropper onto the childs tongue. Repeat the process if the child spits out the vaccine. If the vaccine is spoon-fed, there is a chance that it will not all be licked up by the child. The booster dose of tetanus toxoid is given at intervals of a) 13 years b) 10 years c) 5 years d) 1 year c) 5 years Parks Textbook of Preventive and Social Medicine; 20/e, pg 273-274 A primary course of immunization consists of two doses of tetanus toxoid adsorbed (each dose 0.5 mL, injected into the arm) given at intervals of 1 2 months. The longer the intervals between the two doses, the better is the immune response. The first booster dose (the third in order) should be given a year after the two initial doses. The opinion was expressed that no more than an additional booster dose (a total of 4 doses altogether) given 5 years after the third dose is required in adults (including pregnant women) in developing countries. Frequent boosters must be avoided.

Ans: Ref:

140.

Ans: Ref:

25853434, 42433051, 9873314110, 9953550295, 8447461114

61

DELHI ACADEMY OF MEDICAL SCIENCES PVT. LTD.

MAHARASTRA PAPER - 2011

Ref:

Ananthanarayan and Panickers Textbook of Microbiology; 7/e, pg 261 A full course of immunization confers immunity for a period of at least ten years. A booster dose of toxoid is recommended after ten years.

So, why is the answer 5 years and not 10 years? Read on to understand Ref: Ananthanarayan and Panickers Textbook of Microbiology; 7/e, pg 631

National Immunization Schedule of India: Age At birth 6 weeks 10 weeks 14 weeks 9 months 16-24 months 5-6 years (at school entry) 10 years 16 years For pregnant women One month after TT-1

Vaccine BCG, OPV-0 BCG (if not given at birth), OPV-1, DPT-1 OPV-2, DPT-2 OPV-3, DPT-3 Measles OPV booster, DPT booster DT booster TT booster TT booster TT-1 TT-2

Thus, according to the National Immunization Schedule of India, TT booster is to be given every 5 years. So, option c is the best answer of choice 141. Which of these parameters is an indicator of recent transmission of malaria in a community? a) Slide positivity rate b) Annual blood examination rate c) Spleen rate d) Infant parasite rate d) Infant parasite rate Parks Textbook of Preventive and Social Medicine; 20/e, pg 227-228 Measurement of malaria: PRE-ERADICATION ERA 1. 2. 3. 4. 5. 6. Spleen rate the % of children between 2 to 10 years of age showing enlargement of spleen. The spleen rate is widely used for measuring the endemicity of malaria in a community. Average enlarged spleen average size of the enlarged spleen. Parasite rate - % of children between 2 to 10 years of age showing malarial parasites in their blood films. Parasite Density Index average degree of parasitemia in a sample of well-defined group of the population. Infant parasite rate - % of infants showing malarial parasites in their blood films. It is regarded as the most sensitive index of recent transmission of malaria in a locality. If the infant parasite rate is zero for 3 consecutive years in a locality, it is regarded as absence of malaria transmission. Proportional case rate Number of cases diagnosed as clinical malaria for every 100 patients attending the hospitals and dispensaries. ERADICATION ERA Annual parasite incidence = Confirmed cases during one year x 100 Population under surveillance Annual blood examination rate = Number of slides examined x 100 Population ABER is an index of operational efficiency. Annual falciparum incidence = Number of slides positive for P. falciparum Population Slide positivity rate Slide falciparum rate Obesity is said to be morbid if the BMI is above x 100

Ans: Ref:

1. 2.

3.

4. 5. 142.

25853434, 42433051, 9873314110, 9953550295, 8447461114

62

DELHI ACADEMY OF MEDICAL SCIENCES PVT. LTD.


a) b) c) d) Ans: Ref: Ref: 143. 30 35 40 45

MAHARASTRA PAPER - 2011

c) 40 Sabistons Textbook of Surgery; 18/e, pg Morbid obesity is defined as being either 100 lb above ideal body weight, twice ideal body weight, or a body mass index (BMI, measured as weight in kilograms divided by height in meters squared) of 40 kg/m2. The latter definition is more accepted internationally and has essentially replaced the former ones for all practical and scientific purposes. http://health.nytimes.com/health/guides/symptoms/morbid-obesity/overview.html The term morbid obesity refers to patients who are 50 - 100% or 100 pounds above their ideal body weight. Alternatively, a BMI (body mass index) value greater than 39 may be used to diagnose morbid obesity. Which of the following studies was done to assess whether community health education can decrease the risk of cardiovascular disease? a) The Stanford Three Community Study b) The North Keralia Project c) The Framingham Study d) MRFIT trial a) The Stanford Three Community Study Parks Textbook of Preventive and Social Medicine; 20/e, pg 322 The Stanford-Three-Community Study: To determine whether community health education can reduce the risk of cardiovascular disease, a field experiment was undertaken in 1972 in three northern California towns with populations varying between 12,000 and 15,000. In two of these towns, intensive mass education campaigns were conducted against cardiovascular risk factors over a period of 2 years. The third community served as a control. In the control community, the risk of cardiovascular disease increased over the two years, but in the intervention communities, there was a substantial and sustained decrease in risk. The net difference in the estimated total risk between control and intervention samples was 23 28 %.

Ans: Ref:

Option (b): The North Keralia Project. Ref: Parks Textbook of Preventive and Social Medicine; 20/e, pg 322 North Keralia is a county in the eastern part of Finland. A multiple risk factor intervention trial was started in 1972. The project had two aims: (a): to reduce the high levels of risk factors for cardiovascular disease (smoking, BP and serum cholesterol) (b): to promote the early diagnosis, treatment and rehabilitation of patients with cardiovascular disease. Follow-up surveys at 5 years demonstrated a significant reduction in all three major risk factors. By 1979, mortality began to decrease by 24% in men and 51% in women in North Keralia, compared with 12% in men and 26& in women in rest of Finland.

Option (c): The Framingham Study Ref: Parks Textbook of Preventive and Social Medicine; 20/e, pg 321 Since 1951, one of the best known large prospective studies, the Framingham study, has played a major role in establishing the nature of CHD risk factors and their relative importance. The major risk factors for CHD are elevated serum cholesterol smoking hypertension & sedentary habits. Accordingly, the four main possibilities of intervention in CHD prevention are reduction in serum cholesterol cessation of smoking control of hypertension & promotion of physical activity.

25853434, 42433051, 9873314110, 9953550295, 8447461114

63

DELHI ACADEMY OF MEDICAL SCIENCES PVT. LTD.


Option (d): MRFIT trial Ref: Parks Textbook of Preventive and Social Medicine; 20/e, pg 322

MAHARASTRA PAPER - 2011

The multiple risk factor intervention trial (MRFIT) carried out in USA was aimed at high risk adult males aged 35 57 years. A total of 12,866 men who showed no evidence of CHD either clinically or on ECG were enrolled for the study. Half the group was randomly allocated to an intensive intervention programme, being seen at least every 4 months to ensure adequate control of risk factors. The other half (control group) received a medical examination once yearly, and no specific advice was given to them about the control of risk factors. The intervention programmes included cessation of smoking, control of BP and altering diet to reduce hypercholesterolemia. Over a 7-year follow-up period, IHD mortality was reduced by 22% in the intervention group but this was not statistically significant. This was because the control group had also changed their habits and life-style to a far greater extent than anticipated by the designers of the trial. The trial produced no significant changes at all in mortality or risk factors in much as the control group was not properly chosen. According to the definition as per RNTCP, tubercular peritonitis is considered to fall under which category? a) I b) II c) III d) IV a) I K. D. Tripathi. Essentials of Medical Pharmacology, 6/e, pg ____; 5/e, pg 706 707

144.

Ans: Ref:

Category I: This category includes 1. New (untreated) smear +ve pulmonary TB 2. New smear ve pulmonary TB with extensive parenchymal involvement 3. New cases of severe forms of extra-pulmonary TB, viz. meningitis, military, pericarditis, peritonitis, B/L or extensive pleural effusion, spinal, intestinal, genitourinary. Category II. These are smear +ve failure, relapse and interrupted treatment cases. 1. Treatment failure Patient who remains or again becomes smear +ve 5 months or later after commencing treatment. Also one who was smear ve at start of therapy and becomes smear +ve after the second month. 2. Relapse A patient declared cured from any form of TB in the past after receiving one full course of chemotherapy and now has become sputum +ve. 3. Treatment after interruption (Default) A patient who interrupts treatment for 2 months or more and returns with sputum +ve or clinically active TB. Category III. This includes 1. New cases of smear ve pulmonary TB with limited parenchymal involvement 2. Less severe forms of extrapulmonary TB viz. lymph node, U/L pleural effusion, bone (excluding spine), peripheral joint or skin TB Category IV. These are chronic cases who have remained or have become sputum +ve after completing fully supervised re-treatment (Category II regimen). These are most likely MDR cases. 145. According to the National Leprosy Elimination Programme, Maharashtra is placed in which of the following categories? a) 1 b) 2 c) 3 d) 4 b) 2 Parks Textbook of Preventive and Social Medicine; 19/e, pg 351 MODIFIED LEPROSY ELIMINATION CAMPAIGN (MLEC): A mid-term appraisal of the National Leprosy Elimination Programme in April 1997 indicated that while the progress of the programme is satisfactory at national level, it is uneven in some states. It was decided to launch a leprosy elimination campaign by giving short-term orientation training in leprosy to health staff including medical officer, health workers and volunteers; increase public awareness about

Ans: Ref:

25853434, 42433051, 9873314110, 9953550295, 8447461114

64

DELHI ACADEMY OF MEDICAL SCIENCES PVT. LTD.

MAHARASTRA PAPER - 2011

leprosy; and house-to-house search to be conducted to detect new leprosy cases throughout the country for a period of six days. The first round was conducted during 1997-98. Five such campaigns were carried out in the country. The fourth campaign was different from the first three campaigns. In this, the states were divided into three categories: Bihar Uttar Pradesh Chhattisgarh Madhya Pradesh West Bengal Uttaranchal Orissa Andhra Pradesh Arunachal Pradesh Goa Gujarat Karnataka Maharashtra Tamil Nadu Chandigarh Daman & Diu Dadra & Nagar Haveli Podicherry Andaman & Nicobar islands Delhi Lakshadweep Haryana Punjab Himachal Pradesh Nagaland Sikkim Tripura Meghalaya Mizoram Jammu and Kashmir Assam Manipur Rajasthan Kerala

Category 1 8 endemic states

Category 2 14 moderate-tolow endemic states

Category 3 13 very low endemic states

146.

For which of the following is the SAFE strategy recommended? a) Trachoma b) River blindness c) Herpes simplex keratitis d) Steroid-induced glaucoma a) Trachoma A. K. Khuranas Comprehensive Ophthalmology; 4/e, pg 432 Elimination of blindness due to trachoma is considered feasible, eradication of trachoma is not. Effective interventions have been demonstrated in developing countries using the SAFE strategy Surgery to correct the lid deformities and prevent blindness. Antibiotics for acute infections and community control. Facial hygiene and Environmental change including improved access to water and sanitation and health education The financial aid for the National Program for Control of Blindness was provided by a) US AID b) World Bank c) SIDA d) DANIDA

Ans: Ref:

147.

25853434, 42433051, 9873314110, 9953550295, 8447461114

65

DELHI ACADEMY OF MEDICAL SCIENCES PVT. LTD.


Ans: Ref: d) DANIDA Parks Textbook of Preventive and Social Medicine, 20/e, pg 822

MAHARASTRA PAPER - 2011

DANIDA: The Government of Denmark is providing assistance for the development of services under National Blindness Control Programme since 1978. Ref: Parks Textbook of Preventive and Social Medicine, 20/e, pg 376

Danish assistance to NPCB: During Phase III of Danish assistance (1998-2003) funds were utilized for the training, development of management information system, supply of equipment, preparation of health education material and support to Karnataka. 148. The doubling time of population if the population growth rate is 1.5 to 2.0 is a) 23 28 years b) 46 70 years c) 37 46 years d) 28 35 years c) 37 46 years Parks Textbook of Preventive and Social Medicine, 20/e, pg 413

Ans: Ref:

Relation between growth rate and population: Rating Annual growth rate (%) Stationary Slow growth Moderate growth Rapid growth Very rapid growth Explosive growth No growth < 0.5 0.5 1.0 1.0 1.5 1.5 2.0 2.0 2.5 2.5 3.0 3.0 3.5 3.5 4.0

No. of years required for doubling of population > 139 139 70 70 47 (option b) 47 35 35 28 (option d) 28 23 (option a) 23 20 20 18

149.

The most common complication of IUCD is a) Sterility b) Hypofibrinoginemia c) Cervical laceration/tear d) None of these d) None of these Parks Textbook of Preventive and Social Medicine, 20/e, pg 428 The commonest complaint of women fitted with an IUCD (inert or medicated) is increased vaginal bleeding. It accounts for 10 20% of all IUD removals. The bleeding may take one or more of the following forms: o Greater volume of blood loss during menstruation (menorrhagia) o Longer menstrual periods (polymenorrhoea) or o Mid-cycle bleeding (metrorrhagia) The patient who is experiencing bleeding episodes should receive iron tablets (ferrous sulphate, 200 mg three times daily) Which of the following gives the calculation for the first day of the fertile period in the rhythm method? a) First day of cycle 10 days b) First day of cycle 18 days c) Last day of cycle 18 days d) Last day of cycle 10 days b) First day of cycle 18 days Parks Textbook of Preventive and Social Medicine, 20/e, pg 436 Safe period / Rhtyhm method:

Ans: Ref:

150.

Ans: Ref:

25853434, 42433051, 9873314110, 9953550295, 8447461114

66

DELHI ACADEMY OF MEDICAL SCIENCES PVT. LTD.


MAHARASTRA PAPER - 2011

This is also known as calendar method, first described by Ogino in 1930. The method is based on the fact that ovulation occurs from 12 to 16 days before the onset of menstruation. The shortest cycle minus 18 days gives the first day of the fertile period. The longest cycle minus 10 days gives the last day of the fertile period. For example, if a womans menstrual cycle varies from 26 to 31 days, the fertile period during which she should not have intercourse would be: 26 18 = 8th day to 31 10 = 21st day of the menstrual cycle. For international comparison, stillbirth is defined for the weight of a fetus exceeding a) 1000 g b) 750 g c) 500 g d) 250 g a) 1000 g Parks Textbook of Preventive and Social Medicine, 20/e, pg 483

151.

Ans: Ref:

Some observers have expressed the view that vital statistics reports are less reliable on foetal deaths occurring at 20 27 weeks than those occurring after 28 completed weeks and have preferred to analyze the data separately for the two intervals. Stillbirths are seldom reported in developing countries. Because of the above difficulties, WHO has recommended that within any country, the term stillbirth be applied to a foetus born dead, and weighing over 500 g the birth weight associated with a gestation period of 22 weeks. But for international comparison, however, they suggested a boundary of 1000 g or more, which is more frequently associated with a gestation period of 28 weeks. In the WHO growth chart, the upper reference line corresponds to a) 97th percentile for boys b) 50th percentile for girls c) 50th percentile for boys d) 3rd percentile for girls c) 50th percentile for boys Parks Textbook of Preventive and Social Medicine, 20/e, pg 468-469

152.

Ans: Ref: Note:

The WHO prototype growth chart has two reference curves. The upper reference curve represents the median (50th percentile) for boys, which is slightly higher than that for girls. The lower reference curve represents the 3rd percentile for girls, which is slightly lower than that for boys. Thus, the chart can be used for both the sexes. The space between the two growth curves (weight channel) has been called the road-to-health.

Growth chart recommended by the Government of India has four reference curves. The topmost curve corresponds to 80% of the median (50th percentile) of the WHO reference standard. The lower lines represent 70%, 60% and 50% of that standard. The purpose of these lines is to indicate the degree of malnutrition, as recommended by the Indian Academy of Paediatrics. Which of the following parameters gives the best measure of the quality of protein? a) Digestibility co-efficient b) Net protein utilization c) Biological value d) Amino acid score b) Net protein utilization Parks Textbook of Preventive and Social Medicine, 20/e, pg 549

153.

Ans: Ref:

Assessment of protein quality: 1. Amino acid score: It is a measure of the concentration of each essential amino acid in the test protein expressed as a percentage of that amino acid in the reference protein (usually egg protein). The amino acid (chemical) score is somewhere between 50 and 60 for starches, and 70 and 80 for animal foods.

25853434, 42433051, 9873314110, 9953550295, 8447461114

67

DELHI ACADEMY OF MEDICAL SCIENCES PVT. LTD.

MAHARASTRA PAPER - 2011

2.

Net protein utilization (NPU): It is a product of digestibility coefficient and biological value divided by 100. NPU = Nitrogen retained in the body x 100 Nitrogen intake The NPU gives a more complete expression of protein quality than the amino acid score. It is a biological method that requires special laboratory facilities. In calculating protein quality, 1 gram of protein is assumed to be equivalent to 6.25 g of N If the NPU is low, the dietary requirement is high and vice versa. The NPU of Indian diets varies between 50 and 80. Nalgonda technique is for a) Dechlorination of water b) Defluoridation of water c) Deiodinisation of water d) Filtering of water for cyclops b) Defluoridation of water Parks Textbook of Preventive and Social Medicine, 20/e, pg 549 The National Environmental Engineering Research Insititute (NEERI), Nagpur, has developed a technique for removing fluoride by chemical treatment. It is called Nalgonda technique for defluoridation of water. It involves the addition of two chemicals (lime and alum) in sequence followed by flocculation, sedimentation and filtration. The study of human physical, cultural and social relationships is known as a) sociology b) social psyhcology c) political science d) anthropology d) anthropology Parks Textbook of Preventive and Social Medicine, 20/e, pg 583 Anthropology: The word anthropology is derived from the root words anthropos meaning man and logos meaning science. It is the study of physical, social and cultural history of man. The study of human evolution, racial differences, inheritance of bodily traits, growth and decay of human organism is called physical anthropology. The study of the development and various types of social life is called social anthropology. The study of the total way of life of contemporary primitive man, his ways of thinking, feeling and action is called cultural anthropology. Medical anthropology deals with the cultural component in the ecology of health and disease. Of all the sciences which study various aspects of man, anthropology is the one which comes nearest to being a total study of man.

154.

Ans: Ref:

155.

Ans: Ref:

Option (a): Sociology Ref: Parks Textbook of Preventive and Social Medicine, 20/e, pg 583 Sociology is derived from the Latin socio meaning society and the Greek logos meaning study. Sociology deals with the study of human relationships and human behavious for a better understanding of the pattern of human life. It is also concerned with the effects on the individual of the ways in which other individuals think and act.

Option (b): Social psychology Ref: Parks Textbook of Preventive and Social Medicine, 20/e, pg 583 This discipline sprang from psychology. It is concerned with the psychology of the individuals living in human society or groups. The emphasis is on understanding the basis of perception, thought, opinion, attitudes, general motivation and learning in individuals and how these vary in human society and groups. In other words, it deals with the effect of social environment on perons, their attitudes and their motivations.

25853434, 42433051, 9873314110, 9953550295, 8447461114

68

DELHI ACADEMY OF MEDICAL SCIENCES PVT. LTD.

MAHARASTRA PAPER - 2011

Option (c): Political science Ref: Parks Textbook of Preventive and Social Medicine, 20/e, pg 583 Historically, economics and political science tended to be a single discipline. As a separate discipline, political science is concerned with the study of the system of laws and institutions which constitute government of whole societies. The Juvenile Delinquency Act defines a juvenile as under what age? a) 21 years b) 18 years c) 16 years d) 12 years b) 18 years http://en.wikipedia.org/wiki/Juvenile_delinquency Juvenile delinquency is participation in illegal behavior by minors (juveniles) who fall under a statutory age limit A juvenile delinquent is a person who is typically under the age of 18 and commits an act that otherwise would've been charged as a crime if they were an adult. Juvenile delinquents sometimes have associated mental disorders and/or behavioral issues such as post traumatic stress disorder or bipolar disorder, and are sometimes diagnosed with conduct disorder partially as both the cause and resulting effects of their behaviors. On addition of OT reagent to water, what will be the colour developed if the water contained free chlorine? a) Yellow b) Blue c) Pink d) Green a) Yellow Parks Textbook of Preventive and Social Medicine, 20/e, pg 626 ORTHOTOLIDENE (OT) TEST: The OT test enables both free and combined chlorine in water to be determined with speed and accuracy. The test was developed in 1918. The reagent consists of analytical grade Orthotolidene, dissolved in 10% solution of HCl. When this reagent is added to water containing chlorine, it turns yellow and the intensity of the colour varies with the concentration of the gas. The yellow colour is produced by both free and combined chlorine residuals. OT reacts with free chlorine instantaneously but reacts more slowly with combined chlorine. ORTHOTOLIDENE ARSENITE (OTA) TEST: This is a modification of the OT test to determine the free and combined chlorine residuals separately. Further, the errors caused by the presence of interfering substances such as nitrates, iron and manganese, all of which produce a yellow colour with Orthotolidene, are overcome by the OTA test. Which of these disease/s is/are transmitted by soft ticks? a) KFD b) Q fever c) Relapsing fever d) All of these d) All of these Parks Textbook of Preventive and Social Medicine, 20/e, pg 684-685 Public health importance of ticks: Hard ticks transmit: Tick typhus (Rocky mountain spotted fever) Viral encephalitis (E.g. Russian spring-summer encephalitis) Viral fevers (e.g. Colorado tick fever) Viral haemorrhagic fevers (e.g. Kyasanur forest disease [KFD] in India) Tularemia Tick paralysis

156.

Ans: Ref:

157.

Ans: Ref:

158.

Ans: Ref:

1. 2. 3. 4. 5. 6.

25853434, 42433051, 9873314110, 9953550295, 8447461114

69

DELHI ACADEMY OF MEDICAL SCIENCES PVT. LTD.


7. 1. 2. 3. 159. Human babesiosis Soft ticks transmit: Q fever Relapsing fever KFD Borrelia a) b) c) d) recurrentis is transmitted by which of the following arthropods? Louse Mite Tick Rat flea

MAHARASTRA PAPER - 2011

Ans: Ref:

a) Louse Ananthanarayan and Panickers Textbook of Microbiology; 7/e, pg 386 Relapsing fever (RF) is an arthropod-borne infection. Two types of RF occur louse-borne and tick-borne. The causative agent of epidemic or louse-borne RF is Borrelia recurrentis, first observed by Obermeier in 1873. Over ten species of borrelia are known to cause tick-borne (option c) or endemic RF (B. duttonii, B. hermsii, B. parkeri, etc.) They are generally confined to certain geographical areas. In the disaster management programme, which of these is not considered as a fundamental aspect of disaster management? a) Disaster response b) Disaster mitigation c) Disaster awareness d) Disaster preparedness c) Disaster awareness Parks Textbook of Preventive and Social Medicine, 20/e, pg 700-701 There are three fundamental aspects of disaster management: Disaster response Disaster preparedness and Disaster mitigation. These three aspects of disaster management correspond to different phases in the so-called disaster cycle.

160.

Ans: Ref:

161.

Thermophilus is involved as a cause in all of the following disorders except a) Bagassosis b) Farmers lung c) Mushroom workers lung d) Mill workers lung d) Mill workers lung Harrisons Principles of Internal Medicine; 18/e, pg 2116-2117 In farmers lung (option b), inhalation of proteins, such as thermophilic bacteria and fungal spores that are present in moldy bedding and feed, are most commonly responsible for the development of hypersensitivity pneumonitis. These antigens are probably also responsible for the etiology of mushroom workers disease (option c) (moldy composted growth medium), bagassosis (option a) (moldy sugar cane) and water-related exposure (molds in air conditioners or humidifers)

Ans: Ref:

Option (a): Bagassosis: Ref: Parks Textbook of Preventive and Social Medicine, 20/e, pg 711 Bagassosis is the name given to an occupational disease of the lung caused by inhalation of bagasse or sugarcane dust. Bagassosis has been shown to be due to a thermophilic actinomycete for which the name Thermoactinomyces sacchari was suggested.

Option (b): Farmers lung.

25853434, 42433051, 9873314110, 9953550295, 8447461114

70

DELHI ACADEMY OF MEDICAL SCIENCES PVT. LTD.


Ref: Parks Textbook of Preventive and Social Medicine, 20/e, pg 712

MAHARASTRA PAPER - 2011

Farmers lung is due to the inhalation of mouldy hay or grain dust. In grain dust or hay with a moisture content of over 30%, bacteria and fungi grow rapidly, causing a rise of temperature to 40-50OC. This heat encourages the growth of thermophilic actinomycetes, of which Micropolyspora faeni is the main cause of farmers lung.

[Note: Micropolyspora faeni is now known as Saccharopolyspora rectivirgula] Option (c): Mushroom workers lung Ref: Harrisons Principles of Internal Medicine; 18/e, pg 2117 Mushroom workers lung is caused by thermophilic actinomycetes present in Hypsizygus marmoreus, Bunashimeji and other exotic mushrooms. The source of antigen is the mushroom or its compost.

Option (d): Mill workers lung Ref: Harrisons Principles of Internal Medicine; 18/e, pg 2117 162. Millers lung or mill workers lung is caused by Sitophilus granarius (wheat weevil) present in infested wheat flour. The weevil occurs as a common pest in granaries and can cause significant damage to harvested grains. Hypersensitivity pneumonitis in a case of aspergillosis is almost always to a) A. flavus b) A. fumigatus c) A. nedulans d) A. niger b) A. fumigatus Harrisons Principles of Internal Medicine; 17/e, pg 1256, 1257, 1608 Farmers lung is the term most commonly used for hypersensitivity pneumonitis due to inhalation of antigens present in mouldy hay, such as thermophilic actinomycetes Micropolyspora faeni and Aspergillus species. Aspergillus fumigatus is the most common cause of aspergillosis but A. flavus, A. niger, A. nedulans, A. terreus and several other species can also cause disease. A. fumigatus is responsible for most cases of invasive aspergillosis, almost all cases of chronic aspergillosis and most allergic syndromes.

Ans: Ref:

[Note: Allergic syndromes associated with aspergillosis: Allergic bronchopulmonary aspergillosis Severe asthma with fungal colonization Extrinsic allergic alveolitis (hypersensitivity pneumonitis) Allergic fungal sinusitis Eosinophilic fungal rhinosinusitis] 163. According to the Biomedical Waste Rules, the yellow bag is used for the disposal of a) Human anatomical waste b) Cytotoxic drugs c) Incineration ash d) Waste sharps a) Human anatomical waste Parks Textbook of Preventive and Social Medicine, 20/e, pg 698-699 Waste category Human anatomical waste (human tissues, organs, body parts) Animal waste (animal tissues, organs, body parts, carcasses, bleeding parts, fluids, blood and experimental animals used in research, waste generated by veterinary hospitals, colleges, discharge from hospital, animal house) Treatment & Disposal Incineration/Deep burial Incineration/Deep burial

Ans: Ref: Cat. No 1 2

25853434, 42433051, 9873314110, 9953550295, 8447461114

71

DELHI ACADEMY OF MEDICAL SCIENCES PVT. LTD.


3

MAHARASTRA PAPER - 2011


Local autoclaving / Microwaving / Incineration

Microbiology and biotechnology waste (waste from laboratory cultures, stocks or specimens of micro-organisms, live or attenuated vaccines, human and animal cell culture used in research and infectious agents from research and industrial laboratories, waste from production of biologicals, toxins, dishes and devices and for transfer of cultures Waste sharps (needles, syringes, scalpels, blades, glass, etc. that may cause puncture and cuts. This includes both used and unused sharps)

Discarded medicines and cytotoxic drugs (wastes comprising of outdated, contaminated and discarded medicines) Solid waste (items contaminated with blood and fluids; including cotton, dressings, soiled plaster casts, linen, bedding and other material contaminated with blood) Solid waste (wastes generated from disposable items other than the waste sharps such as tubings, catheters, intravenous sets, etc.)

Liquid waste (waste generated from laboratory and washing, cleaning, housekeeping and disinfecting activities) Incineration ash (ash from incineration of any bio-medical waste) Chemicals used in production of biological, disinfection, as insecticides, etc.

9 10

Disinfection (chemical treatment / autoclaving / microwaving and mutilation shredding) Incineration / Disposal in secure landfills Incineration / Autoclaving / Microwaving Disinfection by chemical treatment / Autoclaving / Microwaving and mutilation shredding Disinfection by chemical treatment and discharge into drains Disposal in municipal landfill Chemical treatment and discharge into drains for liquids and secured landfill for solids

Colour Yellow Red Black Blue / white translucent

Type of container Plastic bag Disinfected container/ Plastic bag Plastic bag Plastic bag

Waste category Cat. 1,2,3,6 Cat. 3,6,7 Cat. 5,9,10 (solid) Cat. 4,7

Treatment options Incineration/Deep burial Autoclaving / Microwaving / Chemical treatment Disposal in secure landfill Autoclaving / Microwaving / Chemical treatment f/b Destruction / Shredding

164.

Which of the following is the best to compare two variables with different units of measurement in a population? a) Standard deviation b) Variance c) d) Coefficient of variance d) Coefficient of variance M. C. Gupta and B. K. Mahajans Textbook of Preventive and Social Medicine; 3/e, pg 377 Coefficient of variation (CV) It is the ratio of the standard deviation to the mean (expressed as percentage). It is used to compare the relative dispersion in two series of dissimilar measurements. It may also be used to compare the variability of one character in two groups, such as height in boys and girls, or of two characters in the same group, such as blood pressure and height in boys. PERT and CPM are used for which of the following a) Cost-benefit analysis b) Network analysis c) d) Input-output analysis b) Network analysis

Ans: Ref:

165.

Ans:

25853434, 42433051, 9873314110, 9953550295, 8447461114

72

DELHI ACADEMY OF MEDICAL SCIENCES PVT. LTD.


Ref: Parks Textbook of Preventive and Social Medicine, 20/e, pg 775.

MAHARASTRA PAPER - 2011

Network analysis: A network is a graphic plan of all events and activities to be completed in order to reach an end objective. It brings greater discipline in planning. The two common types of network technique are PERT and CPM PERT (Programme Evaluation and Review Technique): It is a management technique which makes possible more detailed planning and more comprehensive supervision. Every housewife who plans a meal so that each part of the menu is completed at the same time is using the basic technique of PERT. The essence of PERT is to construct an arrow diagram. The diagram represents the logical sequence in which events must take place. It is possible with such a diagram to calculate the time by which each activity must be completed and to identify those activities that are critical. This simple technique provides a basic discipline by which all concerned in a project can know what is expected of them and to minimize any delays or crises in the implementation of the plan. CPM (Critical Path Method). The longest path of the network is called the critical path. If any activity along the critical path is delayed, the entire project will be delayed. How many of the millennium development goals are directly related to health? a) 1 out of 8 b) 2 out of 8 c) 3 out of 8 d) 4 out of 8 c) 3 out of 8 Parks Textbook of Preventive and Social Medicine, 20/e, pg 794.

166.

Ans: Ref:

In September 2000, representatives from 189 countries met at the Millenium Summit in New York, to adopt the United Nations Millenium Declaration. The goals in the area of development and poverty eradication are now widely referred to as MIllenium Development Goals. 3 of the 8 goals, 8 of the 18 targets required to achieve them and 18 of the 48 indicators of progress are health-related.

OPHTHAL 167. The normal radius of the anterior surface of the cornea is a) 8.4 mm b) 7.8 mm c) d) b) 7.8 mm A. K. Khuranas Comprehensive Ophthalmology; 4/e, pg 89 Dimensions of cornea: The anterior surface of the cornea is elliptical with an average horizontal diameter of 11.7 mm and a vertical diameter of 11 mm. The posterior surface of cornea is circular with an average diameter of 11.5 mm Thickness of cornea in the centre is about 0.52 mm while at the periphery is about 0.7 mm. Radius of curvature: The central 5 mm area of the cornea forms the powerful refracting surface of the eye. The anterior and posterior radii of curvature of this central part of cornea are 7.8 mm and 6.5 mm respectively. Refractive power of the cornea is about 45D, which is roughly three-fourth of the total refractive power of the eye (60D). The visible range for the human eye is a) 200 290 nm b) 290 310 nm c) 310 400 nm d) 400 700 nm

Ans: Ref:

168.

25853434, 42433051, 9873314110, 9953550295, 8447461114

73

DELHI ACADEMY OF MEDICAL SCIENCES PVT. LTD.

MAHARASTRA PAPER - 2011

Ans: Ref:

d) 400 700 nm A. K. Khuranas Comprehensive Ophthalmology; 4/e, pg 19 Light is the visible portion of the electromagnetic radiation spectrum. It lies between ultraviolet and infrared portions, from 400 nm at the violet end of the spectrum to 700 nm at the red end. The white light consists of seven colours denoted by VIBGYOR (violet, indigo, blue, green, yellow, orange and red). The laser that is used for laser-assisted in-situ keratomileusis (LASIK) is a) Argon b) Nd-YAG c) Excimer d) CO2 c) Excimer Parsons Diseases of the Eye; 20/e, pg 74

169.

Ans: Ref:

Laser-assisted in situ keratomileusis (LASIK) with the excimer laser is the most popular and widely accepted treatment modality among all the choices available at present for myopia. Surgical correction is only undertaken in individuals 21 years of age and above, who have a stable refractive error, who have had unsatisfactory results with non-surgical treatment and in whom corneal thinning disorders such as keratoconus have been ruled out 170. Angular a) b) c) d) conjunctivitis is characteristically caused by Staphylococcus aureus Moraxella lacunata Corynebacterium diphtheriae Neisseria gonorrhoeae

Ans: Ref:

b) Moraxella lacunata A. K. Khuranas Comprehensive Ophthalmology; 4/e, pg 61

Angular conjunctivitis: It is a type of chronic conjunctivitis characterized by mild grade inflammation confined to the conjunctiva and lid margins near the angles, (hence the name) associated with maceration of the surrounding skin. 1. Etiology: Causative organism: Moraxella axenfeld is the commonest causative organism. Moraxella axenfeld bacilli are placed end to end, so the disease is also called diplobacillary conjunctivitis. Rarely, staphylococci may also cause angular conjunctivitis. Source of infection: is usually the nasal cavity. Mode of infection: Infection is transmitted from the nasal cavity to the eyes by contaminated fingers or handkerchief. Pathology: The causative organism (Moraxella axenfeld) produces a proteolytic enzyme, which acts by macerating the epithelium of the conjunctiva, lid margin and the skin surrounding the angles of the eye. This proteolytic enzyme collects at the angles by the action of tears. The maceration is followed by vascular and cellular responses in the form of mild grade chronic inflammation. Skin may show eczematous changes. Symptoms: Irritation, smarting sensation and feeling of discomfort in the eyes. h/o collection of dirty-white foamy discharge at the angles. Redness of eyes at the angles. Signs: Hyperaemia of bulbar conjunctiva near the canthi. Hyperaemia of lid margins near the angles. Excoriation of skin around the angles.

2. 3.

25853434, 42433051, 9873314110, 9953550295, 8447461114

74

DELHI ACADEMY OF MEDICAL SCIENCES PVT. LTD.


Presence of foamy, mucopurulent discharge at the angles. Complications: Blepharitis Shallow marginal catarrhal corneal ulceration.

MAHARASTRA PAPER - 2011

Treatment: Oxytetracycline (1%) eye ointment 2-3 times a day for 9-14 days. Zinc lotion instilled in daytime and zinc oxide ointment at bedtime inhibits the proteolytic ferment and helps in reducing the maceration. A slit limp examination showing keratic precipitates and cells in anterior chamber points towards the diagnosis of a) Chronic open angle glaucoma b) Conjunctivitis c) Bacterial keratitis d) Iridocyclitis d) Iridocyclitis A. K. Khuranas Comprehensive Ophthalmology; 4/e, pg 141-145 Anterior uveitis (Iridocyclitis)

171.

Ans: Ref:

1. 2. 3. 4. 5. 6. 1. 2. 3.

Symptoms: Pain. It is the dominating symptom of acute anterior uveitis. Redness Photophobia Blepharospasm Excessive lacrimation Defective vision. Signs: Lid edema Circumcorneal congestion. Corneal signs: a) Corneal edema b) Keratic precipitates (KPs): These are proteinaceous cellular deposits occurring at the back of cornea. Mostly, these are arranged in a triangular fashion occupying the centre and inferior part of the cornea due to convection currents in the aqeous humor. The types of KPs are: Mutton fat KPs typically occur in granulomatous iridocyclitis and are composed od epithelioid cells and macrophages. They are large, thick, fluffy ald lardaceous, having a greasy or waxy appearance and are usually few (10 to 15) in number. Small and medium KPs pathognomonic of non-granulomatous uveitis and are composed of lymphocytes. These small, discrete, dirty-white KPs are arranged irregularly at the back of the cornea. Small KPs may be hundreds in number and form the so-called endothelial dusting. Red KPs are formed when in addition to inflammatory cells, RBCs also take part in the composition. They may be seen in haemorrhagic uveitis. Old KPs are a signof healed uveitis. Either of the above types of KPs with healing process, skrink, fade, become pigmented and irregular in shape (with crenate margins). Old mutton fat KPs usually have a ground glass appearance due to hyalinization. c) Posterior corneal opacities. Anterior chamber signs: a) Aqueous cells: It is an early feature of iridocyclitis. The cells should be counted in an oblique slitlamp beam, 3-mm long and 1-mm wide, with maximal light intensity and magnification. They are graded as: +1 +2 +3 +4 b) 0 cells 1 5 cells 6 10 cells 11 20 cells 21 50 cells Over 50 cells Aqueous flare: It is due to leakage of protein particles into the aqueous humor from damaged blood vessels. It is demonstrated on the slit-lamp examination by a point beam of light passed obliquely to

4.

25853434, 42433051, 9873314110, 9953550295, 8447461114

75

DELHI ACADEMY OF MEDICAL SCIENCES PVT. LTD.

MAHARASTRA PAPER - 2011

the plane of the iris. In the beam of light, protein particles are seen as suspended and moving dust particles. This is based on the Brownian movements or Tyndall phenomenon. Aqueous flare is usually marked in non-granulomatous and minimal in granulomatous uveitis. The flare is graded as: +1 +2 +3 +4 No flare Just detectable flare Moderate flare with clear iris details Marked flare with iris details not clear Intense flare with fixed coagulated aqueous and considerable fibrin c) d) e) f) 5. Hypopyon: When exudates are heavy and thick, they settle down in the lower part of the anterior chamber as hypopyon (sterile pus in the anterior chamber). Hyphaema (blood pus in the anterior chamber): It may be seen in haemorrhagic uveitis. Changes in depth and shape of anterior chamver due to synecheae formation Changes in the angle of the anterior chamber.

Iris signs: a) Loss of normal iris pattern b) Iris colour becomes muddy during active phase and may show hyperpigmented and depigmented areas in healed stage. c) Iris nodules: They occur typically in granulomatous uveitis. Koeppes nodules: are situated at the pupillary border and may initiate posterior synecheae Busaccas nodules: are situated near the colarette. They are large but less common than Koeppes nodules. d) Posterior synecheae e) Neovascularisation of iris (Rubeosis iridis) Pupillary signs: a) Narrow pupil b) Irregular shape of the pupil. Dilation of pupil at this stage results in festooned pupil due to posterior synecheae formation. c) Ectropion pupillae: Eversion of pupillary margin. d) Pupillary reaction becomes sluggish or may even be absent. e) Occlusio pupillae: the pupil is completely occluded due to organization of the exudates across the entire pupillary area. Lens changes: a) Pigment dispersal on the anterios capsule of lens is almost of universal occurrence due to the formation of posterior synecheae b) Exudates may be deposited on the lens c) Complicated cataract may develop due to persistent iridocyclitis. Vitreous changes: Exudates and inflammatory cells in anterior vitreous. To prevent amblyopia, operation for congenital cataract should be carried out before what age? a) 6 weeks b) 6 months c) 1 year d) 2 years a) 6 weeks Parsons Diseases of the Eye; 20/e, pg 74 It is not advisable to operate on lamellar cataracts until the child is one or preferably two years old; but when the lens is completely opaque, or the pupil will not dilate, and when squint or nystagmus is developing, it is wise to operate at an early age. Moreover, the results of surgery in unilateral cataract in children are universally poor, unless the operation is carried out as early as possible within the first 6 weeks of birth and is followed immediately by the fitting of a contact lens. The critical period for developing the fixation reflex in both unilateral and bilateral visual deprivation disorders is between 2 and 4 months of age. Any cataract dense enough to impair vision must be dealt with before this age and the earliest possible time is preferred. Rosette cataract refers to which of the following types of cataract?

6.

7.

8.

172.

Ans: Ref:

173.

25853434, 42433051, 9873314110, 9953550295, 8447461114

76

DELHI ACADEMY OF MEDICAL SCIENCES PVT. LTD.


a) b) c) d) Ans: Ref: Traumatic cataract Diabetic cataract Galactosemic cataract Steroid-induced cataract

MAHARASTRA PAPER - 2011

a) Traumatic cataract A. K. Khuranas Comprehensive Ophthalmology; 4/e, pg 405 Concussion cataract: It occurs mainly due to imbibition of aqueous and partly due to direct mechanical effects of the injury on lens

fibres. 1. 2. 3. 4. 5. 6. It may assume any of the following types: Discrete subepithelial opacities: is of most common occurrence. Early rosette cataract (punctate): It is the most typical form of concussion cataract. It appears as feathery lines of opacities along the star-shaped suture lines; usually in the posterior cortex. Late rosette cataract: It develops in the posterior cortex 1-2 years after the injury. Its sutural extensions are shorter and more compact than early rosette cataract. Traumatic zonular cataract. Diffuse (total) concussion cataract. Early maturation of senile cataract.

Various types of cataract: Type of Cataract Snow flake cataract Snow storm cataract Oil drop cataract Dust-like lenticular opacity Sunflower cataract Rosette cataract Christmas tree cataract Morgagnian cataract Polychromatic luster Posterior cortical bread crumb apperanace Rainbow cataract

Seen in Diabetes mellitus Galactosemia Wilsons disease Chalcosis Penetrating trauma Blunt trauma Myotonic dystrophy Hypermature senile cataract Complicated cataract

174.

Retinal changes due to vascular proliferation resulting in a sea-fan pattern is seen in a) b) c) Retinal detachment d) Sickle cell disease d) Sickle cell disease Parsons Diseases of the Eye; 20/e, pg 301 Fundus picture in sickle cell retinopathy: The fundus shows both proliferative and non-proliferative changes. The former begins with occlusion of the peripheral arterioles leading to neovascularization and vitreous haemorrhage. Fibrovascular proliferation creates formations like sea fans, particularly in the superotemporal periphery of the retina. Leakage of serum into the vitreous cortex, which occurs near the vascular lesions, causes vitreous organization which may, in turn, lead to traction. Retinal detachment due to vitreous traction is a late complication. Latent squint is also known as a) Heterotropia b) Heterophoria c) Microtropia d) Polyopia b) Heterophoria A. K. Khuranas Comprehensive Ophthalmology; 4/e, pg 321

Ans: Ref:

175.

Ans: Ref:

25853434, 42433051, 9873314110, 9953550295, 8447461114

77

DELHI ACADEMY OF MEDICAL SCIENCES PVT. LTD.

MAHARASTRA PAPER - 2011

Heterophoria: Heterophoria is also known as latent strabismus, a condition in which the tendency of the eyes to deviate is kept latent by fusion. Therefore, when the fusion is removed, the visual axis of one eye deviates away. Practically, a small amount of heterophoria is of universal occurrence and is known as physiological heterophoria. Types of heterophoria: Esophoria: It is a tendency of the eyes to converge. Exophoria: It is a tendency of the eyes to diverge. Hyperphoria: It is a tendency of the eyes to deviate upwards Hypophoria: It is a tendency of the eyes to deviate downwards. Incyclophoria: It is a tendency of the eyes to rotate clockwise nasally Excyclophoria: It is a tendency of the eyes to rotate clockwise temporally.

1. 2. 3. 4. 5. 6. ENT 176.

Lucs abscess following otitis media is seen in which of the following locations? a) beneath the mastoid process b) in the zygomatic region c) in the external auditory canal d) beneath the sternocleidomastoid muscle c) in the external auditory canal K. B. Bhargavas A Short Textbook of E.N.T. diseases; 7/e, pg 63-64 Mastoid abscesses: Subperiosteal abscess: The swelling is over the mastoid antrum. Bezolds abscess: It occurs deep to the sternomastoid muscle. Zygomatic abscess: The abscess tracks along the zygomatic process, producing a swelling above and in front of the external ear. Lucs abscess: The abscess may track down the roof of the bony canal to reach a subtemporal position. Citellis abscess: The abscess spreads to the digastrics triangle along the posterior belly of the digastric muscle. P. L. Dhingras Diseases of Ear. Nose and Throat; 3/e, pg 102

Ans: Ref: 1. 2. 3. 4. 5. Ref:

Meatal abscess (Lucs abscess): In this case, pus breakes through the bony wall between the antrum and external osseous meatus. Swelling is seen in the deep part of the bony meatus. Abscess may burst into the meatus. 177. In noise-induced hearing loss, the frequency of sound that causes maximum damage is a) 1000 2000 Hz b) 2000 3000 Hz c) 4000 5000 Hz d) 6000 7000 Hz b) 2000 3000 Hz P. L. Dhingras Diseases of Ear. Nose and Throat; 3/e, pg 46 The damage caused by noise trauma depends on several factors: Frequency of noise: A frequency of 2000 to 3000 Hz causes more damage than lower or higher frequencies. Intensity and duration of noise: As the intensity increases, permissible time for exposure is reduced. Continuous vs interrupted noise: Continuous noise is more harmful. Susceptibility of the individual. Pre-existing ear disease.

Ans: Ref: 1. 2. 3. 4. 5. Notes:

The audiogram in NIHL shows a typical notch at 4KHz, both for air and bone conduction. It is usually symmetrical on both sides. A noise of 90 dB, 8 hours a day for 5 days a week is the maximum safe limit as recommended by the Ministry of Labour, Government of India. The most common site of mucocele is a) Frontal sinus

178.

25853434, 42433051, 9873314110, 9953550295, 8447461114

78

DELHI ACADEMY OF MEDICAL SCIENCES PVT. LTD.


b) c) d) Ans: Ref: Ethmoidal sinus Maxillary sinus Sphenoidal sinus

MAHARASTRA PAPER - 2011

a) Frontal sinus P. L. Dhingras Diseases of Ear. Nose and Throat; 3/e, pg 246

The sinuses affected by mucocele in the order of frequency are the frontal, ethmoidal, maxillary and sphenoidal. There are two views in the genesis of a mucocele: 1. Chronic obstruction to sinus ostium, resulting in accumulation of secretions which slowly expand the sinus and destroy its bony walls. 2. Cystic dilation of mucous gland of the sinus mucosa due to obstruction of its duct. In this case, the wall of the mucocele is surrounded by bormal sinus mucosa. The contents of the mucocele are sterile. Ref: 179. Mucocele of the frontal sinus: P. L. Dhingras Diseases of Ear. Nose and Throat; 3/e, pg 246-247 It usually presents in the superomedial quadrant of the orbit (90%). It displaces the eyeball forwards, downwards and laterally. The swelling is cystic and non-tender. Egg-shell crackling may be elicited. Sometimes, it presents as a cystic swelling in the forehead (10%). The complaints are usually mild and may include headache, diplopia and proptosis. Radiographs of the frontal sinus usually reveal clouding of the sinus with loss of scalloped outline which is so typical of the normal frontal sinus. Treatment is frontoethmoidectomy with free drainage of the frontal sinus into the middle meatus. Which of these conditions is premalignant? a) Atrophic rhinitis b) Rhinitis sicca c) Rhinitis caseosa d) Rhinoscleroma c) Rhinitis caseosa K. B. Bhargavas A Short Textbook of E.N.T. diseases; 7/e, pg 159 180. Rhinitis caseosa is a rare condition in which caseous material collects in the nose and the maxillary sinus as a result of inspissations of secretions. There is inflammation and formation of granulations. The caseous material and granulations are removed. Histopathological examination of granulations is performed to rule out malignancy. All are true about acute retropharyngeal abscess except a) Dysphagia is present b) It presents with a lateral swelling on the posterior pharyngeal wall c) Torticollis is not uncommon d) It is associated with caries of cervical spine d) It is associated with caries of cervical spine P. L. Dhingras Diseases of Ear. Nose and Throat; 3/e, pg 320-321 Acute retropharyngeal abscess: It is commonly seen in children below 3 years of age. It is the result of suppuration of retropharyngeal lymph nodes secondary to infection in the adenoids, nasopharynx, posterior nasal sinuses or nasal cavity. In adults, it may result from penetrating injury of posterior pharyngeal wall or cervical esophagus. Rarely, pus from acute mastoiditis tracks along the undersurface of the petrous bone to present as a retropharyngeal abscess. Clinical features: Dysphagia and difficulty in breathing are the prominent symptoms as the abscess obstructs the air and food passages. Stridor and croupy cough may be present,

Ans: Ref:

Ans: Ref:

25853434, 42433051, 9873314110, 9953550295, 8447461114

79

DELHI ACADEMY OF MEDICAL SCIENCES PVT. LTD.


MAHARASTRA PAPER - 2011

Torticollis. The neck becomes stiff and the head is kept extended. [Therefore, torticollis is not uncommon, i.e. torticollis is common] Bulge in the posterior pharyngeal wall. Usually seen on one side of the midline. Radiograph of soft tissue lateral view shows: widening of the prevertebral shadow and possibly even the presence of gas. Treatment: Incision and drainage of the abscess, usually done without anaesthesia as there is a risk of rupture of the abscess during intubation. Systemic antibiotics. Tracheostomy, if a large abscess causes mechanical obstruction to the airway or leads to laryngeal edema. P. L. Dhingras Diseases of Ear. Nose and Throat; 3/e, pg 321 Chronic retropharyngeal abscess is tubercular in nature. It is the result of: Caries of the cervical spine. Tuberculous infection of the retropharyngeal lymph nodes, secondary to tuberculosis of the deep cervical nodes.

Ref:

MEDICINE 181. According to the definition of FUO, the criteria for neutropenic FUO is that the neutrophil count should be less than or expected to fall in 1 2 days to a) 1000/mm3 b) 500/mm3 c) 350/mm3 d) 250/mm3 b) 500/mm3 Harrisons Principles of Internal Medicine; 18/e, pg 158-159 FUO was defined by Petersdorf and Beesen in 1961 as Temperatures of > 38.3OC (> 101OF) on several occasions Duration of fever > 3 weeks Failure to reach a diagnosis in spite of 1 week of inpatient investigation New classification given by Durack and Street Classic FUO: corresponds closely to earlier definition of FUO, the only difference being a failure to reach a diagnosis in spite of 3 outpatient visits or 3 days in the hospital or 1 week of intelligent and invasive ambulatory investigation. Nosocomial FUO: FUO in a hospitalized patient who is receiving acute care and in whom the infection was not manifest or incubating on admission Neutropenic FUO: The neutrophil count is < 500/ L or is expected to fall to that level in 1 2 days. HIV-associated FUO: FUO in patients with HIV infection. The secondary deficiency of niacin (pellagra) is seen in all of the following except a) Treatment with isoniazid b) Deficiency of pyridoxine c) Deficiency of riboflavin d) None of these d) None of these Harrisons Principles of Internal Medicine; 18/e, pg 598 Niacin deficiency causes pellagra, which is found mostly among people eating corn-based diets in parts of China, Africa and India. Pellagra in North America is found mainly among alcoholics; in patients with congenital defects of iintestinal and kidney absorption of tryptophan (Hartnup disease) and in patients with carcinoid syndrome, in which there is increased conversion of tryptophan to serotonin. In the setting of famine or population displacement, the occurrence of pellagra results from the absolute lack of niacin but also from the deifiency of micronutrients required for the conversion of tryptophan to niacin [e.g: iron, riboflavin, (option c) pyridoxine (option b)]. Clinical features: Loss of appetite Generalized weakness Irritability

Ans: Ref: 1. 2. 3. 1. 2. 3. 4. 182.

Ans: Ref:

25853434, 42433051, 9873314110, 9953550295, 8447461114

80

DELHI ACADEMY OF MEDICAL SCIENCES PVT. LTD.


MAHARASTRA PAPER - 2011

Abdominal pain Vomiting Bright red glossitis Characteristic skin rash that is pigmented and scaling, particularly in areas exposed to sunlight; known as Casals necklace as it forms a ring around the neck Diarrhea (in part due to proctitis and in part due to malabsorption) Vagnitis Esophagitis Depression Seizures Dementia

[Remember the 3Ds: Dermatitis, Diarrhea and Dementia] Treatment: Oral supplementation of niacin 100-200 mg TDS for 5 days.

[Note: Option a: Treatment with isoniazid is correct because it leads to deficiency of pyridoxine; which may indirectly contribute to pellagra] 183. The earliest manifestation in a patient on iron therapy is a) Decrease in irritability b) Increased red cell count c) Reticulocytosis d) Increased haemoglobin c) Reticulocytosis Wintrobes Clinical Haematology; 12/e, pg 828-829 Response to iron therapy: When specific iron therapy is given, patients often show rapid subjective improvement, with disappearance or marked diminution of fatigue, lassitude, and other nonspecific symptoms. This response may occur before any improvement in anemia is observed. Similarly, pica has been reported to be relieved within 1 week of therapy. The earliest hematologic evidence of response to treatment is an increase in the percentage of reticulocytes and their hemoglobin content. The reticulocytes attain a maximal value on the fifth to tenth day after institution of therapy and thereafter gradually return to normal. The maximal value usually ranges from 5 to 10% and is inversely related to the level of hemoglobin. The reticulocyte response may not be detectable in mild iron deficiency anemia. The blood hemoglobin level is the most accurate measure of the degree of anemia in iron deficiency. During the response to therapy, the red cell count may increase temporarily to values above normal, but the hemoglobin value lags behind. The red cell indices may remain abnormal for some time after the normal hemoglobin level is restored. As recovery occurs, a normocytic cell population gradually replaces the microcytic cell population; and one of the early signs of response to therapy is an increase in the RDW from pretreatment levels. When treatment is fully effective, hemoglobin reaches normal levels by 2 months after therapy is initiated, regardless of starting values. Of the epithelial lesions in iron deficiency, those affecting the tongue and nails are the most responsive to treatment. After 1 to 2 weeks, small regenerating filiform papillae are observed. After 3 months, the tongue has usually returned to normal; however, in patients with severe anemia, some atrophy may persist. Koilonychia usually disappears in 3 to 6 months, with the concavity moving toward the end of the nail as the nail grows. Gastritis and the associated defects in gastric secretion often do not respond to therapy, especially in older adults. In patients younger than 30 years of age, gastric acid secretion and normal epithelial architecture may be restored. Dysphagia may be relieved by iron therapy if the associated postcricoid webs are small or medium sized. The webs themselves are not altered, however, and relapse of the anemia is associated with recurring dysphagia. With more severe lesions, dilatation of the esophagus is required for relief.

Ans: Ref:

25853434, 42433051, 9873314110, 9953550295, 8447461114

81

DELHI ACADEMY OF MEDICAL SCIENCES PVT. LTD.

MAHARASTRA PAPER - 2011

Relapse occurs in a significant fraction of patients who respond to iron treatment, in part because of failure to complete the full course of therapy and in part because of recurrence (or continuation) of the predisposing condition or illness Heinz bodies in RBCs are classically seen in a) G6PD deficiency b) Drug-induced haemolysis c) Sickle cell disease d) Unstable haemoglobins a) G6PD deficiency Harrisons Principles of Internal Medicine; 18/e, pg 879

184.

Ans: Ref:

The most typical feature of G6PD deficiency is the presence of bizarre poikilocytes with red cells that appear to have unevenly distributed haemoglobin (hemighosts) and red cells that appear to have had parts of them bitten away (bite cells or blister cells). A classic test, now rarely carried out, is supravital staining with methyl violet, which, if done promptly, reveals the presence of Heinz bodies, consisting of precipitates of denatured haemoglobin and regarded as a signature of oxidative damage to red cells (except for the rare occurrence of an unstable haemoglobin). (option d) Wintrobes Clinical Haematology; 12/e, pg 8

Ref:

Bite cells or degmacytes are formed due to pitting of Heinz bodies by the spleen. They are associated with glucose-6-phosphate dehydrogenase deficiency and drug-induced oxidant haemolysis. (option b) 185. The most common inherited bleeding disorder is a) Glanzmanns thrombasthenia b) Haemophilia c) von Willebrand disease d) Idiopathic thrombocytopenic purpura c) von Willebrand disease Harrisons Principles of Internal Medicine; 18/e, pg 971 von Willebrand disease is the most common inherited bleeding disorder. Estimates from laboratory data suggest a prevalence of approximately 1%, but data based on symptomatic individuals suggest that it is closer to 0.1% of the population Which of these disorders has an acquired intrinsic defect in the RBC cell membrane? a) Paroxysmal cold haemoglobinuria b) Autoimmune haemolytic anaemia c) Paroxysmal nocturnal haemoglobinuria d) Thrombotic thrombocytopenic purpura c) Paroxysmal nocturnal haemoglobinuria Harrisons Principles of Internal Medicine; 18/e, pg 883-884 Clinical PNH is an acquired chronic haemolytic anaemia characterized by persistent intravascular haemolysis subject to recurrent exacerbations. The haemolysis in PNH is due to an intrinsic abnormality of the red cell, which makes it exquisitely sensitive to activated complement, whether through the alternate pathway or antigen-antibody reaction. The former mechanism is mainly responsible for intravascular haemolysis in PNH. The latter mechanism explains why the haemolysis can be drastically exacerbated in the course of a viral or bacterial infection. There is a deficiency of several protective membrane proteins due to a shortage of a unique glycolipid molecule, GPI, which anchors these proteins to the membrane surface. The deficiency of the protective proteins leads to hypersusceptibility to complement-mediated lysis. features: Anaemia Pancytopenia Thrombosis of veins, which may lead to Budd-Chiari syndrome, due to involvement of the hepatic veins. Passage of blood in the urine, noticed in the morning.

Ans: Ref:

186.

Ans: Ref:

Investigations: Ham test.

25853434, 42433051, 9873314110, 9953550295, 8447461114

82

DELHI ACADEMY OF MEDICAL SCIENCES PVT. LTD.


Sucrose haemolysis test. Flow cytometry (gold standard today)

MAHARASTRA PAPER - 2011

Treatment: Allogenic bone marrow transplant Eculizumab (anti CD-5) 187. Which of these tumours spreads in an orderly sequence, hence staging for which is important in determination of the line of management? a) Hodgkins lymphoma b) Multiple myeloma c) B-cell non-Hodgkins lymphoma d) T- cell non-Hodgkins lymphoma a) Hodgkins lymphoma Robbins & Cotrans Pathologic Basis of Disease; 8/e, pg 600

Ans: Ref:

Hodgkins lymphoma spreads in an orderly fashion. In contrast, most forms of NHL spread widely early in their course in a less predictable fashion. Hence, while lymphoma staging provides generally useful prognostic information, it is of most utility in guiding therapy in Hodgkins lymphoma. 188. The drug of choice for treatment of legionella infection is a) Ampicillin b) Ceftriaxone c) Azithromycin d) Doxycycline c) Azithromycin K. D. Tripathis Essentials of Medical Pharmacology; 6/e, pg 189

Ans: Ref:

Because of higher efficacy, better gastric tolerance and convenient once a daily dosing, azithromycin is now preferred over erythromycin as first choice drug for infections such as Legionnaires pneumonia Chlamydia trachomatis (non-specific urthritis) in both men and women Donovanosis Chancroid. 189. The drug of choice for cervicofacial actinomycosis is a) Liposomal amphotericin-B b) Itraconazole c) Ketoconazole d) Penicillin d) Penicillin Harrisons Principles of Internal Medicine; 18/e, pg 1329-1330 Actinomycosis requires prolonged treatment with high doses of antimicrobial agents. The need for intensive treatment is presumably due to the drugs poor penetrance of the thick-walled masses commonly in this infection and/or the sulphur granules themselves. Intravenous administration of 18-24 MU of penicillin daily for 2-6 weeks, followed by oral therapy with penicillin or amoxicillin (total duration of 6-12 months) is a reasonable guideline for serious infections and bulky disease. Less extensive disease, particularly that involving the oral and cervico-facial region, may be cured with a shorter course.

Ans: Ref:

Extensive successful clinical experience has been obtained with Penicillin Erythromycin Tetracycline Doxycycline Minocycline and Clindamycin Anecdotal successful clinical experience has been obtained with

25853434, 42433051, 9873314110, 9953550295, 8447461114

83

DELHI ACADEMY OF MEDICAL SCIENCES PVT. LTD.


Ceftriaxone Ceftizoxime Imipenem and Piperacillin/Tazobactam

MAHARASTRA PAPER - 2011

Agents predicted to be efficacious on the basis of vitro activity Moxifloxacin Vancomycin Linezolid Quinupristin/Dalfopristin Agents that should be avoided: Metronidazole Aminoglycosides Oxacillin Dicloxacillin and Cephalexin 190. Which of these ECG findings indicates a diagnosis of left ventricular hypertrophy? a) RaVL + SV3 > 28 cm b) Left atrial hypertrophy c) RV1 + SV5/SV6 > 35 cm d) All of these b) Left atrial hypertrophy Harrisons Principles of Internal Medicine; 18/e, pg 1835 190b. A number of different voltage critera for left ventricular hypertrophy have been proposed on the basis of the presence of tall left precordial R waves and deep right precordial S waves. [e.g. SV1 + (RV5 or RV6) > 35 mm]. Repolarization abnormalities (ST depression with T-wave inversions, formerly called the left ventricular strain pattern) may also appear in leads with prominent R waves. However, prominent precordial voltages may occur as a normal variant, especially in athletic or young individuals. Left ventricular hypertrophy may increase limb lead voltage with or without increased precordial voltage (e.g. RaVL + SV3 > 20 mm in women and > 28 mm in men). The presence of left atrial abnormality increases the likelihood of underlying left ventricular hypertrophy in cases with borderline voltage criteria. ECG evidence for left ventricular hypertrophy is a major non-invasive marker of increased risk of cardiovascular mortality and morbidity, including sudden cardiac death. However, the sensitivity of conventional voltage criteria for left ventricular hypertrophy is decreased in obese persons and in smokers. Which of these ECG findings indicates a diagnosis of left ventricular hypertrophy? a) RaVL + SV3 > 28 mm b) Left atrial hypertrophy c) RV1 + SV5/SV6 > 35 mm d) All of these d) All of these Harrisons Principles of Internal Medicine; 18/e, pg 1835

Ans: Ref:

Ans: Ref:

We are not sure whether the options (a) and (c) mentioned the voltage criteria in cm or mm. If they are in mm, then the answer would be (d) All of these,. Please refer Q. 190a for detailed explanation. 191. Prominent Y-descent on JVP is seen in all of the following conditions except a) Pericardial effusion with tamponade b) Right ventricular failure c) Restrictive cardiomyopathy d) Constrictive pericarditis a) Pericardial effusion with tamponade Harrisons Principles of Internal Medicine; 18/e, pg 1975 Cardiac tamponade Constrictive pericarditis Restrictive cardio-myopathy Right MI ventricular

Ans: Ref:

Characteristic

25853434, 42433051, 9873314110, 9953550295, 8447461114

84

DELHI ACADEMY OF MEDICAL SCIENCES PVT. LTD.


Clinical Pulsus paradoxus Prominent y descent Prominent x descent Kussmauls sign 3rd heart sound Pericardial knock ECG Low ECG voltage Electrical alternans 2-D echo Thickened pericardium Pericardial calcification Pericardial effusion RV small Myocardial thickness Right atrial collapse and RVDC Increased early filling Increased mitral flow velocity Exaggerated respiratory variation in flow velocity CT/MRI Thickened/calcific pericardium Cardiac catheterization Equalization of diastolic pressures Cardiac biopsy helpful?

MAHARASTRA PAPER - 2011

Common Absent Present Absent Absent Absent May be present May be present Absent Absent Present Usually small Normal Present Absent Absent Present

Usually absent Usually present Usually present Present Absent Often present May be present Absent Present Often present Absent Usually normal Normal Absent Present Present Present

Rare Rare Rare Absent Rare Absent May be present Absent Absent Absent Absent Usually normal Usually increased Absent Present Present Absent

Rare Rare Rare Present May be present Absent Rare Absent Absent Absent Absent Enlarged Normal Absent May be present May be present Absent

Absent Usually present No

Present Usually present No

Absent Usually absent Sometimes

Absent Absent or present No

192.

Which of these drugs can be given as a single IV bolus dose (over 10 min) for thrombolytic action? a) tPA (tissue plasminogen activator) b) Tenecteplase c) rtPA (Alteplase) d) Any of the above K. D. Tripathis Essentials of Medical Pharmacology; 6/e, pg 606 Tenecteplase: It is a mutant variety of rt-PA with higher fibrin selectivity and longer duration of action. A single IV bolus dose (0.5 mg/kg) or split into two doses 30 min apart is given. Reteplase: K. D. Tripathis Essentials of Medical Pharmacology; 6/e, pg 606 It is a modified form of rt-PA that is longer acting, but somewhat less specific for fibrin-bound plasminogen. The longer duration of action enables bolus dose administration (10 mg over 10 min repeated after 30 min). rtPA (Alteplase) K. D. Tripathis Essentials of Medical Pharmacology; 6/e, pg 606 Alteplase is produced by recombinant DNA technology from humn tissue cultures. It specifically activates gel phase plasminogen already bound to fibrin and has little action on circulating plasminogen. It is rapidl cleared by the liver and has a plasma T of 4-8 min. Because of short T, it needs to be given b slow IV infusion and often requires heparin co-administration. It is non-antigenic; but nausea, mild hypotension and fever may occur. It is expensive.

Ref:

Ref:

Ref:

As the options provided for the question are precisely not known, let us consider various versions of the question and the most probable correct answer in each case: 192a. Which of these drugs can be given as a single IV bolus dose (over 10 min) for thrombolytic action? a) tPA (tissue plasminogen activator) b) Tenecteplase c) rtPA (Alteplase) d) Any of the above

25853434, 42433051, 9873314110, 9953550295, 8447461114

85

DELHI ACADEMY OF MEDICAL SCIENCES PVT. LTD.


Ans: 192b. b) Tenecteplase

MAHARASTRA PAPER - 2011

Which of these drugs can be given as a single IV bolus dose (over 10 min) for thrombolytic action? a) Reteplase b) tPA (tissue plasminogen activator) c) rtPA (Alteplase) d) Any of the above a) Reteplase

Ans: 192c.

Which of these drugs can be given as a single IV bolus dose (over 10 min) for thrombolytic action? a) Reteplase b) Tenecteplase c) rtPA (Alteplase) d) Any of the above b) Reteplase

Ans: 193.

All of the following regarding cholorrhic enteropathy are true except a) Serum bile acid pool is normal b) Feacal fat loss is minimal or nil c) Response to low fat diet is good d) Response to cholestyramine is good c) Response to low fat diet is good Harrisons Principles of Internal Medicine; 18/e, pg 2462-2463 Ileal dysfunction caused by either Crohns disease or surgical resection results in a decrease in bile acid reabsorption in the ileum and an increase in the delivery of the bile acids to the large intestine. The resulting clinical consequences diarrhea with or without steatorrhea are determined by the degree of ileal dysfunction and the response of the enterohepatic circulation to bile acid losses. The diarrhea, a result of bile acids in the colon stimulating active Cl secretion, has been called bile acid diarrhea or cholorrheic enteropathy, and responds promptly to cholestyramine, an anion-binding resin. (option d) Such patients do not develop steatorrhea (option b) because hepatic synthesis of bile acids increases to compensate for the rate of fecal bile acid losses, resulting in maintenance of both the bile acid pool size (option a) and the intraduodenal concentrations of bile acids. In contrast, patients who have greater degrees of ileal disease and/or resection will often have diarrhea and steatorrhea that do not respond to cholestyramine. In this situation, ileal disease is also associated with increased amounts of bile acids entering the colon; however hepatic synthesis can no longer sufficiently maintain the bile acid pool size. As a consequence, the intraduodenal concentration of bile acids is also reduced to less than the crtitical micellar concentration, resulting in impaired micelle formation and steatorrhea. This second situation is often called fatty acid diarrhea. Cholestyramine may not be effective, and may even increase the diarrhea by further depleting the intraduodenal bile acid concentration; however, a low fat diet to reduce fatty acids entering the colon can be effective. Two clinical features, the length of ileum removed and the degree of steatorrhea, can predict whether an individual patient will respond to cholestyramine. Bile acid diarrhea Limited Reduced Increased Yes Normal Normal None or mild Yes No Fatty acid diarrhea Extensive Reduced Increased No Reduced Reduced < 20g No Yes

Ans: Ref:

Extent of ileal disease Ileal bile acid absorption Fecal bile acid excretion Fecal bile acid loss compensated synthesis Bile acid pool size Intraduodenal bile acid Steatorrhoea Response to cholestyramine Response to low fat diet

by

hepatic

194.

Toxic megacolon in a case of ulcerative colitis is said to occur if the diameter of the colon exceeds a) 4 cm

25853434, 42433051, 9873314110, 9953550295, 8447461114

86

DELHI ACADEMY OF MEDICAL SCIENCES PVT. LTD.


b) c) d) Ans: Ref: 6 cm 9 cm 7 cm

MAHARASTRA PAPER - 2011

b) 6 cm Harrisons Principles of Internal Medicine; 17/e, Pg 2656 Toxic megacolon is defined as a transverse colon with a diameter of > 5-6 cm, with loss of haustrations in patients with severe attacks of ulcerative colitis. It occurs in about 5% of attacks and can be triggered by electrolyte abnormalities and narcotics. About 50% of acute dilations will resolve with medical therapy alone, but urgent colectomy is required for those that do not improve. Perforation is the most dangerous of the local complications, and the physical signs of peritonitis may not be obvious, especially if the patient is receiving glucocorticoids. The diagnosis of acute hepatitis B infection is established by a) HBsAg + anti-HBc IgG b) HBsAg + anti-HBc IgM c) HBsAg + HBeAg + HBV DNA d) HBsAg + HBeAg / HBV DNA b) HBsAg + anti-HBc IgM Harrisons Principles of Internal Medicine; 18/e, pg 2551 Diagnostic approach in patients presenting with Acute Hepatitis: IgM anti- IgM anti- Anti-HCV Interpretation HAV HBc + Acute hepatitis A + + Acute hepatitis A and B + + Acute hepatitis A and B with HBsAg below the diagnostic threshold + Acute hepatitis A superimposed on chronic hepatitis B + Acute hepatitis B + Acute hepatitis B with HBsAg below the diagnostic threshold Chronic hepatitis B + Hepatitis C

195.

Ans: Ref:

HBsAg + + + + -

196.

A chest X-ray showing a triangular opacity with clear upper border and the base towards the midline with silhouetting of the right heart border indicates a collapse of a) Right middle lobe b) Apical posterior basal segment of right lung c) Apical anterior basal segment of right lung d) Any of the above a) Right middle lobe Graingers Diagnostic Radiology;

Ans: Ref:

Right middle lobe collapse: On chest X-ray: The features of right middle lobe collapse may be extremely subtle on the frontal view and consequently easy to overlook. The collapsed lobe lies adjacent to the right heart border and there is loss of the silhouette of this structure to a variable degree. There may or may not be a recognizable increase in density depending on the orientation of the collapse relative to the X-ray beam. When the collapse is orientated roughly parallel to the beam or if the patient is in a lordotic position, a triangular, sail-shaped density may be seen adjacent to the heart border. However, if the collapsed lobe lies obliquely in the chest, more parallel with the major fissure, the only sign on the frontal radiograph may be indistinctness of a portion of the right atrial border. By comparison, the triangular density of the collapsed right middle lobe is relatively easy to identify on the lateral view, with approximation of the minor and inferior portion of the major fissure, the apex of the triangle being at the hilum. In increasingly severe collapse the triangular shape is less marked as the fissures become almost parallel with only a thin wedge of density separating them. On CT scan:

25853434, 42433051, 9873314110, 9953550295, 8447461114

87

DELHI ACADEMY OF MEDICAL SCIENCES PVT. LTD.


MAHARASTRA PAPER - 2011

The CT appearances are characteristically of a triangular-shaped density of varying size adjacent to the heart border. Depending on the orientation of the collapse, only a small portion may be identified on each section as the collapse represents a relatively flat sheet of tissue. The so-called middle lobe syndrome refers to a collapsed right middle lobe with bronchiectasis due to a focal bronchostenosis secondary to pulmonary tuberculosis. Although in theory any lobe may be affected, the middle lobe is the most common, resulting in characteristic CT features. The type of renal tubular acidosis (RTA) associated with hyperkalemia is a) Type I b) Type I & II c) Type II d) Type IV d) Type IV Harrisons Principles of Internal Medicine; 16/e, Pg 1699-1700

197.

Ans: Ref:

In type 4 RTA, also called hyperkalemic distal RTA, distal tubule secretion of both potassium and hydrogen ions is abnormal, resulting in hyperchloremic acidosis with hyperkalemia. It is an acquired disorder. Such patients are considered to have a generalized distal nephron dysfunction due to either insufficient aldosterone production or intrinsic renal disease causing aldosterone resistance. Comparison of Normal Anion-gap acidosis: Finding Normal anion gap acidosis Minimum urine pH % of filtered HCO3 excreted Serum potassium Fanconi syndrome Stones/nephrocalcinosis Daily acid excretion Urine anion gap Daily HCO3 replacement needs (mmol/kg)

Type I RTA + > 5.5 < 10 Low + Low + <4

Type II RTA + < 5.5 > 15 Low + Normal + >4

Type IV RTA + < 5.5 < 10 High Low + >4

GI HCO3 loss + 5-6 < 10 Low High Variable

198.

Most of the pituitary adenomas are a) Prolactinomas b) GH-secreting adenomas c) ACTH-secreting adenomas d) Non-functional adenomas d) Non-functional adenomas Harrisons Principles of Internal Medicine; 18/e, pg 2880 Pituitary adenomas are the most common cause of pituitary hormone hypersecretion and hyposecretion syndromes in adults. However, about 1/3rd of all adenomas are clinically non-functioning and produce no distinct clinical hypersecretory syndrome. At autopsy, up to a quarter of all pituitary glands harbor an unsuspected microadenoma (< 10 mm in diameter). Similarly, pituitary imaging detects small pituitary lesions in at least 10% of normal individuals. Most of these arise from gonadotrope cells and may secrete small amounts of and glucoprotein hormone subunits.

Ans: Ref:

(Note: Amongst the functional adenomas, the most common type is a prolactinoma] 199. The mad cow disease is caused by which of the following? a) bacteria b) viruses c) fungi d) prions d) prions

Ans:

25853434, 42433051, 9873314110, 9953550295, 8447461114

88

DELHI ACADEMY OF MEDICAL SCIENCES PVT. LTD.


Ref:

MAHARASTRA PAPER - 2011

Robbins and Cotrans The Pathologic Basis of Disease; 7/e, pg 346, 1380. Prions are apparently composed of abnormal forms of a host protein, termed prion protein (PrP). These agents cause transmissible spongiform encephalopathies Kuru (associated with human cannabalism) Creutzfeld-Jacob disease (associated with corneal transplants) Bovine spongiform encephalopathies (better known as mad-cow disease) and Variant CJD (likely to be transmitted to humans from BSE-infected cells) They are predominantly characterized by spongiform change caused by intracellular vacuoles in neurons and

glia. 200.

Most common form of peripheral neuropathy in diabetes is a) Autonomic neuropathy b) Diabetic amyotrophy c) Distal sensorimotor polyneuropathy d) Isolated cranial nerve involvement c) Distal sensorimotor polyneuropathy Harrisons Principles of Internal Medicine; 17/e, Pg 2656 By far the most common form of diabetic neuropathy is a length-dependent diabetic sensorimotor polyneuropathy (DSPN). The lifetime prevalence is ~ 55% for type 1 and 45% for type 2 diabetes. DSPN is a mixed neuropathy with small- and large-fibre sensory, autonomic and motor nerve involvement in various combinations, although sensory and autonomic symptoms are more prominent than motor ones. DSPN has a insidious, progressive course. Initial symptoms may consist of numbness, tingling, buzzing, burning, or pricking sensation affecting the toes and feet. Paresthesias ascend up to the legs and then hands in a stocking-glove distribution. Over time, gait disturbance and distal weakness may occur. Painful or insensitive extremities predispose to foot ulcers, amputation is sometimes required. Examination shows a distal sensory loss to pin, temperature, touch and vibration sense. Ankle reflexes are invariably reduced or absent. Weakness, if present is mild and involves toe flexors and extensors.

Ans: Ref:

Proposed critetia for diagnosis of DSPN are two or more of the following: Symptoms or signs of neuropathy Abnormal electrodiagnostic studies Quantitative sensation test abnormalities Heart rate decrease with deep breathing or Valsalva maneuver. Treatment consists of strict glucose control, which prevents the neuropathy from worsening; established neuropathy does not usually reverse. Option (a): Autonomic neuropathy Ref: Harrisons Principles of Internal Medicine; 17/e, Pg 2656 Autonomic symptoms including impotence, nocturnal diarrhea, difficulty voiding, abnormalities of sweating, abnormal fullness after eating and orthostatic hypotension may be present. Option (b): Diabetic amyotrophy Ref: Harrisons Principles of Internal Medicine; 17/e, Pg 2657 Diabetic amyotrophy (femoral neuropathy or proximal diabetic neuropathy) occurs in older patients, usually with type 2 diabetes. Males are more likely to be affected. Patients present with the abrupt onset of severe pain affecting the anterior thigh. Buttock and lower back pain may also be present. The pain is worse at night and described as burning. Weakness and wasting in the thigh muscles leads to difficulty climbing stairs and walking. Weight loss, at times dramatic, is invariably present.

25853434, 42433051, 9873314110, 9953550295, 8447461114

89

DELHI ACADEMY OF MEDICAL SCIENCES PVT. LTD.

MAHARASTRA PAPER - 2011

Examination shows prominent wasting of the quadriceps muscle umilaterally with weakness of the knee extensor and hip flexor and variably, ankle dorsiflexor; accompanied by sensory loss in the thigh and leg in the distribution of the femoral nerve, and a reduced knee jerk on the affected side. The syndrome progresses over weeks to months, then stabilizes and gradually improves. Treatment with high dose glucocorticoids or intravenous immunoglobulin (IVIg) has been effective in some cases. Physiotherapy and orthotic devices are helpful. The prognosis is generally favourable; improvement occurs over several months.

Option (d): Isolated cranial nerve involvement Ref: Harrisons Principles of Internal Medicine; 17/e, Pg 2657 201. The oculomotor nerves (in the decreasing order of frequency; the sixth, the third and rarely the fourth nerves) are most often affected. In general, cranial neuropathy occurs in patients over the age of 50 who already have evidence of DSPN. B/L facial nerve involvement is seen in a) Sarcoidosis b) Leprosy c) Guillian-Barre syndrome d) All of these d) All of these Current Diagnosis and Treatment in Otorhinolaryngology; 2/e. Conditions associated with B/L facial palsy: Bell palsy Diabetes mellitus Sarcoidosis (Heerfordt syndrome) (option a) Periarteritis nodosa Guillain-Barr syndrome (option c) Myasthenia gravis Basilar skull fracture Bulbar palsies Porphyrias Leukemia Myotonic dystrophia Meningitis Mbius syndrome Botulism Infectious mononucleosis Leprosy (option b) Malaria Poliomyelitis Lyme disease Syphilis Postvaccination neuropathy Isoniazid Osteopetrosis

Ans: Ref:

SURGERY 202. In pre-menopausal females, self breast examination should be done in which period? a) Just before benses b) During the menses c) 8 10 days after menses d) At the time of ovulation b) During the menses www.wikipedia.org. Self-breast examination. For pre-menopausal women, most methods suggest that the self-exam should be performed at the same stage of the womans menstrual cycle, because the natural hormone fluctuations can cause changes in the breasts.

Ans: Ref:

25853434, 42433051, 9873314110, 9953550295, 8447461114

90

DELHI ACADEMY OF MEDICAL SCIENCES PVT. LTD.


MAHARASTRA PAPER - 2011

The most commonly recommended time is just after the end of the period, because the breasts are least likely to be swollen and tender at this time. Women who are post-menopausal or have irregular cycles might do a self-exam once a month regardless of their menstrual cycle. The most common site for lymphangiosarcoma is a) Lower extremity (thigh) b) Mediastinum c) Post-mastectomy irradiated arm d) Retroperitoneum c) Post-mastectomy irradiated arm Robbins and Cotrans The Pathologic Basis of Disease; 7/e, pg 550

203.

Ans: Ref:

Angiosarcomas may arise in the setting of lymphedema, most typically approximately 10 years following radical mastectomy for breast cancer. In such cases, the tumour presumably arises from dilated lymphatic vessels (lymphangiosarcoma)

Note: Tumours classified as hemangiopericytoma most commonly arise in the pelvic retroperitoneum (option d) or the lower extremities (especially the thigh) (option a) of middle aged women. 204. Indications for admission in a case of burns are all except a) Inhalational burns b) Suspicious non-accidental burns c) Any % of burns in a young unmarried female d) Electrical burns c) Any % of burns in a young unmarried female Bailey & Loves Short Practice of Surgery; 24/e, pg 268. Table 22.2: The criteria for acute admissions to a burns unit. Suspected airway or inhalational injury. Any burn likely to require fluid restriction. Any burn likely to require surgery. Patients with burns of any significance to the hands, face, feet or perineum. Patients whose psychiatric or social background makes it inadvisable to send them home. Any suspicion of non-accidental injury, Any burn in a patient at the extremes of age. Any burn with an associated potentially serious sequelae including high-tension electric burns and concentrated hydrofluoric acid burns. The best time to repair an isolated cleft lip is a) 0 3 months b) 5 6 months c) 7 8 months d) 9 12 months b) 5 6 months Bailey & Loves Short Practice of Surgery; 24/e, pg 268

Ans: Ref: 1. 2. 3. 4. 5. 6. 7. 8.

205.

Ans: Ref:

Timing of primary cleft lip and palate procedures (after Delaire) Cleft lip alone U/L One operation at 5 6 months B/L One operation at 4 5 months Cleft palate alone Soft palate only: One operation at 6 months Soft + Hard palate Two operations: Soft palate at 6 months & hard palate at 15 18 months Cleft lip + Palate: U/L Two operations: Cleft lip & soft palate at 5 6 months f/b hard palate and gum pad with/without lip revision at 15 18 months B/L Two operations: Cleft lip & soft palate at 4 5 months f/b hard palate and gum pad with/without lip revision at 15 18 months

206.

All of the following are done in radical parotidectomy except

25853434, 42433051, 9873314110, 9953550295, 8447461114

91

DELHI ACADEMY OF MEDICAL SCIENCES PVT. LTD.


a) b) c) d) Ans: Ref: Removal of the parotid gland Preserving the facial nerve Excision of cervical LN on I/L side Removal of masseter

MAHARASTRA PAPER - 2011

b) Preserving the facial nerve Bailey & Loves Short Practice of Surgery; 24/e, pg 734 Radical parotidectomy Radical parotidectomy is performed for patients when there is clear histological evidence of a high-grade malignant tumour with extensive facial nerve infiltration, for example, squamous cell carcinoma. Low-grade malignant tumours can be usually managed by standard superficial parotidectomy. Radical parotidectomy involves removal of all parotid gland tissue (option a) and elective sectioning of the facial nerve (option b), usually through the main trunk. The surgery inevitably removes the ipsilateral masseter muscle and may also require simultaneous neck dissection (option c), particularly when there is clinical, radiological and cytological evidence of lymph node metastases in ipsilateral neck. Which of the following head and neck cancers has the worst prognosis? a) Verrucous squamous cell carcinoma b) Basaloid squamous cell carcinoma c) Adenoid cystic carcinoma d) Lymphoepithelioma c) Adenoid cystic carcinoma S. Das A Manual on Clinical Surgery; 6/e, pg 275 Adenoid cystic carcinoma (Cylindroma): These are poorly encapsulated infiltrating tumours to which the name cylindroma is commonly applied. Approximately 10% of the malignant tumours of the salivary gland are of this type. Though the tumour arises more frequently in the parotid glands; yet in the submandibular and ectopic salivary glands, this represents a higher proportion of all tumours (20%). The tumour cells are small, darkly stained with relatively little cytoplasm and are arranged about the stromal elements in a pseudoglandular (adenoid) pattern. They display a wide range of patterns either tubular or cribriform or solid. The stroma in most of these tumours is moderately cellular fibrous tissue but is strikingly hyalinized. The tumour is slow growing and may be mistaken as a mixed tumour. But local recurrences and continuous growths involving the surrounding structures will soon reveal itself. Local pain is prominent and sometimes an early symptom. The tendency of this tumour to invade the perineural lymphatics accounts for the high frequency of facial nerve paralysis. Five-year cure rate has been quoted as less than 25%. The mortality seen in cases of # of the 1st rib is a) 5% b) 10 % c) 20 % d) 30 % d) 30 % Bailey and Loves Short Practice of Surgery; 24/e, pg 868 Fracture of the first rib should alert the clinician to a potentially serious chest injury. This rib is well protected and requires a considerable force to fracture, and associated injuries to the great vessels of the abdomen, head and neck are common. The mortality rate associated with a fracture of the first rib exceeds 30%. Similar suspicions are raised when fractures of the sternum and scapula are seen. Intercostal bleeding may be severe, resulting in haemothorax. In a case of trauma to the chest, all of the following are indications of doing a thoracotomy except a) Initial blood loss of 1500 mL b) Cardiac tamponade c) Continued blood loss of 100mL/hr

207.

Ans: Ref:

208.

Ans: Ref:

209.

25853434, 42433051, 9873314110, 9953550295, 8447461114

92

DELHI ACADEMY OF MEDICAL SCIENCES PVT. LTD.


d) Ans: Ref: 1. 2. 3. 4. 5. 210. Persistent air leak

MAHARASTRA PAPER - 2011

d) Persistent air leak Bailey and Loves Short Practice of Surgery; 24/e, pg 868 Box 56.7: Indications for doing a thoracotomy after blunt chest trauma: 1000mL drained at the insertion of chest drain. Continued brisk bleeding of >100 mL per 15 min. Continued bleeding of > 200 mL/hr for 3 hours. Rupture of bronchus, aorta, esophagus or diaphragm. Cardiac tamponade Spigelian hernia is a) b) c) d) A inter-parietal hernia at the level of the arcuate line d) A inter-parietal hernia at the level of the arcuate line Bailey and Loves Short Practice of Surgery; 24/e, pg 1286-1287 Spigelian hernia: It is a variety of inter-parietal hernia occurring at the level of the arcuate line. It is very rare, with only 1000 cases reported in the literature. The fundus of the sac, clothed by the extraperitoneal fat, may lie benath the internal oblique muscle, where it is virtually impalpable. More often, it advances through that muscle and spreads out like a mushroom between the internal and external oblique muscles and gives rise to a more evident swelling. Clinical features: The patient is often corpulent and usually over 50 years of age. Men and women are equally affected. Typically, a soft, reducible mass will be encountered lateral to the rectus muscle and below the umbilicus. Owing to the rigid fascia surrounding the neck, strangulation may occur. Diagnosis: Diagnosis is confirmed by CT or USG. The latter has the advantage that it can be performed in the upright patient if no defect is visible in the reclining position. Treatment: If a defect is palpable, a muscle-splitting approach is used. After isolating the sac, dealing with any contents, and ligating and excising oblique and external oblique muscles are repaired by direct apposition.

Ans: Ref:

it, the transversus, internal

211.

If no sac is palpable, a paramedian approach is used and the sac sought in the extraperitoneal space. The repair then proceeds as above. A transabdominal laparoscopic approach may also be employed. A non-functioning adrenal mass is surgically removed if the size exceeds a) 10 cm b) 7 cm c) 4 cm d) 1 cm c) 4 cm Bailey & Loves Short Practice of Surgery; 24/e, pg 816 Incidentaloma Incidentaloma is an incidentally discovered mass, identified on cross-sectional imaging performed for an unrelated indication. This will be found in 3 5 % of patients undergoing abdominal CT examination; the incidence increases with age and in patients with hypertension. Non-functioning adreno-cortical adenomas are the most common cause of an incidentaloma. A U/L non-functioning adrenal mass greater than 4 cm in diameter is an indication for adrenalectomy.

Ans: Ref:

25853434, 42433051, 9873314110, 9953550295, 8447461114

93

DELHI ACADEMY OF MEDICAL SCIENCES PVT. LTD.


212.

MAHARASTRA PAPER - 2011

The patient with a mass smaller than 4 cm in diameter should undergo repeat CT/MRI 4 6 motnhs after the previous examination. An increase in the size of the mass is an indication for adrenalectomy. The normal lower esophageal sphincter pressure is a) 5 10 mm Hg b) 10 25 mm Hg c) 25 40 mm Hg d) 40 55 mm Hg b) 10 25 mm Hg Bailey & Loves Short Practice of Surgery; 24/e, pg 990 991

Ans: Ref: 213.

The lower esophageal sphincter (LES) prevents gastric and duodenal contents from refluxing into the lower esophagus. The function of the physiological sphincter was first demonstrated by Code using manometry with small balloons. Nowadays, LES pressure is measured by perfused tubes or microtransducers. The normal LES is 3 4 cm long and has a pressure of 10 25 mm Hg. (or 50.5 cm H2O). Cork-screw esophagus is seen in a) Achalasia cardia b) Barrets esophagus c) Diffuse esophageal spasm d) Carcinoma esophagus c) Diffuse esophageal spasm Bailey & Loves Short Practice of Surgery; 24/e, pg 1020. Diffuse esophageal spasm: Diffuse esophageal spasm is a condition in which there are in-coordinate contractions of the esophagus, causing dysphagia and chest pain. The condition may be dramatic, with spastic pressures on manometry of 400 500 mm Hg, marked hypertrophy of the circular muscle and a corkscrew esophagus on barium swallow. Prolonged ambulatory esophageal manometry with a detailed record of the timing of episodes of chest pain may help to make a diagnosis. Dysphagia due to diffuse spasm may respond to forceful dilation of the cardia, but the results are not as predictable as achalasia. In very severe cases, extended esophageal myotomy up to the aortic arch may be required. Surgical treatment of diffuse spasm is more successful in improving dysphagia than chest pain, and caution should be exercised in patients in whom chest pain is the only symptom. String of beads appearance on abdominal X-ray is a feature seen in a) Large bowel obstruction b) Pyloric stenosis c) Intussusception d) Small bowel obstruction d) Small bowel obstruction Graingers Diagnostic Radiology;

Ans: Ref:

214.

Ans: Ref: 215.

Plain radiograph changes may appear after 35 hours if there is complete small bowel obstruction (SBO), and are usually marked after 12 hours. With incomplete obstruction, changes on the plain radiograph may take hours or days to appear, and may even then be nonexistent. Dilated loops of small bowel are readily identified if they are gas filled on the supine radiograph. The string of beads sign, due to a line of gas bubbles trapped between the valvulae conniventes, is seen only when very dilated small bowel is almost completely filled with fluid, and is virtually diagnostic of SBO. Fluid-filled loops are not readily appreciated on plain radiographs, but are more easily seen on CT. Angiodysplasias in the GI tract most commonly occur in a) Sigmoid colon b) Transverse colon c) Caecum and ascending colon

25853434, 42433051, 9873314110, 9953550295, 8447461114

94

DELHI ACADEMY OF MEDICAL SCIENCES PVT. LTD.


d) Ans: Ref: Descending colon

MAHARASTRA PAPER - 2011

c) Caecum and ascending colon Bailey & Loves Short Practice of Surgery; 24/e, pg 1157 Vascular anomalies (Angiodysplasia) Angiodysplasia is a vascular malformation associated with ageing. It has been recognized since the introduction of intestinal angiography and colonoscopy. Angiodysplasias occur particularly in the ascending colon and caecum of elderly patients over the age of 60 years. Occassionally, angiodysplasias are seen in association with cutaneous and oral vascular malformations as a part of hereditary telangiectasia. The manifestation consists of dilated, tortuous submucosal veins, and in severe cases the mucosa is replaced by massive dilated deformed vessels. On histological investigation, angiodysplasias are made up of dilated, distorted, thin-walled vessels with only a scanty amount of muscle in their walls. Inspection of the mucosa is often unremarkable. The lesions are only a few millimeters in size and appear as reddish, raised areas at endoscopy. Bleeding is usually chronic and intermittent, and can be severe. In many patients in whom rectal bleeding has previously been attributed to diverticular disease, bleeding was probably, in fact, from angiodysplasia of the caecum. There is an association with aortic stenosis, although the reasons for this remain obscure. Barium enema is usually unhelpful and should be avoided, not at least because it may mask the lesion on subsequent endoscopy. Provided that the bleeding is not too brisk, colonoscopy may show the characteristic lesion in the caecum or ascending colon. Selective superior and inferior mesenteric angiography shows the site and extent of lesion by a blush. If this fails, a radioactive test using 99mTc-labelled red cells may confirm and localize the site of haemorrhage. Some angiodysplastic lesions can be treated by colonoscopic diathermy, but if bleeding is brisk and the patient seriously ill, emergency surgery will be necessary. Here a catheter is placed in the appendix stump and the colon irrigated progradely with saline or water. On-table colonoscopy is carried out and the site of the bleeding can then be confirmed. Angiodysplastic lesions are sometimes demonstrated by transillumination through the caecum. If it is still not clear exactly which segment of the colon is involved, then a total abdominal colectomy with ileorectal anastomosis may be necessary. Which of the following gastro-intestinal polyps has the highest risk of progression to malignancy? a) Solitary hyperplastic polyp b) Juvenile polyps c) Adenomatous polyps d) Peutz-Jeghers polyps c) Adenomatous polyps Robbins and Cotrans Pathologic Basis of Disease; 8/e, pg 819 The most common and clinically important neoplastic polyps are colonic adenomas, benign polyps that are the precursors to the majority of colorectal adenocarcinomas. The adenomas are intra-epithelial neoplasms that range from small, often pedunculated polyps to large, sessile lesions. These polyps are precursors to colorectal cancer and it is recommended that all adults undergo surveillance colonoscopy by age 50.

216.

Ans: Ref:

Option (a): Solitary hyperplastic polyp. Ref: Robbins and Cotrans Pathologic Basis of Disease; 8/e, pg 818-819 The pathogenesis of hyperplastic polyps is incompletely understood, but they are thought to result from decreased epithelial cell turnover and delayed shedding of surface epithelial cells, leading to a piling up of goblet cells and absorptive cells. It is now appreciated that these lesions are without malignant potential. Their chief significance is that they must be distinguished from sessile, serrated adenomas, histologically similar lesions that have malignant potential.

Option (b): Juvenile polyps. Ref: Robbins and Cotrans Pathologic Basis of Disease; 8/e, pg 816-817

25853434, 42433051, 9873314110, 9953550295, 8447461114

95

DELHI ACADEMY OF MEDICAL SCIENCES PVT. LTD.


MAHARASTRA PAPER - 2011

Juvenile polyps are focal malformations of the mucosal epithelium and lamina propria. The vast majority of juvenile polyps occur in children less than 5 years of age. The majority of juvenile polyps are located in the rectum and most present with rectal bleeding. Although the morphogenesis is incompletely understood, some have suggested that mucosal hyperplasia is the initiating event. Dysplasia occurs in a small proportion of juvenile polyps and the juvenile polyposis syndrome is associated with an increased risk of colonic adenocarcinoma.

Option (d): Peutz-Jeghers polyps Ref: Robbins and Cotrans Pathologic Basis of Disease; 8/e, pg 816-817 This rare autosomal dominant syndrome presents at a median age of 11 years with multiple GI hamartomatous polyps and mucocutaneous hyperpigmentation. Of greater importance, it is associated with an increased risk of several malignancies including cancers of the colon, pancreas, breast, lung, ovaries, uterus and testicles, as well as other unusual neoplasms such as sex cord stromal tumours.

[Note: The risk of malignancy in the decreasing order is: Villous adenoma > Other adenomas > Polyps associated with juvenile polyposis syndrome > Peutz-Jeghers syndrome] 217. Charles-Phillips technique is used for a) Sigmoid volvulus b) Caecal volvulus c) Recurrent small intestinal obstruction d) Large intestinal obstruction c) Recurrent small intestinal obstruction Bailey & Loves Short Practice of Surgery, 24/e, pg 1192-1193 Treatment of recurrent intestinal obstruction due to adhesions: Repeat adhesiolysis (enterolysis) alone Nobles plication operation: In this, all involved intestine is fixed. Adjacent coils (average length 15-20 cm) are sutured with serosal sutures to form gentle curves. If only a proportion of the small bowel is plicated, the mesentery must be united to prevent intestinal hernias. This procedure is time-consuming and associated with a high morbidity and recurrent symptoms. Charles-Phillips transmesenteric plication: In this, following adhesiolysis, the bowel is placed in an orderly fashion. Three long synthetic sutures are passed through the mesentery of the plicated bowel, each doubled back upon itself and tied loosely. The stitch should pass a few centimeters from the bowel wall and not be adjacent to it. The resultant bowel should look like a packet of sausages. Results from this procedure are relatively good. Intestinal intubation Intraluminal tube insertion (Baker), via a Witzel jejunostomy or gastrostomy, may facilitate the formation of gentle curves. Most tubes have an inflatable balloon near the tip to facilitate placement within the caecum. This procedure is associated with a long postoperative ileus, and reports of outcomes are conflicting. The most common site of a solitary rectal ulcer in on which wall of rectum? a) Anterior b) Posterior c) Right lateral d) Left lateral a) Anterior Bailey & Loves Short Practice of Surgery; 24/e, pg 1228-1229 Solitary Rectal Ulcer: Classically, it takes the form of an ulcer on the anterior wall of the rectum. In this form, it must be differentiated from a rectal carcinoma or inflammatory bowel disease, particularly Crohns disease.

Ans: Ref: 1. 2. 3. 4. 218.

Ans: Ref:

25853434, 42433051, 9873314110, 9953550295, 8447461114

96

DELHI ACADEMY OF MEDICAL SCIENCES PVT. LTD.


219.

MAHARASTRA PAPER - 2011

In recent years, it has been appreciated that the ulceration may heal, leaving a polypoid appearance. A variety of explanations as to its cause have been suggested, including persistent trauma by sexual malpractices. However, recent proctographic studies indicate that the cause may be due to a combination of internal intussusception or anterior rectal wall prolapsed, and an increase in intra-rectal pressure. This combination of factors is usually sue to chronic straining as a result of constipation. The histological appearances confirm the diagnosis and they are similar to the appearances of biopsies from a full thickness overt rectal prolapse. The condition, although benign, is difficult to treat. Symptomatic relief from bleeding and discharge may sometimes be achieved by preventing the internal prolapsed by an abdominal rectopexy. In rare cases, rectal excision may be required. Delorme a) b) c) d) surgery is used to treat Rectal prolapse Solitary rectal polyp Rectal carcinoma Ulcerative proctitis

Ans: Ref:

a) Rectal prolapse Bailey & Loves Short Practice of Surgery; 24/e, pg 1225-1226 Treatment of rectal prolapse: Surgery is required and the operation can be performed via the perineal or abdominal approaches. An abdominal rectopexy is often recommended, but when the patient is elderly and very frail, or is suffering from injury or disease of the spinal cord, or in very early life, a perineal operation is indicated. Since an abdominal procedure risks damage to the pelvic autonomic nerves, resulting in possible impotence, a perineal approach is also usually preferred in young men.

PERINEAL APPROACH: 1. Delormes operation In this procedure, the rectal mucosa is removed circumferentially from the prolapsed rectum over its length, apart from 0.5 cm strips at its proximal end and at its tip. The underlying muscle is then imbricated with a series of chromic catgut sutures, such that, when these are tied, the rectal muscle is concertinaed towards the anal canal. The anal canal mucosa is then sutured circumferentially to the rectal mucosa remaining at the tip of the prolapse. This manouvre has the effect of reducing the prolapse and creating a ring of muscle within the anal canal, which narrows the orifice and prevents recurrence. 2. 3. Thiersch operation This procedure, which aimed to placea steel wire or, more commonly, a silastic or nylon suture around the anal canal has become obsolete. The reasons for its lack of popularity were that the suture would often break or cause chronic perineal sepsis, or both; or the anal stenosis so created would produce severe functional problems. Delormes operation is now the preferred perineal operation. Altemeirs procedure It consists of excision of the prolapsed rectum and associated sigmoid colon from below, and a colo-anal anastomosis is then constructed. It can be combined with other procedures designed to strengthen the anal sphicnters and pelvic floor.

ABDOMINAL APPROACH: The principle of all abdominal operations for rectal prolapse is to replace and hold the rectum in its proper position. Of the many operations described, the relatively simpler ones are recommended in patients with complete prolapse who are otherwise in good health. 1. Wells operation In this operation, the rectum is fixed firmly to the sacrum by inserting a sheet of polypropylene mesh between them. The rectum is separated from the sacrum and mobilized in the usual way. The mesh is fixed by a series of sutures to the periosteum over the midline of the sacrum and is then wrapped loosely about the rectum covering all exceot the anterior wall. The free margins of the mesh are sutured to the lateral margins of the anterior wall of the rectum, after exerting upward tension on it.

25853434, 42433051, 9873314110, 9953550295, 8447461114

97

DELHI ACADEMY OF MEDICAL SCIENCES PVT. LTD.


2. 220.

MAHARASTRA PAPER - 2011

The peritoneal floor is resutured so that the mesh is excluded from the peritoneal cavity. The mesh does not give rise to a foreign body reaction, but it does produce very marked fibrous tissue formation. Recently, the technique has been performed laparoscopically, thus reducing the operative trauma and limiting the time in hospital. Ripsteins operation In this operation, the rectosigmoid junction is hitched up by a Teflon sling to the front of the sacrum just below the sacral promontory. The operation is very safe and simple and the recurrence rate is low. Instead of using a Teflon sling, many surgeons merely suture the mobilized rectum to the sacrum using 4-6 interrupted non-absorable sutures so called sutured rectopexy. Since an abdominal rectopexy may lead to severe constipation, some surgeons recommend combining this procedure with resection of the sigmoid colon, so called resection rectopexy. However, only about 50% of adult patients with a complete rectal prolapsed are incontinent, and rectopexy cures only about one-third. Consequently, it may be necessary to perform a subsequent procedure to correct the incontinence. The earliest presenting feature of renal tuberculosis is a) increased daytime urinary frequency b) painless haematuria c) sterile pyuria d) painful haematuria a) increased daytime urinary frequency Bailey & Loves Short Practice of Surgery; 24/e, pg 1327 Clinical features of renal TB: Renal TB usually occurs between 20 and 40 years of age. It is twice as common in men as in women. The right kidney is affected slightly more often than the left. Urinary frequency is the earliest symptom and may be the only one. The patient complains that, over a period of months, there has been a progressive increase in both daytime and night-time frequency. Sterile pyuria: In early cases, the urine is pale and slightly opalescent. Routine urine culture is negative. Painful micturition is a feature as soon as tuberculous cystitis sets in. First there is suprapubic pain if voiding is delayed; later a burning pain accompanies micturition. When there is secondary infection, a superadded agonizing pain referred to the tip of the penis or to the vulva is often associated with haematuria or strangury. Renal pain is often minimal but there is a dull ache in the loin. In 5% of cases, the first symptom is haematuria occurring from an ulcer on a renal papilla. Constitutional symptoms are common. Weight loss is usual and a slight evening pyrexia is typical. A high temperature suggests secondary infection or dissemination, i.e. military tuberculosis. The tuberculous lesion may be difficult to detect radiologically and mycobacteria may not be cultured from the urine until the onset of more suggestive symptoms months later. A tuberculous kidney is edematous and friable and is more liable to damage than a normal kidney. However, it is unusual for a tuberculous kidney to be palpable. The prostate, seminal vesicles, vasa and scrotal contents should be examined for nodules or thickening. The treatment for a testicular teratoma is a) Orchidectomy + Retroperitoneal lymph node dissection b) Orchidectomy + Radiotherapy c) Orchidectomy + Chemotherapy d) Radiotherapy + Chemotherapy c) Orchidectomy + Chemotherapy Bailey & Loves Short Practice of Surgery; 24/e, pg 1413-1414

Ans: Ref:

221.

Ans: Ref:

Treatment of testicular tumours by staging and histological diagnosis after orchidectomy: SEMINOMAS Seminomas are radio-sensitive and excellent results have been obtained by irradiating stage 1 and stage 2 tumours. More recently, the tumour has been shown to be highly sensitive to cisplatin, which is already being used for patients with metastatic disease.

25853434, 42433051, 9873314110, 9953550295, 8447461114

98

DELHI ACADEMY OF MEDICAL SCIENCES PVT. LTD.

MAHARASTRA PAPER - 2011

The experts are divided as to whether patients with stage 1 disease should be treated with adjuvant chemotherapy.

TERATOMAS: Teratomas are less sensitive to radiation. Stage 1 tumours can be managed by watching the levels of serum markers and by repeated CT. Teratomas at stages 2-4 are managed by chemotherapy. Cisplatin, methotrexate, bleomycin and vincristine have been used in combination with great success. There are those who advocate adjuvant chemotherapy for stage 1 teratoma, arguing that effective prophylaxis is less troublesome to the patient than prolonged surveillance. Retroperitoneal lymph node dissection is sometimes needed when retroperitoneal masses remain after chemotherapy. Note: Stages of testicular tumours: Stage 1: Testis lesion only; no spread. Stage 2: Nodes below the diaphragm only Stage 3: Nodes above the diaphragm Stage 4: Pulmonary or hepatic metastases. 222. Posterior urethral valves are most commonly located at a) At the verumontanum b) Distal to verumontanum c) Proximal to verumontanum d) In the navicular fossa b) Distal to verumontanum Bailey & Loves Short Practice of Surgery; 24/e, pg 1388-1389 Congenital valves of the posterior urethra: These are symmetrical folds of urothelium which can cause obstruction to the urethra of boys. They are usually found just distal to the verumontanum, but they may lie within the prostatic urethra. They behave as flap valves; so, although urine does not flow normally, a urethral catheter can be passed without difficulty. The diagnosis of dilation of the urinary tract caused by urethral valves is noe commonly made by ultrasound scanning before birth. In some instances, the valves are incomplete and the patient remains without symptoms until adolescence or adulthood. In such cases, the prostatic urethra is grossly dilated and saccules and diverticulae are present within it. The valves are difficult to see on urethroscopy because the flow of irrigant sweeps them into the open position. If the bladder is filled with contrats medium, the dilation of urethra above the valves can be demonstrated on a voiding cystogram. Treatment: A suprapubic catheter is inserted to relieve the backpressure and allow the effects of renal failure to subside before definitive treatment by transurethral resection of the valves using a paediatric resectoscope. Carcinoma of the prostate most commonly arises from which zone? a) Peripheral b) Central c) Transitional d) None of these a) Peripheral Bailey & Loves Short Practice of Surgery; 24/e, pg 1370 Surgical anatomy of prostate: The contemporary classification of the prostate into different zones was based on the work of McNeal. He showed that it is divided into Peripheral zone (PZ), which lies mainly posteriorly and from which most carcinomas arise; Central zone (CZ), which lies posterior to the urethral lumen and above the ejaculatory ducts as they pass through the prostate, the two zones PZ and CZ rather being like an egg in its cup; Transitional zone (TZ), which is periurethral and from which most benign prostatic hyperplasia arises. Peyronies disease is

Ans: Ref:

223.

Ans: Ref:

224.

25853434, 42433051, 9873314110, 9953550295, 8447461114

99

DELHI ACADEMY OF MEDICAL SCIENCES PVT. LTD.


a) b) c) d) Ans: Ref: deformity of the erect penis # of penile shaft fixed bowing of the penis none of these

MAHARASTRA PAPER - 2011

a) deformity of the erect penis Bailey & Loves Short Practice of Surgery; 24/e, pg 1399-1400 Peyronies disease: It is a relatively uncommon cause of deformity of the erect penis. On examination, hard plaques of fibrosis can usually be palpated in the tunica of one or both corpora cavernosa. The plaques may be calcified. The presence of unyielding plaque tissue within the normally elastic wall of the corpus cavernosum causes the erect penis to bend, often dramatically, towards the side of the plaque. The etiology is uncertain, but it may be a result of past trauma there is an association with Dupuytrens contracture. Treatment: Treatment is difficult. Some cases continue to progress. Others seem to remit after 3-5 years. Various drug treatments have been suggested, but their beneficial effect is hard to prove in such a chronic condition. When the deformity of the penis is causing distress, it may be possible to straighten it by placing nonabsorbable sutures in the corpus cavernosum opposite the plaque. This reduces the elasticity in the region to balance that caused by the plaque (Nesbitts operation).

Option (c): fixed bowing of the penis Ref: Bailey & Loves Short Practice of Surgery, 24/e, pg 1399 The term chordee refers to a fixed bowing of the penis due to hypospadias, or more rarely, chronic urethritis. Erection is deformed and sexual intercourse may be impossible. Treatment is surgical.

OBSTETRICS AND GYNAECOLOGY 225. Which of these is the largest fetal diameter? a) Biparietal b) Bitemporal c) Occipitotemporal d) Submentovertical d) Submentovertical D. C. Duttas Textbook of Obstetrics; 7/e, pg 85 Antero-posterior diameters of skull Sub-occipito - bregmatic: 9.5 cm. from the nape of neck to the centre of bregma. Sub-occipito frontal: 10 cm. from the nape of neck to the anterior end of anterior fontanelle or centre of the sinciput. Occipito frontal: 11.5 cm. from the occipital eminence to the root of the nose (glabella) Mento vertical: 14 cm. from the mid-point of chin to the highest point on the sagittal suture. Sub-mento vertical: 11.5 cm. from junction of the floor of the mouth and neck to the highest point on the sagittal suture. Sub-mento bregmatic: 9.5 cm. from junction of the floor of the mouth and neck to the centre of the bregma. Transverse diameters of skull Biparietal diameter: 9.5 cm. between the two parietal eminences. Whatever may be the position of the head, this diameter nearly always engages. Super-subparietal: 8.5 cm. from a point placed below one parietal eminence to a point placed above the other parietal eminence of the opposite side. Bitemporal: 8 cm. between the antero-inferior ends of the coronal suture. Bimastoid: 7.5 cm. between the tips of the mastoid processes. The diameter is incompressible and it is impossible to reduce the length of the bimastoid diameter by obstetrical operation. If the lowermost part of the fetal head is at the level of ischial spine, then the presenting part is said to be at a) - 1 station b) zero station c) + 1 station

Ans: Ref: 1. 2. 3. 4. 5. 6. 1. 2. 3. 4.

226.

25853434, 42433051, 9873314110, 9953550295, 8447461114

100

DELHI ACADEMY OF MEDICAL SCIENCES PVT. LTD.


d) Ans: Ref: none of these

MAHARASTRA PAPER - 2011

b) zero station D. C. Duttas Textbook of Obstetrics; 7/e, pg 133 Station of the head in relation to ischial spines Spines are the most prominent bony projections felt on internal examination and the bispinous diameter is the shortest diameter of the pelvis in transverse plane being 10.5 cm. The level of ischial spines is the halfway between pelvic inlet and outlet. This level is known as station zero (0). The levels above and below the spines are divided into the fifths to represent centimeters. The station is said to be 0 if the presenting part is at the level of the spines. The station is said in minus figures, if it is above the spines (-1cm, -2cm, -3cm, -4cm, -5cm) and in plus figures if it is below the spines (+1cm, +2cm, +3cm, +4cm, +5cm). The fetal heart rate is classified as tachycardia if the value is more than _____ beats/min. a) 120 b) 140 c) 160 d) 180 c) 160 Williams Obestetrics; 23/e, pg

227.

Ans: Ref:

The National Institute of Child Health and Human Development Research Planning Workshop Definitions of Fetal Heart Rate Patterns (on a Non-stress Test): Pattern Interpretation Baseline The mean FHR rounded to increments of 5 bpm during a 10-min segment, excluding: Periodic or episodic changes Periods of marked FHR variability Segments of baseline that differ > 25 bpm Baseline variability The baseline must be for a minimum of 2 min in any 10-min segment. Fluctuations in the FHR of two cycles per min or greater Absentamplitude range undetectable Minimalamplitude range detectable but 5 bpm Moderate (normal)amplitude range 625 bpm Markedamplitude range > 25 bpm. Tachycardia Baseline FHR > 160 bpm Bradycardia Baseline FHR < 110 bpm Acceleration A visually apparent increaseonset to peak in less than 30 secin the FHR from the most recently calculated baseline. Prolonged acceleration lasts 2 min, but < 10 min. If an acceleration lasts 10 min, it is baseline change. Early deceleration In association with a uterine contraction, a visually apparent, usually symmetrical, gradualonset to nadir 30 secdecrease in FHR with return to baseline. Nadir of the deceleration occurs at the same time as the peak of the contraction. Late deceleration In association with a uterine contraction, a visually apparent, gradualonset to nadir 30 sec decrease in FHR with return to baseline. Onset, nadir, and recovery of the deceleration occur after the beginning, peak, and end of the contraction, respectively. Variable deceleration An abrupt onset to nadir < 30 sec, visually apparent decrease in the FHR below the baseline The decrease in FHR is 15 bpm, with a duration of 15 sec but < 2 min Prolonged deceleration Visually apparent decrease in the FHR below the baseline Deceleration is 15 bpm, lasting 2 min but < 10 min from onset to return to baseline Ref: Williams Obestetrics; 23/e, pg

The fetal heart rate now ranges from 110 to 160 bpm and is heard as a double sound resembling the tick of a watch under a pillow. 228. The best parameter on ultrasonography to detect the well-being of fetus beyond the 2nd trimester is a) Femur length b) Biparietal diameter c) Abdominal circumference d) Head circumference

25853434, 42433051, 9873314110, 9953550295, 8447461114

101

DELHI ACADEMY OF MEDICAL SCIENCES PVT. LTD.


Ans: Ref: c) Abdominal circumference D. C. Duttas Textbook of Obstetrics; 7/e, pg 643

MAHARASTRA PAPER - 2011

Gestational age assessment in the second trimester using obstetric ultrasound: In the second trimester, the optimal time for most accurate assessment of gestational age is between 14 and 20 weeks. The parameters used are biparietal diameter (BPD), head circumference (HC), abdominal circumference (AC) and femur length (FL). The fetal head is imaged in a transverse axial section at the level of falx cerebri, cavum septum pellucidum and the thalamic nuclei. BPD is recorded from outer skull edge to the inner edge on the opposite side. Femur length is measured when the beam from the transducer is perpendicular to the shaft. AC is measured preferable at the level of the junction of the left and right portal veins and as round as possible. AC is the sngle most sensitive parameter for assessment of fetal growth. Transcerebellar diameter (TCD) is an accurate predictor of gestational age when done between 14 and 28 weeks. It is rarely affected by fetal growth aberrations. The uterus has retained the aborted products of conception within a closed cervical os for many days (maybe weeks). Such an abortion is termed as a) Complete b) Threatened c) Missed d) Inevitable c) Missed D. C. Duttas Textbook of Obstetrics; 7/e, pg 163

229.

Ans: Ref:

When the fetus is dead and retained inside the uterus for a variable period, it is called missed abortion or silent miscarriage or early fetal demise. Option (a): Complete Ref: D. C. Duttas Textbook of Obstetrics; 7/e, pg 162 When the products of conception are expelled en masse, it is called complete abortion Option (b): Threatened Ref: D. C. Duttas Textbook of Obstetrics; 7/e, pg 160 Threatened abortion is a clinical entity where the process of abortion has started but has not progressed to a state from which recovery is impossible. Option (d): Inevitable. Ref: D. C. Duttas Textbook of Obstetrics; 7/e, pg 161 It is the clinical type of abortion where the changes have progressed to a state from where recovery is impossible. 230. Mifepristone and misoprostol combination can be safely used for medical termination of preganancy till what gestational age? a) 63 days b) 56 days c) 49 days d) 72 days c) 49 days D. C. Duttas Textbook of Obstetrics; 7/e, pg 174

Ans: Ref:

Mifepristone (RU-486) and misoprostol: Mifepristone, an analogue of progestin (norethindrone), acts as an antagonist, blocking the effect of natural progesterone. Addition of low dose prostaglandins (PGE1) improves in the efficiency of first trimester abortion. It is highly successful when used within 50 days of gestation. C/I: MIfepristone should not be used in women aged over 35 years, heavy smokers and those on long term corticosteroids. Methorexate and misoprostol:

25853434, 42433051, 9873314110, 9953550295, 8447461114

102

DELHI ACADEMY OF MEDICAL SCIENCES PVT. LTD.

MAHARASTRA PAPER - 2011

Methotrexate 50mg/m2 IM (before 56 days of gestation) (option b) followed 7 days later by misoprostol 800 g vaginally is highly effective. Misoprostol may have to be repeated 24 hours later if it fails. Tamoxifen and misoprostol: Oral tamoxifen 20 mg daily for 4 days followed by misoprostol 800 g vaginally results in complete abortion in 92% of cases. Duration of pregnancy should be less than 63 days. (option a) 231. A female presenting with h/o amenorrhoea, bleeding PV, excessive vomiting and height of uterus more than the period of amenorrhoea is most likely suffering from a) Hydatidiform mole b) Pregnancy c) Choriocarcinoma d) Tubal ectopic a) Hydatidiform mole D. C. Duttas Textbook of Obstetrics; 7/e, pg 192-193

Ans: Ref:

Hydatidiform mole Clinical features: The patient gives a h/o amenorrhea of 8-12 weeks with initial features suggestive of pregnancy. Vaginal bleeding is the commonest presentation (90%). The blood may be mixed with a gelatinous fluid from ruptured cysts giving the appearance of the discharge as white currant in red currant juice. Varying degrees of abdominal pain The patient becomes sick due to no apparent reason. The vomiting of pregnancy becomes excessive to the stage of hyperemesis (15%) probably due to excess of hCG. Breathlessness due to pulmonary embolization of the trophoblastic cells. Thyrotoxic features of tremors or tachycardia may be present on occasion. Expulsion of grape-like vesicles per vaginum is diagnostic of the condition. Signs Features suggestive of early months of pregnancy are evident. The patient looks more ill than can be accounted for. Pallor is present and may be unusually out of proportion to the visible blood loss. Features of pre-ecclampsia are present in about 50%.

Per abdomen The size of the uterus is more than the period of amenorrhea (70%), equal (20%) or smaller (10%). The feel of the uterus is firm elastic (doughy) due to the absence of the amniotic fluid sac. Fetal parts are not felt. External ballotment cannot be elicited. Absence of fetal heart sound even by Doppler effect cardioscope. Per vaginum Internal ballotment cannot be elicited. U/L or B/L enlargement (theca lutein cysts) of ovary may be palpable. Finding of vesicles in the vaginal discharge is pathognomonic. If the cervical os is open, blood clot or the vesicles may be felt instead of the membranes. Option (b): Pregnancy. The complaints of bleeding PV and excessive vomiting go against a normal pregnancy. Option (c): Choriocarcinoma Refer Q. no. 245 Option (d): Tubal ectopic Ref: D. C. Duttas Textbook of Obstetrics; 7/e, pg 180-181 The classic triad of symptoms of acute ectopic are abdominal pain (100%) preceded by amenorrhea (75%) and lastly, vaginal bleeding (70%). The absence of abdominal pain in the question rules out tubal ectopic. 232. The most common variety of conjoint twins is a) ischiopagus

25853434, 42433051, 9873314110, 9953550295, 8447461114

103

DELHI ACADEMY OF MEDICAL SCIENCES PVT. LTD.


b) c) d) Ans: Ref: thoracopagus pyopagus omphalopagus

MAHARASTRA PAPER - 2011

b) thoracopagus D. C. Duttas Textbook of Obstetrics; 7/e, pg 200 Monozygotic twins: If the division of the zygote takes place within 72 hours after fertilization (prior to morula stage), the resulting embryos will have two separate placentae, chorions and amnions: Diamniotic-Dichorionic If the division takes place between the 4th and the 8th day after the formation of the inner cell mass when chorion has already developed: Diamniotic-Monochorionic If the division occurs after the 8th day of fertilization, when the amniotic cavity also has already formed: Monoamniotic-Monochorionic

On extremely rare occasions, the division of the zygote occurs after 2 weeks of the development of the embryonic disc, resulting in the formation of conjoint twins: Siamese twins. Four types of fusion may occur: Thoracopagus (commonest) Pyopagus (posterior fusion) Craniopagus (cephalic) Ischiopagus (causal) [Note: The order of frequency of Siamese twins is given by the mnemonic: TOPIC; i.e. Thoracopagus (at the chest) Omphalopagus (at the umbilicus) Pyopagus (at the back) Ischiopagus (at the ischium) Craniopagus (at the head)] 233. According to the ACOG definition, for the diagnosis of gestational diabetes mellitus, the 2-hr post-prandial blood glucose should be above what level? a) 180 b) 155 c) 140 d) 126 b) 155 D. C. Duttas Textbook of Obstetrics; 7/e, pg 282

Ans: Ref:

Criteria for diagnosis of GDM with 100g oral glucose (OSullivan and Mahan, modified by Carpenter and Coustan) and National Diabetes Data Group. GTT: Venous plasma glucose levels (mg/dL) Time Carpenter & Coustan NDDG Fasting 95 105 1 hour 180 190 2 hours 155 165 3 hours 140 145 GDM is diagnosed when any two values are met or elevated Criteria for diagnosis of impaired glucose tolerance and diabetes with 75g oral glucose (American Diabetic Association) Plasma glucose (mg% or mg/dL) Time Normal tolerance Impaired glucose tolerance Diabetes Fasting < 100 100 and < 126 126 2 hour post glucose < 140 140 and < 200 200 Venous whole blood values are 15% less than plasma m mol/L = mg% x 0.0555

234.

Which of these is the most common obstetric cause of acute renal failure? a) HELLP syndrome b) Abruptio placentae c) Sepsis d) Pre-eclampsia/HTN

25853434, 42433051, 9873314110, 9953550295, 8447461114

104

DELHI ACADEMY OF MEDICAL SCIENCES PVT. LTD.


Ans: Ref: d) Pre-eclampsia/HTN Williams Obstetrics; 23/e.

MAHARASTRA PAPER - 2011

Today, obstetric reanl failure is most often associated with pre-eclampsia / eclampsia. Ref: D. C. Duttas Textbook of Obstetrics; 7/e, pg 622

Pre-renal ARF is the most common form of ARF in pregnancy. It is due to mild to moderate degree of renal hypoperfusion. Acute cortical necrosis is relatively uncommon and seen in abruptio placentae (option b) and endotoxic shock following Gram negative septicemia. (option c) Causes of ARF in pregnancy Early pregnancy: 1. Acute and massive haemorrhage Abortion Ectopic pregnancy Hydatidiform mole 2. Severe dehydration Hyperemesis gravidarum Acute pyelonephritis 3. Septic abortion Septicemia Endotoxic shock Hypotension Late pregnancy and labour 1. Acute and massive haemorrhage Post-partum haemorrhage Placenta previa Traumatic delivery Obstetric shock 2. Abruptio placentae 3. Severe pre-eclampsia/eclampsia HELLP syndrome 4. Severe infection Chorioamnionitis IUFD Pyelonephritis [Note: HELLP syndrome is itself a complication of pre-eclampsia] 235. HIV transmission from mother to child is maximally and most effectively carried out in which of the following stages? a) 1st trimester b) 3rd trimester c) Breast-feeding d) Birth/Delivery d) Birth/Delivery Harrisons Principles of Internal Medicine; 18/e, pg 1515 Maternal-Fetal/Infant Transmission of HIV HIV infection can be transmitted from an infected mother to her fetus during pregnancy, during delivery, or by breast-feeding. Virologic analysis of aborted fetuses indicates that HIV can be transmitted to the fetus during the first or second trimesters of pregnancy. However, maternal transmission to the fetus occurs most commonly in the perinatal period. The relative proportions of mother-to-child transmissions are 23-30% before birth, 50-65% during birth and 12-20% via breast-feeding. Midpelvis is said to be contracted if the interischial spinous diameter is less than a) 5 cm b) 8 cm c) 10 cm

Ans: Ref:

236.

25853434, 42433051, 9873314110, 9953550295, 8447461114

105

DELHI ACADEMY OF MEDICAL SCIENCES PVT. LTD.


d) Ans: Ref: 11 cm

MAHARASTRA PAPER - 2011

c) 10 cm D. C. Duttas Textbook of Obstetrics; 7/e, pg 93

Transverse diameter of mid-pelvis: Bispinous diameter (10.5 cm): It measures the distance between the two ischial spines. Ref: D. C. Duttas Textbook of Obstetrics; 7/e, pg 345

Anatomically, contracted pelvis is defined as one where the essential diameters of one or more planes are shortened by 0.5 cm. The obstetric definition states that alteration in the size and/or shape of the pelvis of sufficient degree so as to alter the normal mechanism of labour in an average size baby. Thus, it follows that midpelvis would be said to be contracted if the interischial spinous diameter is reduced by 0.5 cm from normal, i.e. it is 10 cm. 237. The incision that can be given for assisting the delivery of the head in breech is a) Zavanellis incision b) MacDonald incision c) Kellys incision d) Duherssens incision d) Duhrssens incision D. C. Duttas Textbook of Obstetrics; 7/e, pg 388 Delivery of the head through an incompletely dilated cervix in breech: If the baby is living, the cervix is to be pushed up while traction of the fetal trunk is made by malar flexion and shoulder traction (shoe-horn method). If necessary, Duhrssens incision can be made at 2 and 10 oclock position on the cervix. If the baby is dead, perforation of the head is better than watchful expectancy, hoping for full dilation of the cervix. In face-to-pubis delivery, maximum pressure is exerted along which fetal diameter? a) Suboccipitobregmatic b) Submentobregmatic c) Submentovertical d) Mentovertical b) Submentobregmatic D. C. Duttas Textbook of Obstetrics; 7/e, pg 368 Occiptito-Posterior position: In extreme deflexion (= complete extension) of head, the sinciput touches the pelvic floor first, resulting in anterior rotation of sinciput to 1/8th of a circle and putting the occiput to the sacral hollow. This is, in the true sense, Persistent Occipito-Posterior Position of the vertex. In favourable circumstances, spontaneous delivery may occur as face-to-pubis; such as with an averagesized baby, good uterine contractions and an adequate pelvis such as anthropoid or spacious gynaecoid. Further descent of the head occurs until the root of the nose hinges under the symphysis pubis. Flexion occurs, releasing successively the brow, vertex and occiput out of the stretched perineum and then the face is born by extension. The head moves 1/8th of a circle in the opposite direction of internal rotation thus turning the face towards the mothers left thigh in ROP and right thigh in LOP. The occiput further rotates to the same direction of restitution to 1/8th of a circle placing finally, the face looking directly towards the left thigh in ROP and the right thigh in LOP. In unfavourable circumstances, when arrest occurs, it is called occipito-sacral arrest.

Ans: Ref:

238.

Ans: Ref:

Now, read the following: Ref: D. C. Duttas Textbook of Obstetrics; 7/e, pg 85 Diameter Suboccipitobregmatic Suboccipitofrontal Occipitofrontal Mentovertical Attitude of head Complete flexion Incomplete flexion Marked deflexion Partial extension Presentation Vertex Vertex Vertex Brow

25853434, 42433051, 9873314110, 9953550295, 8447461114

106

DELHI ACADEMY OF MEDICAL SCIENCES PVT. LTD.


Submentovertical Submentobregmatic Incomplete extension Complete extension

MAHARASTRA PAPER - 2011


Face Face

Thus, in face-to-pubis delivery, where there is extreme deflexion or complete extension of head, submentobregmatic diameter bears the maximum pressure. 239. The first sutures to be taken while repairing a cervical tear is a) above the angle b) at the angle c) below the angle d) none of these a) above the angle D. C. Duttas Textbook of Obstetrics; 7/e, pg 424 240. The anterior and posterior margins of the torn cervix are grasped by the sponge holding forceps. Instead of giving traction to the forceps, it is better to push down the fundus gently by an assistant. This makes the tear more accessible for effective suturing. The apex is to be identified first and the first vertical mattress suture is placed just above the apex using a curved round bodied needle and chromic catgut no. 0, taking whole thickness of the cervix. The bleeding stops immediately. The rest of the tear is repaired by similar mattress sutures. Mattress suture is preferable as it prevents rolling in of the edges. Strawberry vagina is caused by a) Candida albicans b) Trichomonas vaginalis c) Gardnerella vaginalis d) Chlamydia trachomatis b) Trichomonas vaginalis Shaws Textbook of Gynaecology; 13/e, pg 125-126 In clinical practice, trichomoniasis is the most common specific vaginitis. Clinical features: 20% remain asymptomatic; others develop symptoms 4 28 days following sexual contact with an infected partner, or infected material like towel or toilet. 70% show typical discharge, which is profuse, thin, creamy or slightly green in colour, irritating and frothy. The discharge causes pruritus and inflammation of the vulva. The vaginal walls are tender, angry looking. There are often multiple, small, punctuate haemorrhagic spots on the vaginal vault and portio vaginalis of the cervix. (strawberry vagina) The patient may also complain of urinary symptoms such as dysuria and frequency, and a low-grade urethritis may be discovered on examination. Abdominal pain, low backache and dyspareunia may also be complained of. Diagnosis: In all suspected cases, it is necessary to examine a wet film preparation under the microscope. Treatment: Metronidazole 200 mg oral three times a day for 7 days should be prescribed for both the partners, and they may be advised to abstain from intercourse or use a condom. The recent modality of treatment is to shorten the duration of therapy by giving 2 g metronidazole for 1 day only. Alternatives are tinidazole 300 mg twice daily for 7 days or secnidazole in a single dose of 1000 mg daily for two days. The commonest degeneration of fibroid to occur in pregnancy is a) hyaline b) myxomatous c) calcareous d) red d) red Shaws Textbook of Gynaecology; 13/e, pg 340

Ans: Ref:

Ans: Ref:

241.

Ans: Ref:

25853434, 42433051, 9873314110, 9953550295, 8447461114

107

DELHI ACADEMY OF MEDICAL SCIENCES PVT. LTD.

MAHARASTRA PAPER - 2011

Red degeneration of fibroid: This complication of uterine myomas develops most frequently during pregnancy, although it is not rare in women over the age of 40. The myoma becomes tense and tender and causes severe abdominal pain with constitutional upset and fever. The tumour itself assumes a peculiar red colour and develops a fishy odour. The discolouration is possibly caused by diffusion of blood pigments from the thrombosed vessels. D/D: Appendicitis Twisted ovarian cyst Pyelitis Accidental haemorrhage. Ultrasound is helpful in the diagnosis Which of the following is not an indication for colposcopy? a) visible growth from cervix b) positive pap smear c) follow up of a case of CIN d) obtaining cervical biopsy a) visible growth from cervix Shaws Textbook of Gynaecology; 13/e, pg 80, 384

242.

Ans: Ref:

Colposcopy: The colposcope is a binocular microscope giving a 10 to 20 times magnification. It is useful in accurately obtaining directed biopsy from the suspicious areas on the cervix in women with positive Pap smears. This way the frequency of false-negative biopsy is reduced, so also the need for conization, a procedure which is accompanied by considerable amount of bleeding and morbidity. Indications for colposcopy: Abnormal Pap smear cytology (option b) To locate the abnormal areas To obtain directed biopsy (option d) Conservative therapy under colposcopic guidance Follow-up of cases treated conservatively (option c) [Note: Colposcopy is done to obtain biopsy and confirm the presence of cervical cancer when the Pap smear is positive or there is a suspicion of malignancy. If a growth is already visible from the cervix, a direct punch biopsy can be taken without colposcopic guidance to confirm malignancy. This is common sense.] 243. The total dose of radiation that is used as external beam radiotherapy for the treatment of CA cervix is a) ~ 80 Gy b) ~ 50 Gy c) ~ 45 Gy d) ~ 35 Gy ? D. C. Duttas Textbook of Gynaecology; 4/e, pg 325-326 For early stages of Ca Cervix, brachytherapy technique is employed. The radium dose is conventionally calculated with respect to the amount of radiation received at two arbitrary points A and B. Point A is 2 cm cephalic and 2 cm lateral to the external os and is the point of crossing of the uterine artery and the ureter. Point B is 2 cm cephalic and 5 cm lateral at the same plane and is approximately the site of the obturator gland. It has been calculated that point A gets about 7000-8000 cGy and point B 2000 cGy. Taking into consideration that the cancerolytic dose is approximately 7000 to 7500 cGy, the rest of the dose at point B is supplemented by external beam irradiation of 4000 cGy spreading over another three weeks. For external irradiation, linear accelerator with energy of 4 million electron volts or more are commonly used. In the immediate vicinity of the source, -

Ans: Ref:

25853434, 42433051, 9873314110, 9953550295, 8447461114

108

DELHI ACADEMY OF MEDICAL SCIENCES PVT. LTD.


The vagina and cervix can tolerate about 20,000 30,000 cGy. Bladder, ureter and rectum can tolerate upto 7000 cGy. Small gut on the other hand has a tolerance limit of only 4500 cGy.

MAHARASTRA PAPER - 2011

[Note: The question specifically asks the dose of radiotherapy that is given as external beam irradiation. The initial primary radiotherapy is administered as brachytherapy] 244. In the treatment of CA cervix the radiation dose given to point B is a) 2000 rad b) 3500 rad c) 5000 rad d) a) 2000 rad D. C. Duttas Textbook of Gynaecology; 4/e, pg 325-326

Ans: Ref:

Please refer previous question for explanation. [Note: 1 Gy = 100 cGy = 100 rad] 245. True about choriocarcinoma is all except a) Highly aggressive tumour b) Increased hCG levels c) Usually occurs below 15 years of age d) Ovarian tumour has excellent response to chemotherapy c) Usually occurs below 15 years of age

Ans:

Option (a): Highly aggressive tumour. Ref: D. C. Duttas Textbook of Gynaecology; 4/e, pg 336 Choricarcinoma is a highly malignant tumour arising from the chorionic epithelium. Option (b): Increased hCG levels. Ref: D. C. Duttas Textbook of Gynaecology; 4/e, pg 338 Irrespective of the site of lesion non-metastatic or metastatic, hCG levels are raised both in the 24-hour urine and serum. Excess of 2 mIU/mL in serum is of value, if pregnancy is excluded. Option (d): Ovarian tumour has excellent response to chemotherapy. Ref: D. C. Duttas Textbook of Gynaecology; 4/e, pg 338-339 The advent of chemotherapy has revolutionized the treatment of both the non-metastatic and metastatic lesions of choriocarcinoma. Chemotherapy is now the mainstay of treatment. Drug Dose Low-risk cases: Single drug regimen Methotrexate 1-1.5 mg/kg Folinic acid 0.1-0.15 mg/kg The courses are repeated at intervals of 7 days Low-risk cases: Multi-drug regimen (MAC protocol) Methotrexate 1-1.5 mg/kg Folinic acid 0.1-0.15 mg/kg Actinomycin D 12 g/kg Cyclophosphamide 3 mg/kg The courses are repeated at intervals of 2 weeks High-risk cases: EMA-CO protocol Etoposide 100 mg/m2 in 200 mL saline over 30 minutes Actinomycin 0.5 mg Methotrexate 100 mg/m2 bolus f/b 200 mg/m2 infusion over 12 Route IM/IV IM On days 1,3,5,7 2,4,6,8

IM/IV IM IV IV

1,3,5,7 2,4,6,8 15 15

IV IV bolus IV

Day 1

25853434, 42433051, 9873314110, 9953550295, 8447461114

109

DELHI ACADEMY OF MEDICAL SCIENCES PVT. LTD.


hours 100 mg/m2 in 200 mL saline over 30 minutes Actinomycin 0.5 mg Folinic acid 15 mg every 12 hours for 4 doses, beginning 24 hours after starting methotrexate Cyclophosphamide 600 mg/m2 in saline Vincristine 1 mg/m2 stat The course will restart after 7-12 days if possible. Etoposide PAEDIATRICS 246. The lactose content of human breast milk is around a) 5g b) 4.4 g c) 4.6 g d) 7g d) 7g O. P. Ghais Essential Paediatrics; 7/e, pg 122

MAHARASTRA PAPER - 2011

IV IV bolus IM

Day 2

IV IV

Day 8

Ans: Ref:

Lactose is in a high concentration (6-7 g/dL) in breast milk. The galactose is necessary for the formation of galactocerebrosides. Lactose helps in the absorption of calcium and enhances the growth of lactobacilli in the intestine. Ref: Parks Textbook of Preventive and Social Medicine; 20/e, pg 155

Nutritive value of milks compared (value per 100 g): Buffalo Fat (g) 6.5 Protein (g) 4.3 Lactose (g) 5.1 Calcium (mg) 210 Iron (mg) 0.2 Vitamin C (mg) 1 Minerals (g) 0.8 Water (g) 81 Energy (kcal) 117

Cow 4.1 3.2 4.4 120 0.2 2 0.8 87 67

Goat 4.5 3.3 4.6 170 0.3 1 0.8 86.8 72

Human 3.4 1.1 7.4 28 3 1 88 65

247.

The best way to maintain the temperature of the baby during transport is a) Kangaroo-mother care b) Transport incubator c) Thermacol box d) Hot water bottles b) Transport incubator O. P. Ghais Essential Paediatrics; 7/e, pg 153 Different workers have suggested the use of thermocol box, basket, padded pouch, polythene covering, etc. for ensuring temperature stability during transport. The ideal modality of transport incubator is a rarity in most situations. One of the best methods to provide warmth to the neonate is to use skin-to-skin contact. The naked baby is kept on the mothers (or another adults) chest. (option a) The use of hot water bottle (option d) is fraught with considerable danger due to accidental burns to the baby if the bottle is not wrapped properly and remains in touch with the babys body. It is best avoided, but if no other means are available, this method may be employed. But the accompanying members of the team should be explained to take care of the bottle on the way. The initial treatment of an infant with asymptomatic hypoglycemia is a) 100 mg/kg dextrose b) 2 mL/kg of 10% dextrose c) 4 mL/kg of 10% dextrose d) 2 mL/kg of 25% dextrose

Ans: Ref:

248.

25853434, 42433051, 9873314110, 9953550295, 8447461114

110

DELHI ACADEMY OF MEDICAL SCIENCES PVT. LTD.


Ans: Ref: b) 2 mL/kg of 10% dextrose Nelsons Textbook of Paediatrics; 18/e.

MAHARASTRA PAPER - 2011

Treatment of acute symptomatic neonatal or infant hypoglycemia includes IV administration of 2 mL/kg of D10W (10% dextrose), followed by a continuous infusion of glucose at 6-8 mg/kg/min, adjusting the rate to maintain blood glucose levels in the normal range. If hypoglycemic seizures are present, some recommend a 4 mL/kg bolus of D10W. The management of persistent hypoglycemia includes increasing the rate of IV glucose infusion to 10-15 mg/kg/min or more. All are true about ruminous behaviour except a) Most common between 3 5 years of age b) Associated with mental retardation c) Associated with regurgitation of food d) No nausea a) Most common between 3 5 years of age Rudolphs Paediatrics; 21/e.

249.

Ans: Ref:

250.

Rumination, or apparently pleasurable, habitual regurgitation and reswallowing of the stomach contents (option c), may occur with or without gastro-esophageal reflux (GER) as a result of increases in intraabdominal pressure. The fingers or fist are often inserted into the mouth to initiate a gag. This behavior also may be a symptom of esophagitis. Psychogenic rumination is a symptom of inadequate nurturing in infancy, with an average age at onset of 5 months. Self-stimulatory rumination presents at any age but almost exclusively in children with severe or profound mental retardation (option b). However, the latter also can be perpetuated by the secondary gain of the attention it attracts, especially if nurturing is inadequate. Five times as many males as females develop either kind of rumination. Clinically significant rumination is rare, making up less than 1% of admissions to children's hospitals. However, the complications of malnutrition, electrolyte imbalance, and aspiration account for a 15% mortality rate. Persistent rumination requires an evaluation for GER, developmental assessment, and investigation of all the child's caregiving environments. Most symptoms of psychogenic rumination resolve in 1 week if the caregivers hold the infant for 15 minutes before, during, and after feedings, although problems related to parenting dysfunction may require months of treatment. Rumination in children with mental retardation may require behavioral feedback. A 6-month old child is brought with three features fever, cough and fast breathing (respiratory rate = 50/min). According to the IMNCI guidelines, he will be classified as a) No pneumonia b) Pneumonia c) Severe pneumonia d) Very severe pneumonia b) Pneumonia Parks Textbook of Preventive and Social Medicine; 20/e, pg 155

Ans: Ref:

Management of a child aged 2 months up to 5 years: SIGNS Chest indrawing (If also recurrent wheezing, go directly to treat wheezing).

CLASSIFY AS: Treatment:

SEVERE PNEUMONIA Treat fever, if present. Treat wheezing, if present. Refer to hospital. (If referral is not possible, treat with an antibiotic and follow

No chest indrawing. Fast breathing ( 50/min if child 2 months up to 12 months; 40/min if child 12 months up to 5 years). PNEUMONIA Advise mother to give hime care. Give an antibiotic. Treat fever, if present. Treat wheezing, if present.

No chest indrawing. No fast breathing (< 50/min if child 2 months up to 12 months; < 40/min if child 12 months up to 5 years). NO PNEUMONIA: COUGH OR COLD If coughing for more than 30 days, refer for assessment. Assess and treat ear problem or sore throat, if

25853434, 42433051, 9873314110, 9953550295, 8447461114

111

DELHI ACADEMY OF MEDICAL SCIENCES PVT. LTD.


closely).

MAHARASTRA PAPER - 2011


present. Assess and treat other problems. Advise mother to give hime care. Treat fever, if present. Treat wheezing, if present.

Advise mother to return with child in 2 days for reassessment, or earlier if the child is getting worse.

Reassess in 2 days the child who is taking an antibiotic for pneumonia SIGNS WORSE. THE SAME Not able to drink. Has chest indrawing. Has other danger signs. TREATMENT Refer URGENTLY to Change antibiotic or Refer hospital Danger 251. signs: Not able to drink. Convulsions. Abnormally sleepy or difficult to wake. Stridor in a calm child OR Severe malnutrition The amount of trisodium citrate in a solution of ORS is a) 2.5 g b) 1.5 g c) 2.9 g d) 3.5 g c) 2.9 g O. P. Ghais Essential Paediatrics; 7/e, pg

IMPROVING Breathing slower. Less fever. Eating better Finish 5 days of antibiotic

Ans: Ref:

Oral rehydration solution (ORS) should be preferably given with a teaspoon or consumed in small sips from a cup or tumbler. A child with profuse vomiting is more likely to retain the fluid if it is consumed in small sips. Large gulps of fluid stimulate gastrocolic reflex resulting in a quick passage of stools and even vomiting. Table 11.5: Composition of WHO Oral Rehydration Salts Solution: Ingredients per liter of ORS solution (grams) Concentration of various ingredients in WHO ORS solution (mmol/L) Sodium chloride 3.5 Sodium 90 Potassium chloride 1.5 Potassium 20 Trisodium citrate dehydrate 2.9 Chloride 80 Glucose 20 Citrate 10 Water 1 litre Glucose 111 1 mmol of citrate provides 3 mEq of base. 252. The amount of ORS fluid given to a 1-year old child in the first 4 hours is a) 200 400 mL b) 400 600 mL c) 600 800 mL d) 800 1200 mL c) 600 800 mL O. P. Ghais Essential Paediatrics; 7/e, pg 265

Ans: Ref:

Table 11.8 Guidelines for treating patients with Some (But not Severe) Dehydration When Body Weight is not known (Plan B) Approximate amount of ORS solution to be given in the first 4 hours: Age Approx wt (kg) ORS (mL) < 4 months <5 200 400 4 11 months 58 400 600 12 23 months 8 11 600 800 2 4 years 11 16 800 1200

Local measure (glass) 12 23 34 46

25853434, 42433051, 9873314110, 9953550295, 8447461114

112

DELHI ACADEMY OF MEDICAL SCIENCES PVT. LTD.


5 14 years 15 years 16 30 > 30 1200 2200 > 2200

MAHARASTRA PAPER - 2011


6 11 12 20

253.

The most common cause of chylothorax in a paediatric chest is a) Damage to thoracic duct during surgery b) Lymphoma c) Chest trauma d) Metastasis a) Damage to thoracic duct during surgery Nelsons Textbook of Paediatrics; 18/e, pg Chylothorax is a pleural collection of fluid formed by the escape of chyle from the thoracic duct or lymphatics into the thoracic cavity. Chylothorax in children occurs most frequently due to thoracic duct injury as a complication of cardiothoracic surgery. Other cases are associated with chest injury or with primary or metastatic intrathoracic malignancy, particularly lymphoma. Less common causes include lymphangiomatosis; restrictive pulmonary diseases; thrombosis of the duct, superior vena cava, or subclavian vein; and congenital anomalies of the lymphatic system In newborns, rapidly increased venous pressure during delivery may lead to thoracic duct rupture. The features of Henoch-Schonlein Purpura include all except a) Abdominal pain b) Arthralgia c) Thrombocytopenic palpable purpura d) GI haemorrhage c) Thrombocytopenic palpable purpura O. P. Ghais Essential Paediatrics; 7/e, pg 587

Ans: Ref:

254a.

Ans: Ref:

Henoch-Schonlein Purpura is one of the most common vasculitides of childhood and is characterized by the presence of non-thrombocytopenic palpable purpura, transient arthralgia and abdominal symptoms. Table 21.5. ACR 1990 Criteria for Diagnosis of HSP: Criterion 1. Palpable purpura 2. Age <20 years at disease onset 3. Bowel angina

Definition Slightly raised palpable haemorrhagic skin lesions, not related to thrombocytopenia Patient <20 years at onset of first symptoms Diffuse abdominal pain, worse after meals, or the diagnosis of bowel ischemia, usually including bloody diarrhea. Histologic changes showing granulocytes in the walls of arterioles or venules

4. Wall granulocytes on biopsy

Ref: Clinical features: The illness begins with a purpuric rash more prominent over the extensor aspects of lower extremities and buttocks. It may be macular, macula-papular, or even urticarial to begin with and can be difficult to diagnose in the first few days of the illness. Glomerulonephritis is seen in approximately half of the patients but only 10 20% of these have azotemia or nephrotic range proteinuria. Clinically it may manifest as isolated haematuria, hypertension or a nephritic/nephrotic syndrome. This is the only long-term complication of HSP. Significant renal involvement is uncommon in children below 6 years of age. Gastrointestinal manifestations usually occur within the first week of the illness and may often be wrongly diagnosed as surgical abdomen. Abdominal pain is usually intermittent, colicky and periumbilical. Vomiting occurs in about 60% of patients but haematemesis and malaena are relatively less common.

25853434, 42433051, 9873314110, 9953550295, 8447461114

113

DELHI ACADEMY OF MEDICAL SCIENCES PVT. LTD.


Ref:

MAHARASTRA PAPER - 2011

Most clinical features are self-limiting and resolve in a few days. Rare manifestations of HSP include CNS vasculitis, coma, Guillian-Barre syndrome, pulmonary haemorrhage, carditis and orchitis. Nelson (18/e, chapter 166) The disease onset may be acute, with the appearance of several manifestations simultaneously, or insidious, with sequential occurrence of symptoms over a period of weeks or months. Low-grade fever and fatigue are present in more than half of affected children. The typical rash and the clinical symptoms of HSP are a consequence of the usual location of the acute small vessel damage primarily in the skin, gastrointestinal tract, and kidneys. The hallmark of the disease is the rash, beginning as pinkish maculopapules that initially blanch on pressure and progress to petechiae or purpura, which are characterized clinically as palpable purpura that evolve from red to purple to rusty brown before they eventually fade. The lesions tend to occur in crops, last from 310 days, and may appear at intervals that vary from a few days to as long as 34 months. In <10% of children, recurrences of the rash may not end until as late as a yr, and, rarely, several yr, after the initial episode. Damage to cutaneous vessels also results in local angioedema, which may precede the palpable purpura. Edema independent of purpura occurs primarily in dependent areas such as below the waist, over the buttocks (or on the back and posterior scalp in the infant), or in areas of greater tissue distensibility such as the eyelids, lips, scrotum, or dorsum of the hands and feet. Edema and damage to the vasculature of the gastrointestinal tract may also lead to intermittent abdominal pain that is often colicky in nature. There may be peritoneal exudate, enlarged mesenteric lymph nodes, segmental edema, and hemorrhage into the bowel. More than half of patients have occult heme-positive stools, diarrhea (with or without visible blood), or hematemesis. Intussusception may occur, which is suggested by an empty right lower abdominal quadrant on physical examination or by currant jelly stools, which may rarely be followed by complete obstruction or infarction with bowel perforation. If not resolved by hydrostatic reduction during a contrast study, surgical intervention is necessary. Renal involvement occurs in 2550% of children and may manifest with hematuria, proteinuria, or both; nephritis or nephrosis; or acute renal failure. Renal involvement at presentation may lead to chronic hypertension or end-stage renal disease in the future. Hepatosplenomegaly and lymphadenopathy may also be present during active disease. A rare but potentially serious outcome of neurological involvement is the development of seizures, paresis, or coma. Other rare complications include rheumatoid-like nodules, cardiac and eye involvement, mononeuropathies, pancreatitis, and pulmonary or intramuscular hemorrhage. The features of Henoch-Schonlein Purpura include all except a) Abdominal pain b) Arthralgia c) Palpable purpura d) GI haemorrhage d) GI haemorrhage (most probably):

254b.

Ans:

We are not sure whether the word thrombocytopenic was mentioned in option (c). If it was mentioned, then option (c) is the best answer, since both O.P.Ghai and Nelson mention the purpura of HSP to be unrelated to thrombocytopenia and Nelson clearly mentions the occurrence of GI haemorrhage. But if it was not mentioned, then we do not know what is the correct answer, but the best option in that case would be (d) since the other three options would then form a part of the classical tetrad of HSP. Tetrad of HSP 1. Non-thrombocytopenic palpable purpura 2. Arthritis 3. Pain in abdomen 4. Glomerulonephritis 255. Cloudy cornea is seen in a) Hurler syndrome b) Morteaux-Lamy syndrome

25853434, 42433051, 9873314110, 9953550295, 8447461114

114

DELHI ACADEMY OF MEDICAL SCIENCES PVT. LTD.


c) d) Ans: Ref: Morquio syndrome All of these

MAHARASTRA PAPER - 2011

d) All of these O. P. Ghais Essential Paediatrics; 7/e, pg 637

Table 23.3: Clinical features of mucopolysaccharidoses: Eponym / MPS number Mental Coarse facies retardation Hurler/IH + + Scheie/IS Hunter/II + + Sanfilipo/III + Morquio/IV Morteaux-Lamy/VI Sly/VII -

HS-megaly + + +

Dysostosis multiplex + + + + +

Corneal clouding + + + + -

Cloudy cornea is observed in Types I, IS and VI but it may occur in some cases of Type IV. Cloudiness of cornea is minmal in Type III, but it is not seen in Type II. (O.P.Ghai) 256. Absence seizures is characterized on EEG by a) 3-Hz spike pattern b) c) 1-2 Hz spike pattern d) a) 3-Hz spike pattern O. P. Ghais Essential Paediatrics; 7/e, pg 530 Electroencephalogram shows a characteristic 3 per second spike and slow wave pattern Absence seizures Start abruptly in childhood. Peak prevalence is between 6 8 years. Not preceeded by aura. Brief, abrupt loss of consciousness with sudden discontinuation of activity being performed, with staring spell, eye fluttering or rhythmic movements. Lasts less than 30s (usually 10s) No loss of posture No urinary incontinence No breathing difficulty Post-ictal confusion and drowsiness characteristically absent and the patients can resume their normal activity immediately after the seizure. Usually considered by teachers as inattentive pupils. Attacks occurring in close succession indicate petit-mal status or pyknolepsy. Distinguished from complex partial seizures from o Absence of aura o Shorter duration o Abrupt return of full consciousness

Ans: Ref:

ORTHOPAEDICS 257. Tension a) b) c) d) wire principle can be used for managing # of all of these except Patella Colles Olecranon Medial malleolus

Ans: Ref:

b) Colles Maheshwaris Essential Orthopaedics; 3/e, pg 95

The treatment of Colles fracture is essentially conservative. For an undisplaced fracture, immobilization in a below-elbow plaster cast for 6 weeks is sufficient. For displaced fractures, the standard method of treatment is manipulative reduction followed by immobilization in Colles cast.

25853434, 42433051, 9873314110, 9953550295, 8447461114

115

DELHI ACADEMY OF MEDICAL SCIENCES PVT. LTD.


Options (a): Patella and (c): Olecranon. Ref: Maheshwaris Essential Orthopaedics; 3/e, pg 24

MAHARASTRA PAPER - 2011

A fractured bone has a tension side, i.e. the side which tends to open up on the use of the limb because it is subjected to a distracting force. If this force is counteracted by some device fixed on the tension side, the whole of the fracture comes under compression. Such a compression can be produced by: 1. Tension-band wire: This can be used for producing dynamic compressions in fractures of patella and the olecranon 2. Tension-band plate: This can be used for fractures of the humerus and tibia 3. Tension-band fixation: This can be used for fractures of the tibia. Option (d): Medial malleolus Ref: Maheshwaris Essential Orthopaedics; 3/e, pg 139 The following techniques of internal fixation are preferred for treating ankle injuries depending upon the type of fracture Medial malleolus fracture: Transverse fracture: compression screw tension-band wiring Oblique fracture: compression screws Avulsion fracture: tension-band wiring Lateral malleolus fracture: Transverse fracture: tension-band wiring Spiral fracture: compression screws Comminuted fracture: buttress plating Fracture of lower 1/3rd of tibia: 4-hole plate Posterior malleolus fracture: Involving < 1/3rd of articulating surface of tibia: no additional treatment Involving > 1/3rd of articulating surface of tibia: internal fixation with compression screws Tibio-fibular syndesmosis disruption: Insertion of a long screw from fibula into tibia 258. Hill-Sachs lesion is seen in a) Recurrent posterior shoulder dislocation b) Inferior shoulder dislocation c) Recurrent anterior shoulder dislocation d) # neck/shaft of humerus c) Recurrent anterior shoulder dislocation Maheshwaris Essential Orthopaedics; 3/e, pg 75 The following pathological changes are seen in the commoner (anterior) shoulder dislocation: Bankarts lesion: Dislocation causes stripping of the glenoidal labrum along with the periosteum from the anterior surface of the glenoid and scapular neck. The head thus comes to lie in front of the scapular neck, in the pouch thereby created. Hill-Sachs lesion: This is a depression on the humeral head in its postero-lateral quadrant, caused by impingement by the anterior edge of the glenoid on the head as it dislocates. Rounding off of the anterior glenoid rim occurs as the head dislocates over it. There may be associated fractures of the greater tuberosity of the humerus or of the rim of the glenoid. Avascular necrosis of femur head is common in a) Intracapsular # neck of femur b) Extracapsular # neck of femur c) Inter-trochanteric # d) # shaft of femur a) Intracapsular # neck of femur Maheshwaris Essential Orthopaedics; 3/e, pg 110 Blood supply to the femur head: comes from three main sources The medullary vessels from the neck The retinacular vessels entering from the lateral side of the head. The foveal vessel from the ligamentum teres.

Ans: Ref: 1. 2. 3. 4. 259.

Ans: Ref: 1. 2. 3.

25853434, 42433051, 9873314110, 9953550295, 8447461114

116

DELHI ACADEMY OF MEDICAL SCIENCES PVT. LTD.

MAHARASTRA PAPER - 2011

The most important of these, the medullary and retinacular vessels, are generally cut off following a fracture of the neck of femur and sometimes, result in avascular necrosis of the head. Ref: Maheshwaris Essential Orthopaedics; 3/e, pg 118 Avascular necrosis of femur head as a complication of intracapsular # neck of femur: After a fracture through the neck, all the medullary and most of the capsular blood supply to the head are cut off. The viability of the femoral head may therefore depend almost entirely on the blood supply from the ligamentum teres. If this blood supply is insufficient, avascular necrosis of a segment or whole of the head occurs. This may, in addition, be a cause of non-union.

Treatment of intracapsular # neck of femur with avascular necrosis: In younger patients: 1. Arthrodesis of hip 2. Bipolar arthroplasty (a special type of prosthesis) 3. Meyers procedure 4. Girdlestones arthroplasty (rarely performed) In elderly: 1. Hemi-replacement arthroplasty. 2. Total hip replacement (in c/o associated osteoarthritis) [Note: Extracapsular # neck of femur (option b) is also known as inter-trochanteric #; (optionc) and is usually complicated by mal-union and osteoarthritis. Ref: Maheshwaris Essential Orthopaedics; 3/e, pg 118-119] 260. Which of the following bursa always communicates with the knee joint cavity? a) Suprapatellar b) Prepatellar c) Superficial infrapatellar d) Deep infrapatellar a) Suprapatellar S. Das A Manual of Clinical Surgery; 6/e, pg 189, 193

Ans: Ref:

Extra-articular swellings are quite common around the knee due to enlragement of the different bursae around the joint. 1. 2. 3. 4. 5. 6. Prepatellar bursa It is situated just in front of the lower part of the patella and ligamentum patellae. Inflammation and enlargement results from friction between the skin and the patella while cleaning the floor in kneeling posture (Housemaids knee). Infrapatellar bursa It lies deep to the lower half of the ligamentum patellae. Inflammation also results from repeated kneeling (Clergymans knee). Suprapatellar bursa It almost always communicates with the knee joint. It becomes swollen in case of effusion of the joint. Bicipital bursa It lies deep to the bicipital tendon. It may occasionally be enlarged. Semimembranous bursa It is seen behind the knee on its medial aspect and slightly above the joint knee. It becomes more prominent and tense on extension of the knee. Morrant-Bakers cyst It is a herniation of the synovial membrane posteriorly through the fibres of the oblique popliteal ligament. It forms a swelling in the middle of the posterior aspect of the knee slightly below the joint line. It becomes prominent on extension and disappears on flexion of the joint. It is often associated with tuberculosis or osteoarthritis of the joint.

[Note: Bursae around the knee joint:

25853434, 42433051, 9873314110, 9953550295, 8447461114

117

DELHI ACADEMY OF MEDICAL SCIENCES PVT. LTD.

MAHARASTRA PAPER - 2011

Anterior: Prepatellar Superficial infrapatellar Deep infrapatellar Suprapatellar Lateral: A bursa deep to the lateral head of gastrocnemius A bursa between the fibular collateral ligament and biceps femoris A bursa between the fibular collateral ligament and tendon of popliteus A bursa between the tendon of popliteus and lateral condyle of tibia Medial: A bursa deep to the medial head of gastrocnemius Ansarine A bursa deep to the tibial collateral ligament A bursa deep to the semimembranosus (occasionally) A bursa between the tendons of semitendinosus and semimembranosus.] 261. In congenital dislocation of knee, the knee is held in a) Valgus b) Hyperextension c) Flexion d) Varus b) Hyperextension Campbells Operative Orthopaedics; 11/e, pg Congenital hyperextension and dislocation of knee: Congenital hyperextension of the knee is only the lowest grade of an abnormality that is divided into three according to severity: Grade 1, congenital hyperextension; grade Grade 2, congenital hyperextension with anterior subluxation of the tibia on the femur; and Grade 3, congenital hyperextension with anterior dislocation of the tibia on the femur. Congenital hyperextension or dislocation of the knee usually is associated with skeletal abnormalities elsewhere in the extremity. The pathological condition usually varies with the severity of the deformity, but always the anterior capsule of the knee and the quadriceps mechanism are contracted. As the severity of the anterior displacement of the tibia increases, other findings include intraarticular adhesions and other abnormalities within the joint and hypoplasia or absence of the patella. Treatment: The treatment of congenital hyperextension of the knee depends on the severity of the subluxation or dislocation and the age of the patient. In a newborn with mild-to-moderate hyperextension or subluxation, conservative treatment methods, such as the use of the Pavlik harness for posturing of the knee in a continued position and serial casting to increase knee flexion, are most likely to succeed. According to most authors, nonoperative treatment can be continued for 3 months. In children who do not respond to conservative measures, the use of skeletal traction for correction is an option, but the deformity is difficult to correct with this method. In older children with moderate or severe subluxation or dislocation, surgery is indicated. In a child with congenital dislocation of the knee and congenital dislocation of the hip, surgical correction of the knee first is advisable. Curtis and Fisher described a procedure for correction of congenital dislocation of the knee that is recommended for children 6 to 18 months old. The technique combines anterior capsular release, lengthening of the quadriceps mechanism, and release of intraarticular adhesions. Occasionally, the articular surfaces of the knee remain abnormal if the deformity recurs. Ideally, a functional range of motion can be obtained. In rare cases osteotomy of the femur or tibia may be required in an older child. The deformity most commonly seen in a case of osteoarthritis of knee is a) Valgus b) Recurvatum c) Varum d) None of these

Ans: Ref:

grades

262.

25853434, 42433051, 9873314110, 9953550295, 8447461114

118

DELHI ACADEMY OF MEDICAL SCIENCES PVT. LTD.

MAHARASTRA PAPER - 2011

Ans: Ref:

c) Varum Maheshwaris Essential Orthopaedics; 3/e, pg 252-253 Clinical features of OA Symptoms: Pain is the earliest symptom Swelling of the joint is a late feature Stiffness initially due to pain and muscle spasm, but later, capsular contracture and incongruity of the joint surface.

Signs on examination: Tenderness on the joint line Crepitus on moving the joint Irregular and enlarged-looking joint due to formation of peripheral osteophytes Deformity varus of the knee, flexion-adduction-external rotation of the hip Effusion rare and transient Terminal limitation of joint movement with wasting of quadriceps muscle Subluxation detected on ligament testing 263. The ligament most commonly involved in ankle sprain is a) Anterior talofibular b) Posterior talofibular c) Calcaneo-fibular d) Deltoid a) Anterior talofibular

Ans:

Before coming to ankle sprain, let us know the following first: LIGAMENTS OF THE ANKLE: (Ref: Maheshwaris Essential Orthopaedics; 3/e, pg 252-253) 1. Medial collateral ligament (Deltoid ligament) (option d): This is a strong ligament on the medial side. It has a superficial tibio-calcaneal part and a deep tibio-talar part. Lateral collateral ligament: This is a weak ligament and is often injured. It has three parts: a) Anterior talo-fibular (option a) b) Middle calcaneo-fibular (option c) c) Posterior talo-fibular (option b)

2.

Now, coming to ANKLE SPRAIN Ref: Maheshwaris Essential Orthopaedics; 3/e, pg 140 It is the term used for the ligament injuries of the ankle. Commonly, the lateral collateral ligament is sprained, mostly so the anterior talo-fibular component. Diagnosis: The patient gives history of a twisting injury to the ankle followed by pain and swelling over the injured ligament. Weight-bearing leads to excruciating pain. If a torn ligament is subjected to the following manouvres, the patient experiences severe pain 1. Inversion of a plantar-flexed foot (for anterior talo-fibular ligament sprain) 2. Inversion in neutral position (for complete lateral collateral ligament sprain) 3. Eversion in neutral position (for medial collateral ligament sprain) Treatment: Grade I: Below-knee plaster cast x 2 weeks f/b mobilization Grade II: Below-knee plaster cast x 4 weeks f/b mobilization Grade III: Below-knee plaster cast x 6 weeks f/b mobilization The trend today is to treat ligament injuries by functional method, i.e. without immobilization. The functional treatment consists of rest, ice packs, compression and elevation (RICE) for the first 2-3 days. The patient begins early, protected range-of-motion exercises.

25853434, 42433051, 9873314110, 9953550295, 8447461114

119

DELHI ACADEMY OF MEDICAL SCIENCES PVT. LTD.


264.

MAHARASTRA PAPER - 2011

Surgical repair is preferred by some surgeons for Grade III injuries, especially in young athletic individuals. A sequestrum is a) a dead piece of bone b) new bone formed around a dead piece of bone c) a dead piece of bone surrounded by infected granulation tissue d) none of these c) a dead piece of bone surrounded by infected granulation tissue Maheshwaris Essential Orthopaedics; 3/e, pg 160-161 Sequestrum: Sequestrum is a piece of dead bone, surrounded by infected granulation tissue, trying to eat the sequestrum away. It appears pale and has a smooth inner and rough outer surface, because the latter is being constantly eroded by the surrounding granulation tissue. Involucrum: Involucrum is the dense sclerotic bone overlying the sequestrum. There may be some holes in the involucrum for pus to drain out. These holes are called cloacae. The bony cavities are lined by infected granulation tissue. On X-ray of the bone, Codmans triangle is a feature seen in which of the following? a) Rickets b) Scurvy c) Osteosarcoma d) Chondrosarcoma c) Osteosarcoma Maheshwaris Essential Orthopaedics; 3/e, pg 218 219 X-ray features of osteosarcoma: An area of irregular destruction in the metaphysis, sometimes overshadowed by new bone formation The cortex overlying the lesion is eroded. New bone formation in the matrix of the tumour. Periosteal reaction: As the tumour lifts the perisoteum, it incites an intense periosteal reaction. The periosteal reaction in osteosarcoma is irregular, unlike in osteomyelitis, where it smooth and in layers. Codmans triangle: A triangular area of subperiosteal new bone is seen at the tumour-host cortex junction at the ends of the tumour. Sun-ray appearance: As the periosteum in unable to contain the tumour, the tumour grows into the overlying soft tissues. New bone is laid down along the blood vessels within the tumour growing centrifugally, giving a sun-ray appearance. Which of the following statements about giant cell tumour of the bone is true? a) Usually occurs below 15 years of age b) Epiphyseal origin c) Sclerotic lesion always seen on X-ray d) Always benign b) Epiphyseal origin Maheshwaris Essential Orthopaedics; 3/e, pg 216

Ans: Ref:

265.

Ans: Ref:

266.

Ans: Ref:

Giant cell tumour. The tumour is located at the epiphysis; the area which was the epiphysis before its fusion with the metaphysis. If often reaches almost up to the joint surface. Option (a): Usually occurs below 15 years of age Ref: Maheshwaris Essential Orthopaedics; 3/e, pg 216 The tumour is commonly seen in the age group of 20 40 years, i.e. after the epiphyseal fusion. Option (c): Sclerotic lesion always seen on X-ray Ref: Maheshwaris Essential Orthopaedics; 3/e, pg 216

25853434, 42433051, 9873314110, 9953550295, 8447461114

120

DELHI ACADEMY OF MEDICAL SCIENCES PVT. LTD.

MAHARASTRA PAPER - 2011

X-ray features: A solitary, may be loculated, lytic lesion. Eccentric location, often subchondral Expansion of the overlying cortex (expansile lesion) Soap-bubble appearance the tumour is homogenously lytic with trabeculae of the remnants of bone traversing it, giving rise to a loculated appearance. No calcification within the tumour. None or minimal reactive sclerosis around the tumour The cortex may be thinned out, or perforated at places. The tumour usually does not enter the adjacent joint.

Option (d): Always benign Ref: Maheshwaris Essential Orthopaedics; 3/e, pg 216 Though the tumour is generally classified as benign, it tends to recur after local removal. 1/3rd of the tumours are benign, 1/3rd are locally malignant and 1/3rd are frankly malignant. 267. The joint most commonly involved in ankylosing spondylitis is a) Sacroiliac b) Zygapophyseal (intervertebral joint) c) Atlanto-axial d) Cervical spine a) Sacroiliac Harrisons Principles of Internal Medicine; 18/e, pg 2774-2778 Ankylosing spondylitis (AS): It is an inflammatory disorder of unknown cause that primarily affects the axial skeleton. Peripheral joints and extra-articular structures are also frequently affected.

Ans: Ref:

Etiology: Age: 2nd 3rd decade Sex: F:M = 2:1 to 3:1 Pathology: Sacroilitis is often the earliest manifestation of AS. Synovitis, pannus, myxoid marrow, subchondral granulation tissue and marrow edema, enthesitis and chondroid differentiation are found. Macrophages, T cells and osteoclasts are prevalent. Eventually, the eroded joint margins are gradually replaced by fibrocartilage regeneration and then by ossification. The joint may become totally obliterated. Clinical manifestations: The initial symptom is usually dull pain, insidious in onset, felt deep in the lumbar or gluteal region. It is accompanied by low-back morning stiffness of up to a few hours duration that improves with activity and returns following inactivity. Within a few months, the pain usually becomes persistent and bilateral. Nocturnal exacerbation of pain often forces the patient to rise and move around. 268. All of the following are features of hyperparathyroidism except a) Osteosclerosis is seen b) Pepper-pot skull is seen c) There is loss of lamina dura around the teeth d) Brown tumour is seen a) Osteosclerosis is seen Maheshwaris Essential Orthopaedics; 3/e, pg 266 X-ray features of hyperparathyroidism Irregular, diffuse rarefaction of the bones Salt-pepper appearance: The skull bones show a well-marked stippling, but the opaque areas are small, pin-head size. Loss of lamina dura: A tooth socket is made up of thin cortical bone seen as a white line surrounding the teeth. This is called the lamina dura. It gets absorbed in hyperparathyroidism.

Ans: Ref:

25853434, 42433051, 9873314110, 9953550295, 8447461114

121

DELHI ACADEMY OF MEDICAL SCIENCES PVT. LTD.


269.

MAHARASTRA PAPER - 2011

Sub-periosteal resorption of the phalanges is a diagnostic feature of hyperparathyroidism (generalized variety). It may occur at the lateral end of the clavicle also The spine shows the central collapse of the vertebral body and bi-convex discs. The pelvis and other bones show coarse striations with clear cyst-like spaces. Brown tumour is an expansile lytic lesion generally affecting the maxilla/mandible. Extra-osseous radiological features such as renal calculi, may be present. Blue sclera is seen in a) Marfans syndrome b) Osteogenesis imperfecta c) Osteopetrosis d) All of these b) Osteogenesis imperfecta Harrisons Principles of Internal Medicine; 18/e, pg 3207-3208 Ocular features of Osteogenesis imperfecta: The sclera can be normal, slightly bluish or bright blue. The colour is probably caused by a thinness of the collagen layers of the sclerae that allows the choroid layers to be seen. Blue sclerae, however, are an inherited trait in some families who do not have increased bone fragility.

Ans: Ref:

Expanded classification of Osteogenesis Imperfecta: Type Bone fragility Blue sclerae I Mild Present II Extreme Present III IV V VI VII SKIN 270. Severe Variable Moderate to severe Moderate to severe Moderate Bluish at birth Absent Absent Absent Absent

Abnormal dentition Present in some Present in some Present in some Absent in IVa Present in IVb Absent Absent Absent

Hearing loss Present in most Unknown High incidence High incidence

Inheritance AD Sporadic, rarely AR AD, rarely AR AD AD Unknown AR

Absent

Koebners phenomenon is NOT a feature of which of the following? a) Erythema multiforme b) Scleromyxedema c) Molluscum contagiosum d) Dariers disease b) Scleromyxedema http://en.wikipedia.org/wiki/Koebner_phenomenon

Ans: Ref:

Koebners phenomenon: The Koebner phenomenon, also called the "isomorphic response", refers to skin lesions appearing on lines of trauma. The Koebner phenomenon may result from either a linear exposure or irritation. It is seen in: Psoriasis Lichen planus Lichen nitidus Pityriasis rubra pilaris Vitiligo Lichen sclerosus Elastosis perforans serpiginosa Kaposi sarcoma Dariers disease A similar response occurs in pyoderma gangrenosum and Adamantiades-Behcet's syndrome, and is referred to as pathergy. Warts and molluscum contagiosum are often listed as causing a Koebner reaction, but this is by direct inoculation of viral particles.

25853434, 42433051, 9873314110, 9953550295, 8447461114

122

DELHI ACADEMY OF MEDICAL SCIENCES PVT. LTD.

MAHARASTRA PAPER - 2011

Option (a): Erythema multiforme: Ref: http://www.nature.com/jid/journal/v124/n6/full/5602835a.html; http://emedicine.medscape.com/article/1122915-overview and Fitzpatricks Dermatology in General Medicine; 7/e, pg 346 Erythema multiforme (EM) is an acute, self-limited, and sometimes recurring skin condition that is considered to be a type IV hypersensitivity reaction associated with certain infections, medications, and other various triggers. Erythema multiforme minor - Typical targets or raised, edematous papules distributed acrally Erythema multiforme major - Typical targets or raised, edematous papules distributed acrally with involvement of one or more mucous membranes; epidermal detachment involves less than 10% of total body surface area (TBSA). The lesions appear as papules that evolve into pathognomonic target lesions or iris lesions that appear within a 72-hour period and begin on the extremities. Lesions remain in a fixed location for at least 7 days and then begin to heal. Lesions may also appear as arcuate lesions. Because this condition may be related to a persistent antigenic stimulus, recurrence is the rule rather than the exception, with most affected individuals experiencing 1-2 recurrences per year. Precipitating factors include herpes simplex virus (HSV), Epstein-Barr virus (EBV), and histoplasmosis. The insult to the skin seemingly must happen after HSV lesions appear and during a proposed viremic phase in order for erythema multiforme (EM) lesions to occur. Prior investigators noted that the Koebner phenomenon occurs in HSV-associated EM, but only before the appearance of skin lesions. Attempts to induce additional skin lesions after the appearance of target lesions have failed. However, mechanical factors including Koebners phenomenon and actinic factors (predilection of sunexposed sites) do appear to influence the distribution of lesions.

Option (b): Scelromyxedema: Ref: http://en.wikipedia.org/wiki/Papular_mucinosis Papular mucinosis is a rare skin disease. Localized and disseminated cases are called papular mucinosis or lichen myxedematosus while generalized, confluent papular forms with sclerosis are called scleromyxedema. Another form, acral persistent papular mucinosis is regarded as a separate entity. Papular mucinosis is chronic and may be progressive. The dermal layer of the skin breaks out into small and solid bumps of 2 to 4 mm, usually conical in shape or sometimes flat-topped papules. Unlike pustules, these bumps do not contain pus. Instead they contain mucin. Histopathology of individual papules revealed the presence of mucinous deposits in the mid-dermal region that stained positively with toluidine blue. They usually come in clusters such as linear arrays. Less frequently, urticarial, nodular, or sometimes annular lesions may be appreciated. The dorsal aspect of the hands, face, elbows, and extensor portions of the extremities are most frequently affected. Mucosal lesions are absent. The coalescence of papules on the face, particularly on the glabella, results in longitudinal folding and gives the appearance of a leonine facies. In scleromyxedema, symptoms can occur on larger part of the body. Erythema and scleroderma-like induration occurs on the skin. The Koebners phenomenon has been described.* In addition, the mobility of the lips, hands, arms, and legs is reduced. Proximal myopathy, inflammatory polyarthritis, central nervous system symptoms, esophageal aperistalsis, and hoarseness are among the notable systemic symptoms. If viscera is involved, the disease will be fatal. The dermatoneuro syndrome is a rare neurological complication of the disease presenting with fever, seizures and altered mental status.

* Ref: Scleromyxedema: a case treated with oral prednisone by da Trindade, de Oliveira, et al; Scleromyxedema revisited by Pomann, Rudner, et al. Option (c): Mollascum contagiosum Ref: Fitzpatricks Dermatology in General Medicine; 7/e, pg 1911-1913 Molluscum contagiosum: A benign viral infection that frequently affects children. It is caused by a pox virus Molluscipox. It tends to be more severe in immunocompromised individuals and those with atopic dermatitis.

25853434, 42433051, 9873314110, 9953550295, 8447461114

123

DELHI ACADEMY OF MEDICAL SCIENCES PVT. LTD.

MAHARASTRA PAPER - 2011

Transmission Direct skin or mucous membrane contact Fomites Autoinoculation and koebnerization also play a role Clinical features: Extremely small, pink, pearly, flesh-coloured papules that enlarge, occasionally reaching the size of up to 3 cm (giant molluscum). A dome-shaped, opalescent morphology becomes apparent as the papule enlarges. The lesions may have a central dell or umbilication, within which a curd-like substance can be expressed with pressure. Most patients develop multiple papules often in the intertriginous areas. The lesions may be grouped in clusters or may appear in a linear array. The latter often results from koebnerization or development of lesions at the sites of trauma. Erythema and eczematous changes may occur around lesions; this is termed molluscum dermatitis. Treatment: Topical cantharidin 0.7% or 0.9% Curettage Cryotherapy Topical anaesthetic agents

Option (d): Dariers disease Ref: Fitzpatricks Dermatology in General Medicine; 7/e, pg 432-436 Dariers disease Also known as Darier-White disease or keratosis follicularis. Late onset (around puberty) genetic disorder of keratinization. Caused by loss of function mutations in ATP2A2 gene encoding the sarco/endoplasmic reticulum calcium ATP-ase pump isoform 2 (SERCA2); which leads to impairment in calcium signaling. Clinical features: Warty papules and plaques in seborrheic areas Fragile nails that show a combination of red and white longitudinal stripes with a V-shaped neck at the free margin of the nail, associated with subungual hyperkeratosis. Papules on the dorsum of hands and feet Palmo-plantar pits Treatment: Mild disease Avoidance of sun and use of sunscreens Emollients containing urea and lactic acid to reduce crusting Topical tretinoin/isotretinoin/tazarotene Topical steroids Severe disease: Oral acitretin

[Note: Scleromyxedema is the answer as it is the rarest of the four options] 271. Dennie-Morgan folds are seen in a) Seborrheic dermatitis b) Plaque psoriasis c) Impetigo herpetiformis d) Atopic dermatitis d) Atopic dermatitis Fitzpatricks Dermatology in General Medicine; 7/e, pg 147 Features of Atopic Dermatitis: Major features Pruritus Rash on face and/or extensors in infants and young children Lichenification in flexural areas in older children Tendency towards chronic or chronically relapsing dermatitis Personal or fmily history of atopic disease: asthma, allergic rhinitis, atopic dermatitis, etc.

Ans: Ref:

25853434, 42433051, 9873314110, 9953550295, 8447461114

124

DELHI ACADEMY OF MEDICAL SCIENCES PVT. LTD.

MAHARASTRA PAPER - 2011

272.

Other common findings Dryness Dennie-Morgan folds accentuated lines or grooves below the margin of the lower eyelid. Allergic shiners darkening beneath the eyes Facial pallor Pityriasis alba Keratosis pilaris Ichthyosis vulgaris Hyperlinearity of palms and soles White dermatographism white line appears on the skin within 1 minute of being stroked with blunt instrument Conjunctivitis Keratoconus Anterior subcapsular cataracts Elevated serum IgE Immediate skin test reactivity Auspitz a) b) c) d) sign is seen in which of the following disorders? Pemphigus Lichen planus Psoriasis Pityriasis rosea

Ans: Ref:

c) Psoriasis Uday Khopkars An Illustrated Handbook of Skin Diseases and Sexually Transmitted Infections; 5/e, pg 127

Fully established psoriasis consists of well-defined rounded erythematous plaques covered with thick silvery scales. When the scales are removed, pinpoint bleeding is visible on the involved skin (Auspitz sign). 273. On histopathology, elongation of rete pegs and Max Jospehs space are seen in a) Psorasis b) Lichen planus c) Pityriasis alba d) Pityriasis rosea b) Lichen planus Fitzpatricks Dermatology in General Medicine; 7/e, pg 251 Histology of lichen planus: The two major pathologic findings in lichen planus are basal epidermal keratinocyte damage and lichenoidinterface lymphocytic reaction. The epidermal changes include hyperkeratosis, wedge-shaped areas of hypergranulosis and elongation of rete ridges that resemble a saw-tooth pattern. Multiple apoptotic cells or colloid-hyaline bodies (Civette bodies) are seen at the dermo-epidermal junction. Eosinophilic colloid bodies are present in the papillary dermis. They are PAS-positive and measure approximately 20 in diameter. A band-like lymphocytic infiltrate is seen in the papillary dermis that abuts the epidermis. Many histiocytes and few plasma cells are seen. Plasma cells are more prominent in mucous membrane specimens. Few eosinophils are seen in drug-induced lichen planus or lichenoid drug eruptions. Melanin pigmentation is invariably present and is more pronounced in older, waning lesions and in darkskinned individuals. Separation of the epidermis in small clefts (Max Joseph space) is occasionally seen. Intra-epidermal split is seen in a) Pemphigus vulgaris b) Bullous pemphigoid c) Epidermolysis dystrophica bullosa d) All of these a) Pemphigus vulgaris Fitzpatricks Dermatology in General Medicine; 6/e, pg 558

Ans: Ref:

274.

Ans: Ref:

25853434, 42433051, 9873314110, 9953550295, 8447461114

125

DELHI ACADEMY OF MEDICAL SCIENCES PVT. LTD.


Ref:

MAHARASTRA PAPER - 2011

In pemphigus vulgaris and pemphigus vegitans, acantholysis selectively involves the layer of cells immediately above the basal cell layer. The single layer of intact basal cells that forms the blister base has been likened to a row of tomb stones. The suprabasal acantholytic blister is characteristic of pemphigus vulgaris. Neena Khanna, 2/e, pg 62 Immuno-fluorescence Intercellular Intercellular Intercellular subepidermal Pattern IgG, fish net IgG, fish net IgG, fish net Target antigen Desmoglein 3 Desmoglein 1 Plakins (desmo-plakin, envoplakin, periplakin, BP230) BP230 > BP180 BP230 > BP180 (?) Epidermal tissue transglutaminase BP180

Pemphigus vulgaris Pemphigus foliaceus Paraneoplastic pemphigus

and

Bullous pemphigoid Herpes gestationalis Dermatitis herpetiformis Linear IgA disease

Basement membrane zone Basement membrane zone Dermal papillae Basement membrane zone

IgG, linear IgG, linear IgA, granular IgA, linear

275.

A 24-year old male following a sexual intercourse with a CSW develops a painless penile ulcer that bleeds on touch. Most probable diagnosis is a) Syphilis b) Herpes simplex c) Donovanosis d) Lymphogranuloma venereum c) Donovanosis Uday Khopkars An Illustrated Handbook of Skin Diseases and Sexually Transmitted Infections; 5/e, pg 280

Ans: Ref:

D/D of penile ulcer Causative organism Chancroid Haemophilus ducreyi Donovanosis Calymmatobacterium granulomatis 8-80 days Variable Insidious Papule Single / Multiple 2-10 Variable + ++ Dark red granulation tissue Deep ++ Rolled out, variable Serosanguinous, profuse Variable Slow Wrights stain coccobacilli within macrophages Streptomycin, Cotrimoxazole, Erythromycin Primary syphilis Treponema pallidum Herpes genitalis Herpes simplex virus

Incubation period Symptoms Onset Lesions preceding ulcer No. of ulcers Size of ulcers (cm) Base Tenderness Bleeding Floor Slough Depth Ulcer raised the surface Edge Discharge Surrounding skin Progression Smear

2-7 days Painful Acute Vesiculo-pustule Multiple 0.5-2 Soft + + Slough ++ Deep

9-90 days Painless Insidious Papule Single 1-2 Indurated granulation Variable Variable, sometimes punched out Serosanguinous, scanty Normal Slow, self-healing Dark ground illumination T. pallidum Injectable penicillin

3-6 days Painful Acute Grouped vesicles Multiple 0.2-0.3 Soft + Slough Superficial Sloping Serosanguinous, variable Variable Fast, self-healing Wrights stain multinucleated epithelial giant cells Acyclovir

Pale tissue

above

Undermined Purulent, profuse Red Fast Gm ve bacilli parallel chains Cotrimoxazole, Erythromycin

in

Rx

ANAESTHESIA

25853434, 42433051, 9873314110, 9953550295, 8447461114

126

DELHI ACADEMY OF MEDICAL SCIENCES PVT. LTD.


276. The technique of capnography can be used for a) the depth of anaesthesia b) monitoring of O2 saturation c) confirming the position of the endotracheal tube d) monitoring the pH c) confirming the position of the endotracheal tube Ajay Yadavs Short Textbook of Anaesthesia; 3/e, pg 48

MAHARASTRA PAPER - 2011

Ans: Ref:

1. 2. 3. 4. 5. 6. 7. 8.

Capnography: It is the continuous measurement of end tidal carbon dioxide (ETCO2) and its waveform. [Normal: 32-42 mm Hg; 3-4 mm Hg less than arterial pCO2, which is 35-45 mm Hg] It works on the principle that infrared light is absorbed by carbon dioxide. It is a very important and sensitive tool for monitoring in anaesthesia. Uses of capnography: It is the surest confirmatory sign of correct intubation (esophageal intubation will yield ETCO2 = 0) Intra-operative displacement of the endotracheal tube (ETCO2 will become 0) Diagnosing malignant hyperthermia (ETCO2 may rise to > 100 mm Hg) For detecting obstructions and disconnections of endotracheal tubes (ETCO2 will fall) Diagnosing pulmonary embolism by air, fat or thrombus (sudden fall in ETCO2 occurs. It may become 0 if embolus is large enough to block total pulmonary circulation) Exhausted sodalime or defective valves of closed circuit will show high ETCO2 values. To control level of hypocapnia during hyperventilation in neurosurgery. Indicator of cardiac output: In cardiac arrest, ETCO2 is 0.

Option (b): monitoring the O2 saturation. Ref: Ajay Yadavs Short Textbook of Anaesthesia; 3/e, pg 48 Pulse oximetry: It is to measure the oxygen saturation in blood. (SpO2) [Normal SpO2 = 97-98%] A source of light is emitted at two wavelengths (660 and 940 nm) from a probe which, when passed through tissue containing bood, is absorbed by pulsatile arterial and capillary blood and non-pulsatile venous blood and tissue. The ratio of the two is calculated by pulse oximeter which converts it into oxygen saturation. It is used for detection of hypoxia in the intra-op and peri-op period.

Option (a): Monitoring the depth of anaesthesia. Ref: Ajay Yadavs Short Textbook of Anaesthesia; 3/e, pg 51 The depth of anaesthesia is mainly monitored clinically. s/s of light anaesthesia are Tachycardia Hypertension Lacrimation Perspiration Movement response to painful stimuli Tachypnea, breath holding, coughing, laryngospasm, bronchospasm Eye movements Preserved reflexes Desynchronisation in EEG Patient-evoked responses (auditory is best) The internationally accepted colour of nitrous oxide cylinder is a) White b) Blue c) Black d) Grey b) Blue Ajay Yadavs Short Textbook of Anaesthesia; 3/e, pg 22

277.

Ans: Ref:

Colours of gas cylinders: Gas cylinder Oxygen

Colour Black with white shoulders

25853434, 42433051, 9873314110, 9953550295, 8447461114

127

DELHI ACADEMY OF MEDICAL SCIENCES PVT. LTD.


Air Nitrous oxide Entonox Carbon dioxide Cyclopropane Helium Also remember: Pin indices. Gas Nitrogen Air Carbon dioxide (> 7.5%) Oxygen Carbon dioxide (< 7.5%) Nitrous oxide Cyclopropane Entonox

MAHARASTRA PAPER - 2011

Grey with black and white shoulders Blue Blue with blue and white shoulders Grey Orange Brown

Pin index 1,4 1,5 1,6 2,5 2,6 3,5 3,6 7

278.

All of the following are contraindicated in a patient of chronic renal failure requiring anaesthesia except a) Enflurane b) Pancuronium c) Pethidine d) Midazolam d) Midazolam Ajay Yadavs Short Textbook of Anaesthesia; 3/e, pg 78 Midazolam is 3 times more potent than diazepam. The advantages over diazepam are: Water-based preparation. So injection is painless. Elimination t is 2-3 hours. So can be safely used for day-care procedures. Reversal with flumazenil is complete.

Ans: Ref:

1. 2. 3.

Option (a): Enflurane Ref: Ajay Yadavs Short Textbook of Anaesthesia; 3/e, pg 64 Enflurane depresses renal functions due to decreased renal blood flow. C/I to enflurane: Epilepsy Renal disease. Option (b): Pancuronium. Ref: Ajay Yadavs Short Textbook of Anaesthesia; 3/e, pg 64 Only 10-20% of pancuronium is metabolized in liver. 80-90% is excreted by kidneys. So it should be avoided in renal diseases. Option (c): Pethidine Ref: Goodman & Gilmans The Pharmacological Basis of Therapeutics; 12/e, pg 502, 504 In humans, pethidine (meperidine) is hydrolyzed to meperidinic acid, which in turn is partially conjugated. Meperidine is also N-demethylated to normeperidine. In patients with cirrhosis, the bioavailability of meperidine is increased to as much as 80% and the T of both meperidine and normeperidine are prolonged. Since normeperidine is eliminated by the kidney and the liver, decreased renal or hepatic function increases the likelihood of toxicity. 279. The neuromuscular blocker that is eliminated by Hoffmans elimination is a) Atracurium b) Vecuronium c) Rocuronium d) Mivacurium a) Atracurium Ajay Yadavs Short Textbook of Anaesthesia; 3/e, pg 92

Ans: Ref:

25853434, 42433051, 9873314110, 9953550295, 8447461114

128

DELHI ACADEMY OF MEDICAL SCIENCES PVT. LTD.

MAHARASTRA PAPER - 2011

Atracurium: Available as atracurium besylate. To be stored at 4OC Pharmacokinetics: It is an acidic compound. So can precipitate if given in IV line containing alkaline solution like thiopentone. Dose: 0.5 mg/kg Onset of action: 2-3 min Duration of action: 10-15 min Metabolism: It undergoes Hoffmann degradation (95%) and ester hydrolysis (5%) in plasma. Its metabolism is independent of hepatic and renal functions. Uses: Atracurium is the relaxant of choice in: Hepatic failure Renal failure Myasthenia gravis Newborns Old age Side effects: Histmaine release: much less than d-Tubocurarine. Cardiovascular system: can cause hypotension because of histamine release. Respiratory system: can cause bronchospasm because of histamine release. CNS: At higher doses, its metabolic product, laudanosine can produce convulsions. Allergic: Allergic reactions ranging from pruritic rash to angioneurotic edema can occur. Cistracurium: It is an isomer of atracurium. It is 4 times more potent than atracurium. The chief advantage of cis-atracurium over atracurium is that it does not release histamine. The effects of scoline can be potentiated by which of the following? a) Isoflurane b) Halothane c) Xenon d) Nitrous oxide a) Isoflurane Goodman & Gilmans The Pharmacologic Basis of Therapeutics; 12/e, pg 260

280.

Ans: Ref:

In other animal species and occasionally in humans, depolarizing agents like succinylcholine (scoline) produce a blockade that has unique features. Some of these combine are those of depolarizing and non-depolarizing (competitive) agents. This is termed as a dual mechanism. The depolarizing agent initially produces characteristic fasciculations and potentiation of the maximal twitch, followed by the rapid onset of neuromuscular block. This phase I block is potentiated by anticholinesterase agents: Ambenonium Edrophonium Neostigmine Pyridostigmine Donepezil Galantamine Rivastigmine Tacrine Under clinical conditions, with increasing concentrations of succinylcholine and over time, the block may convert slowly from a depolarizing phase I block to a non-depolarizing phase II block. This pattern of neuromuscular blockade produced by depolarizing agents in anaesthesized patients appears to depend, in part, on the anaesthetic. Fluorinated hydrocarbons may be more apt to predispose the motor end plate to non-depolarization blockade after prolonged use of succinylcholine Enflurane

25853434, 42433051, 9873314110, 9953550295, 8447461114

129

DELHI ACADEMY OF MEDICAL SCIENCES PVT. LTD.


Isoflurane Desflurane Sevoflurane

MAHARASTRA PAPER - 2011

[Note: The best potentiator of Phase II action of Sch is enflurane. Hence, if the question is repeated with enflurane as one of the options, prefer that as the answer over isoflurane] 281. Which of these local anaeshetics has maximum binding with plasma proteins? a) Lignocaine b) Amethocaine c) Procaine d) Bupivacaine d) Bupivacaine Ajay Yadavs Short Textbook of Anaesthesia; 3/e, pg 92 Bupivacaine: 4 times more potent than xylocaine. Available as 0.5% solution. The solution is very stable. Highly cardiotoxic; hence should not be used for Biers block. Metabolized in liver: t = 3.5 hours. Duration of effect: o Without adrenaline: 2-3 hours o With adrenaline: 3-5 hours. Addition of adrenaline has no effect on motor blockade, it only prolongs the duration of sensory block. Maximum safe dose: 2 mg/kg with/without adrenaline. Concentrations used: For nerve block: 0.5% Epidural: 0.25 to 0.5% Spinal: 0.5% (heavy) Side effect: Cardiotoxicity of bupivacaine is much higher than lignocaine. This cardiotoxicity increases in pregnancy, hypoxia and acidosis. Its high tissue binding and high protein binding makes resuscitation prolonged and very difficult. Drug of choice for ventricular tachycardia produced by bupivacaine is bretylium. In case of non-availability of bretylium, lignocaine may be considered an alternative. For ventricular tachycardia not responding to drugs, cardioversion should be contemplated immediately. The most effective and reliable way to treat post-dural puncture headache is a) Epidural infusion of Hartmanns solution b) Epidural blood patch c) Using smaller bore needles d) IV fluids + NSAIDs b) Epidural blood patch Morgans Anaesthesia; 3/e, pg 275 Post-Dural Puncture Headache: Any breach of dura, as may follow a lumbar puncture, myelogram, spinal anaesthesia, wet epidural tap, (in which the epidural needle has pierced the dura) or uncomplicated epidural tap (due to scratching of the dura) may result in post-dural puncture headache. It results from the decreased intracranial pressure following the leak of CSF from the dural defect. It is bilateral, frontal or retro-orbital or occipital, extending into the neck. It may be throbbing and associated with photophobia and nausea. The hallmark of the headache is its association with the body posture; it is aggravated by sitting or standing and relieved by lying down. The onset is usually at 12-72 hours following the procedure and may last for weeks. The incidence is higher with a Larger needle Cutting point needle (in comparison to a pencil point needle) Young age Females Pregnancy

Ans: Ref:

282a.

Ans: Ref:

25853434, 42433051, 9873314110, 9953550295, 8447461114

130

DELHI ACADEMY OF MEDICAL SCIENCES PVT. LTD.


1. 2. 3. 4. 5. 6. Ref: The management of the condition is Prevention by thinner needle Supine position IV fluids Analgesics (NSAIDs) Caffeine Epidural blood patch (most effective treatment) Millers Anaesthesiology; 7/e,

MAHARASTRA PAPER - 2011

Possibly more important than knowing all the variables resulting in an increased incidence of postspinal puncture headache is understanding how and when to carry out the definitive therapyepidural blood patchfor this complication. Successful application of spinal anesthesia necessitates the early use of epidural blood patching when indicated. This therapy was introduced by Gormley. Its safety and efficacy (>90% effective in relieving headache per epidural patch) have been well documented, and contemporary practice has validated that epidural blood patch continues to have a greater than 90% improvement rate. The most effective and reliable way to prevent post-dural puncture headache is a) Epidural infusion of Hartmanns solution b) Epidural blood patch c) Using smaller bore needles d) IV fluids + NSAIDs

282b.

Ans: c) Using smaller bore needles Please refer Q. 282a We are not very sure whether the question asked treatment or prevention of post-dural puncture headache. Hence, we have given both the versions. [Note: The best way to prevent hypotension during spinal anaesthesia is by preloading with IV fluids] RADIOLOGY 283. Radioactive elements emit all of the following radiations except a) -rays b) -rays c) -rays d) X-rays d) X-rays Graingers Diagnostic Radiology; Gamma rays and X-rays are both a form of electromagnetic radiation, and have the same properties. Roentgen discovered and named X-rays; these were produced by a beam of electrons striking a heavy object, the same mechanism as in a modern X-ray tube. A few years later, the names alpha, beta and gamma were given to the different rays emitted by radioactive uranium. It was only later realized that X-rays and gamma rays are essentially the same. The modern definitions are that gamma rays are emitted by an atomic nucleus in an excited state, whereas X-rays are the result of changes involving electrons. Some beta emitters produce gamma photons in the course of beta decay. The decay is a two-stage process, with beta emission leaving the nucleus in an excited state. The properties exhibited by gamma rays and X-rays are both wave-like and particle-like. The particles are called photons. Each photon has a definite energy and zero mass. Its wave-like properties are a frequency which is in proportion to its energy, and a wavelength which is in inverse proportion.

Ans: Ref:

Alpha decay occurs only when a heavy nucleus ejects an alpha particle. The alpha particle () consists of two neutrons and two protons and is identical to a helium nucleus. Due to their very short range, alpha particles have the potential to deliver a lethal radiation dose to small metastatic cell clusters, while mostly sparing the surrounding tissue

25853434, 42433051, 9873314110, 9953550295, 8447461114

131

DELHI ACADEMY OF MEDICAL SCIENCES PVT. LTD.


284.

MAHARASTRA PAPER - 2011

A beta particle (beta- or -) is an electron emitted by a nucleus. Beta emitters are routinely used to deliver therapeutic radiation doses to tumours. Which of these following molecules is the best sensitizer of tissues to radiation? a) Hydrogen b) Oxygen c) Cisplatin d) Gemcitabine b) Oxygen Harrisons Principles of Internal Medicine; 18/e, pg 691

Ans: Ref: Ref:

Radiation delivery causes breaks in DNA and generates free radicals that may damage cell membranes, proteins and organelles of tumour cells. Radiation damage is augmented dependent on only oxygen; hypoxemic cells are more resistant. Augmentation of oxygen is the basis of radiation sensitization. Certain drugs used in cancer treatment may also act as radiation sensitizers. For e.g. compounds that incorporate into DNA and alter their stereochemistry (halogenated pyrimidines, cisplatin) augment radiation effects; as does hydroxyurea, another DNA synthesis inhibitor. Oxford Oncology; 2/e, pg 462

Drugs that potentiate the action of radiotherapy to hypoxic tissues: (A): Radiosensitizers: They are oxygen mimetics; they can substitute for O2 1. Nitroimidazoles: Metronidazole Misonidazole Etanidazole Pimonidazole Nimorazole 2. Anticancer drugs: Actinomycin D Bleomycin Cisplatin Doxorubicin 5-Fluorouracil Hydroxyurea 3. Idurd Brdud 4. Hyperbaric O2 (most potent) (B): Bioreductive drugs: They act on the hypoxic tissue directly. 1. Quinones: Mitomycin C Porfiromycin 2. Nitroimidazoles: RSU 1069 3. Benzotriazines: Di-N-oxide Tirazapamine 285. Which of the following is a stable isotope? 18O a) 14C b) 125I c) 32P d)
18O a) Graingers Diagnostic Radiology;

Ans: Ref:

Unstable radionuclide
14Carbon 15Oxygen 125

Iodine

Half-life 5730 years 2 minutes 60 days

Main use Non-imaging diagnosis PET Non-imaging diagnosis

25853434, 42433051, 9873314110, 9953550295, 8447461114

132

DELHI ACADEMY OF MEDICAL SCIENCES PVT. LTD.

MAHARASTRA PAPER - 2011

Beta-emitting radionuclides are most commonly used for therapy as they deliver most of the radiation dose locally. Examples are 131iodine- for thyrotoxicosis and thyroid cancer, 32phosphorus- for polycythaemia vera, 90yttrium- for arthritic conditions and 89strontium- for bone metastases 286. Which of these conditions detected on a plain X-ray film of the abdomen is a sufficient indication by itself for cholecystectomy? a) porcelain gall bladder b) gall stones c) limey bile d) a) porcelain gall bladder Graingers Diagnostic Radiology; Ref: Porcelain gallbladder is an uncommon condition of mural calcification associated with chronic cholecystitis. The calcification follows the contour of the gallbladder wall, may be generalized or localized, and may be visible on CT or plain radiography. On USG, it can mimic emphysematous cholecystitis or gallstones but the double-arc shadow sign of stones is absent. It may be asymptomatic but cholecystectomy is often advocated as the reported incidence of a complicating carcinoma in up to 33 per cent. Schwartzs Principles of Surgery; 8/e, pg Since few patients with gall stones (option a) develop complications without previous biliary symptoms, prophylactic cholecystectomy in asymptomatic persons with gallstones is rarely indicated. For elderly patients with diabetes, for individuals who will be isolated from medical care for extended periods of time, and in populations with increased risk of gallbladder cancer, a prophylactic cholecystectomy may be advisable. Porcelain gallbladder, a rare premalignant condition in which the wall of the gallbladder becomes calcified, is an absolute indication for cholecystectomy. Which of these conditions can be best diagnosed by HRCT-thorax? a) Pneumonia b) Bronchogenic carcinoma c) Tuberculosis d) Interstitial lung disease d) Interstitial lung disease Harrisons Principles of Internal Medicine; 18/e, pg 2163 High resolution computed tomography is superior to the plain CXR for early detection and confirmation of suspected ILD. In addition, HRCT allows better assessment of the extent and distribution of the disease, and is especially useful in the investigation of patients with a normal chest radiograph. Co-existing disease e.g. mediastinal adenopathy, carcinoma or emphysema, is often best recognized on HRCT scanning. When a lung biopsy is required, HRCT scanning is useful for determining the most appropriate area from which biopsy samples are taken. In the appropriate clinical setting, HRCT may be sufficiently characteristic to preclude the need for lung biopsy in idiopathic pulmonary fibrosis, sarcoidosis, hypersensitivity pneumonitis and lymphangitic carcinoma.

Ans: Ref:

287.

Ans: Ref:

Option (a): Pneumonia Ref: Harrisons Principles of Internal Medicine; 18/e, pg 2132 Chest radiography is often necessary too differentiate community acquired pneumonia from other conditions. CT is rarely necessary but may be of value in a patient with suspected postobstructive pneumonia caused by a tumour or foreign body.

Option (b): Bronchogenic carcinoma Ref: Harrisons Principles of Internal Medicine; 18/e, pg 743

25853434, 42433051, 9873314110, 9953550295, 8447461114

133

DELHI ACADEMY OF MEDICAL SCIENCES PVT. LTD.


MAHARASTRA PAPER - 2011

Tissue sampling is required to confirm the diagnosis in all patients with suspected lung cancer. Tumour tissue may be obtained via minimally invasive techniques such as bronchial or transbronchial biopsy during fiberoptic bronchoscopy , fine-needle aspiration or percutaneous biopsy using imaging guidance or endobronchial ultrasound-guided biopsy.

Option (c): Tuberculosis Ref: Harrisons Principles of Internal Medicine; 18/e, pg 1350 The initial suspicion of pulmonary TB is often based on abnormal chest radiographic findings in a patient with respiratory symptoms. Although the classic pattern is that of upper lobe disease with infiltrates and cavities, virtually any radiographic pattern from a normal film or a solitary pulmonary nodule to diffuse alveolar infiltrates in a patient with adult respiratory distress syndrome may be seen. CT may be useful in interpreting questionable findings on plain chest radiography and may be helpful in diagnosing some forms of extrapulmonary TB (e.g. Potts disease). MRI may be useful in the diagnosis of intracranial TB.

PSYCHIATRY 288. The MMSE (Mini-Mental Status Examination) is said to be abnormal if the score is below a) 30 b) 28 c) 25 d) 23 c) 25 Kaplan and Sadocks Synopsis of Psychiatry; 10/e, pg 185-186 Although a formal evaluation of cognitive impairment requires time-consuming consultation with an expert in psychological testing, one practical and clinically useful test for practitioners is the Mini-Mental Status Examination (MMSE). The MMSE is a screening test that can be used during a patients clinical examination It is also a practical test to track the changes in a patients cognitive state. Of a possible 30 points, a score below 25 suggests possible impairment, and a score below 20 indicates definite impairment. Harrisons Principles of Internal Medicine; 18/e, pg 3303

Ans: Ref:

Ref:

Mini-Mental Status Examination: Points Orientation Name: season/date/day/month/year Name: hospital/floor/town/state/country Registration Identify three objects by name and ask patient to repeat Attention and calculation Serial 7s; subtract from 100 (e.g: 93 86 79 72 65) Recall Recall the three objects presented earlier Language Name pencil and watch Repeat No ifs, ands or buts Follow a 3-step command (e.g: Take this paper, fold it in half and place it on the table) Write close your eyes and ask patient to obey written command Ask patient to write a sentence Ask patient to copy a design (e.g: intersecting pentagons) Total 5 (1 for each name) 5 (1 for each name) 3 (1 for each object) 5 (1 for each subtraction) 3 (1 for each object) 2 (1 for each object) 1 3 (1 for each object) 1 1 1 30

289.

Catastrophic reaction is characteristic of which of the following? a) Depression b) Schizophrenia c) Dementia d) Anxiety c) Dementia

Ans:

25853434, 42433051, 9873314110, 9953550295, 8447461114

134

DELHI ACADEMY OF MEDICAL SCIENCES PVT. LTD.


Ref: Niraj Ahujas A Short Textbook of Psychiatry; 6/e, pg 24

MAHARASTRA PAPER - 2011

290.

Clinical features of dementia: Impairment of intellectual functions. Impairment of memory, predominantly of recent memory, especially in early stages. Deterioration of personality, with lack of personal care. Emotional lability: marked variation in emotional expression. Catastrophic reaction: when confronted with an assignment which is beyond the residual intellectual capacity, patient may go into a sudden rage. Thought abnormalities: delusions, perseverations. Urinary and faecal incontinence may develop in later stages. Disorientation in time, place and person may also develop in later stages. Neurological signs may or may not be present, depending upon the cause. A middle aged female presents for the past 6 months of decreased sleep and withdrawn attitude and complains of her neighbours putting a camera behind her to watch her every step. She is most likely suffering from a) Psychotic depression b) Paranoid schizophrenia c) Delusional disorder d) Major depressive disorder a) Psychotic depression

Ans:

In the clinical situation given above, the h/o decreased sleep and withdrawn attitude suggests depression. In addition, the delusion of her neighbours spying upon her suggestes psychosis. Hence, the best answer in this case would be option a, i.e. psychotic depression. Ref: Kaplan and Sadocks Synopsis of Psychiatry; 10/e, pg 534

Features of major depressive disorder: 1. Depressed mood for most of the day, nearly everyday 2. Markedly diminished interest or pleasure in all, or almost all, activities for most of the day, nearly everyday 3. Significant weight loss when not dieting or weight gain (a change of > 5% of body weight) 4. Insomnia or hypersomnia nearly everyday 5. Psychomotor agitation or retardation nearly everyday 6. Fatigue or loss of energy nearly every day 7. Feelings of worthlessness or excessive or inappropriate guilt nearly every day 8. Diminished ability to think or concentrate, or indecisiveness, nearly every day 9. Recurrent thoughts of death or recurrent suicidal ideation without a specific plan or a specific plan for commiting suicide or suicide attempt. Ref: Kaplan and Sadocks Synopsis of Psychiatry; 10/e, pg 536-538

Features of psychotic depression: The presence of psychotic features in a major depressive disorder reflects severe disease and is a poor prognostic indicator. The psychotic symptoms are often categorized as Mood-congruent (in harmony with the mood disorder) such as those of Guilt Disease Death Nihilism or Deserved punishment Mood-incongruent (not in harmony with the mood disorder) such as those of Persecutory delusions Thought insertion Thought broadcasting and Control 291. Which of the following symptoms is highly suggestive of a diagnosis of mania? a) Thought broadcasting b) Echolalia c) Perseveration d) Flight of ideas

25853434, 42433051, 9873314110, 9953550295, 8447461114

135

DELHI ACADEMY OF MEDICAL SCIENCES PVT. LTD.

MAHARASTRA PAPER - 2011

Ans: Ref:

d) Flight of ideas Niraj Ahujas A Short Textbook of Psychiatry; 6/e, pg 75 Speech and thought characteristics in mania: The person is more talkative than usual; describes thoughts racing in his mind, develops pressure of speech, uses playful language with punning, rhyming, joking and teasing; and speaks loudly. Later, there is flight of ideas (rapidly produced speech with abrupt shifts from topic to topic, using external environmental cues). Typically, the connections between the shifts are apparent. When the flight becomes severe, incoherence may occur. A less severe and more ordered flight, in the absence of pressure of speech is called prolixity. Early morning awakening and disturbance of REM sleep is characteristic of a) Mania b) Major depressive disorder c) Hypomania d) Insomnia b) Major depressive disorder http://en.wikipedia.org/wiki/Melancholic_depression

292.

Ans: Ref:

Melancholic depression: Melancholic depression, or 'depression with melancholic features' is a subtype of major depression characterized by major depressive disorder with the following specific features: Anhedonia (the inability to find pleasure in positive things) Severe weight loss Psychomotor agitation or retardation Insomnia with early morning awakenings Guilt. Roughly 10% of people with depression suffer from melancholic depression Another feature is diurnal variation, typically with worse symptoms in the morning and improvement at night.

[Note: Insomnia per se is not a separate psychiatric disorder. It occurs as a symptom of other disorders, most commonly depression] Also refer to Q. 290 for features of major depressive disorder. 293. A Sri Lankan female after suffering severe losses in a Tsunami still complains of recollecting those events and inability to forget them with marked anxiety. She is most likely suffering from a) Somatization disorder b) Schizophreniform disorder c) Post-traumatic stress disorder d) Major depressive disorder c) Post-traumatic stress disorder Niraj Ahujas A Short Textbook of Psychiatry; 6/e, pg 120 Post-traumatic stress disorder (PTSD) It arises as a delayed and/or protracted response to an exceptionally stressful or catastrophic life event or situation, which is likely to cause pervasive distress in almost any person (e.g. disasters, war, rape or torture, serious accident). The symptoms of PTSD may develop, after a period of latency, within six months after the stress or may be delayed beyond this period. PTSD is characterized by recurrent and intrusive recollections of the stressful event, either in flashbacks (images, thoughts or perceptions) and/or in dreams. There is an associated sense of re-experiencing of the stressful event. There is marked avoidance of the events and situations that arouse recollections of the stressful event, with marked symptoms of anxiety and increased arousal. The other important clinical features include partial amnesia for some aspects of the stressful event, feeling of numbness and anhedonia. Treatment: Prevention: Anticipation of disasters in the high-risk areas, with training of personnel in disaster management. Disaster management: Here the speed of providing help is of parampunt importance.

Ans: Ref:

1. 2.

25853434, 42433051, 9873314110, 9953550295, 8447461114

136

DELHI ACADEMY OF MEDICAL SCIENCES PVT. LTD.


3. 4.

MAHARASTRA PAPER - 2011

Supportive psychotherapy Drug treatment: Antidepressants and benzodiazepines (in low doses for short periods) are useful in treatment, if anxiety and/or depression are important components of the clinical picture. Munchausen syndrome by proxy is characterized by all of these except a) Repeated hospitalizations b) History of abuse by parents c) Intentional production of manifestations d) Repeated use of medications b) History of abuse by parents Niraj Ahujas A Short Textbook of Psychiatry; 6/e, pg 131 Munchausen syndrome: Munchausen syndrome (also known variously as hospital addiction, hospital hoboes or professional patients) is used for those patients who repeatedly simulate or fake diseases for the sole purpose of obtaining medical attention. There is no other motive. (Hence, it is different from malingering) The patients distort their clinical history, laboratory tests reports, and even facts about other aspects of their lives (pseudologia fantastica). Sometimes, they distort physical signs by self-inflicted injuries and secondary infections. Drug abuse, especially abuse of prescription drugs, is common. Evidence of earlier treatment, usually surgical procedures, is often available in the form of multiple scars (grid-iron abdomen). The patients are often manipulative and convincingly tell lies, create problems in the inpatient setting and often leave against medical advice, usually after the surgical procedure has been performed. The prognosis is usually poor and treatment is often unsuccessful. A 6-year a) b) c) d) old boy who presents with communication impairment and repetitive actions is most likely Aspergers syndrome Autistic Learning disorder Rett syndrome

294.

Ans: Ref:

295.

Ans: Ref:

b) Autistic Kaplan and Sadocks Synopsis of Psychiatry; 10/e, pg 1194

DSM-IV Diagnostic Criteria for Autistic disorder A: A total of 6 (or more) items from (1), (2), (3); with at least two from (1) and one each from (2) and (3): (1): (2): (3): (B): Qualitative impairment in social interaction, as manifested by at least two of the following: Marked impairment in the use of multiple non-verbal behaviours such as eye-to-eye gaze, facial expression, body postures and gestures to regulate social interaction Failure to develop peer relationships appropriate to developmental level A lack of spontaneous seeking to share enjoyment, interests or achievements with other people (e.g: by a lack of showing, bringing, or pointing out objects of interest) Lack of social or emotional reciprocity Qualitative impairment in communication, as manifested by at least one of the following: Delay in, or total lack of, the development of spoken language (not accompanied by an attempt to compensate through alternative modes of communication such as gesture or mime) In individuals with adequate speech, marked impairment in the ability to initiate or sustain a conversation with others Stereotyped and repetitive use of language or idiosyncratic language Lack of varied, spontaneous make-believe play or social imitative play appropriate to developmental level Restricted repetitive and stereotyped patterns of behavior, interests and activities, as manifested by at least one of the following: Encompassing preoccupation with one or more stereotyped and restricted patterns of interest that is abnormal in intensity or focus Apparently inflexible adherence to specific non-functional routines or rituals Stereotyped and repetitive motor mannerisms, (e.g: hand or finger flapping or twisting, or complex whole body movements) Persistent preoccupation with parts of objects Delays or abnormal functioning in at least one of the following areas, with onset prior to age of 3 years:

25853434, 42433051, 9873314110, 9953550295, 8447461114

137

DELHI ACADEMY OF MEDICAL SCIENCES PVT. LTD.


(1): (2): (3): (C): 296. Social interaction Language as used in social communication Symbolic or imaginative play

MAHARASTRA PAPER - 2011

The disturbance is not better accounted for by Retts disorder or childhood disintegrative disorder. Suppression of unpleasant stimuli by negating the sensory data is known as a) Distortion b) Denial c) Displacement d) Dissociation b) Denial Niraj Ahujas A Short Textbook of Psychiatry; 6/e, pg 221 Denial: Involuntary exclusion of unpleasant or painful reality from conscous awareness.

Ans: Ref:

Option (a): Distortion. Ref: Niraj Ahujas A Short Textbook of Psychiatry; 6/e, pg 221 Distortion: Unconscious gross reshaping of external reality to satisfy inner needs. Option (c): Displacement. Ref: Niraj Ahujas A Short Textbook of Psychiatry; 6/e, pg 221 Displacement: Unconscious shifting of emotions, usually aroused by perceived threat from an unconscious impulse to a less threatening external object which is then felt to be the source of threat. Option (d): Dissociation Ref: Niraj Ahujas A Short Textbook of Psychiatry; 6/e, pg 221 Dissociation: Involuntary splitting or suppression of a mental function or a group of functions from the rest of the personality in a manner that allows expression of forbidden conscious impulses without having any sense of responsibility for actions. 297. Neuroleptic malignant syndrome can be caused by which of these? a) Phenothiazines b) Haloperidol c) Metoclopromide d) All of these

Ans: d) All of these Ref: Goodman & Gilmans Pharmacologic Basis of Therapeutics; 11/e, pg & Niraj Ahujas A Short Textbook of Psychiatry; 6/e, pg 192 Neuroleptic malignant syndrome: Clinical features Catatonia Stupor Fever Unstable blood pressure Raised CPK Myoglobinemia, which can be fatal. Time of maximal risk: Weeks. Can persist for days after stopping neuroleptic. Maximal risk with: Haloperidol (probably) Proposed mechanism: Antagonism of dopamine may contribute. Treatment: Stop neuroleptic immediately. Dantrolene / Bromocriptine may help. Baclofen for rigidity. General supportive care.

25853434, 42433051, 9873314110, 9953550295, 8447461114

138

DELHI ACADEMY OF MEDICAL SCIENCES PVT. LTD.


Occassionally ECT may be required. Antiparkinsonian agents are not effective.

MAHARASTRA PAPER - 2011

Option (a): Phenothiazines & Haloperidol. Ref: Niraj Ahujas A Short Textbook of Psychiatry; 6/e, pg 189 Phenothiazines & Haloperidol are typical antipsychotics (neuroleptics) They block D2-receptors which are mainly present in the mesolimbic system (which is concerned with emotional reactions) and the nigro-striatal system. It is currently believed that the extra-pyramidal side-effects are caused by the blockade of the D2 receptors situated in the nigro-striatal system.

Option (c): Metoclopramide. Ref: Goodman & Gilmans Pharmacologic Basis of Therapeutics; 11/e, pg The major side effects of metoclopramide include extrapyramidal effects, such as those seen with the phenothiazines. Extrapyramidal effects appear to occur more commonly in children and young adults and at higher doses. Like other dopamine antagonists, metoclopramide also can cause galactorrhea by blocking the inhibitory effect of dopamine on prolactin release, but this adverse effect is relatively infrequent in clinical practice. Therapeutic exposure is a form of which of the following therapies? a) b) Biofeedback c) Cognitive d) Behavioural d) Behavioural Kaplan and Sadocks Synopsis of Psychiatry; 10/e, pg 953-956

298.

Ans: Ref:

BEHAVIOUR THERAPY: 1. Systematic desensitization which includes three steps: Relaxation training Heirarchy construction and Desensitizaton of the stimulus 2. Therapeutic-graded exposure 3. Flooding 4. Participant modeling 5. Exposure to stimuli presented in virtual reality 6. Assertiveness training 7 Social skills training 8. Aversion therapy 9. Eye movement desensitization and reprocessing Option (b): Biofeedback: Ref: Kaplan and Sadocks Synopsis of Psychiatry; 10/e, pg 949-952 BIOFEEDBACK THERAPY: 1. Instrumentation Electromyogram (EMG) Galvanic skin response (GSR) gauge Thermistor 2. Relaxation therapy 3. Later Adaptation to Progressive Muscular Relaxation 4. Autogenic training 5. Applied tension 6. Applied relaxation Option (c): Cognitive. Ref: Kaplan and Sadocks Synopsis of Psychiatry; 10/e, pg 958-960 COGNITIVE THERAPY: 1. Eliciting automatic thoughts 2. Testing automatic thoughts 3. Identifying maladaptive assumptions 4. Testing the validity of maladaptive assumptions

25853434, 42433051, 9873314110, 9953550295, 8447461114

139

DELHI ACADEMY OF MEDICAL SCIENCES PVT. LTD.

MAHARASTRA PAPER - 2011

25853434, 42433051, 9873314110, 9953550295, 8447461114

140

You might also like